You are on page 1of 62

1. PEOPLE VS.

BOLANOS
[211 SCRA 262; G.R. NO. 101808; 3 JUL 1992]

Facts: Oscar Pagdalian was murdered in Marble Supply, Balagtas Bulacan. According to Pat.
Rolando Alcantara and Francisco Dayao, deceased was with two companions on the previous
night, one of whom the accused who had a drinking spree with the deceased. When they
apprehended the accused they found the firearm of the deceased on the chair where the
accused was allegedly seated. They boarded accused along with Magtibay, other accused on the
police vehicle and brought them to the police station. While in the vehicle Bolanos admitted that
he killed the deceased. RTC convicted him hence the appeal.

Issue: Whether or Not accused-appellant deprived of his constitutional right to counsel.

Held: Yes. Being already under custodial investigation while on board the police patrol jeep on
the way to the Police Station where formal investigation may have been conducted, appellant
should have been informed of his Constitutional rights under Article III, Section 12 of the 1987
Constitution, more particularly par. 1 and par. 3.

2. PEOPLE VS. JUDGE AYSON


[175 SCRA 216; G.R. NO. 85215; 7 JUL 1989]

Facts: Felipe Ramos was a ticket freight clerk of the Philippine Airlines, assigned at its Baguio
City station. It was alleged that he was involved in irregularities in the sales of plane tickets, the
PAL management notified him of an investigation to be conducted. That investigation was
scheduled in accordance with PAL's Code of Conduct and Discipline, and the Collective
Bargaining Agreement signed by it with the Philippine Airlines Employees' Association (PALEA) to
which Ramos pertained. A letter was sent by Ramos stating his willingness to settle the amount
of P76,000. The findings of the Audit team were given to him, and he refuted that he misused
proceeds of tickets also stating that he was prevented from settling said amounts. He proffered a
compromise however this did not ensue. Two months after a crime of estafa was charged against
Ramos. Ramos pleaded not guilty. Evidence by the prosecution contained Ramos’ written
admission and statement, to which defendants argued that the confession was taken without the
accused being represented by a lawyer. Respondent Judge did not admit those stating that
accused was not reminded of his constitutional rights to remain silent and to have counsel. A
motion for reconsideration filed by the prosecutors was denied. Hence this appeal.

Issue: Whether or Not the respondent Judge correct in making inadmissible as evidence the
admission and statement of accused.

Held: No. Section 20 of the 1987 constitution provides that the right against self-incrimination
(only to witnesses other than accused, unless what is asked is relating to a different crime
charged- not present in case at bar).

This is accorded to every person who gives evidence, whether voluntarily or under compulsion of
subpoena, in any civil, criminal, or administrative proceeding. The right is not to "be compelled
to be a witness against himself.” It prescribes an "option of refusal to answer incriminating
questions and not a prohibition of inquiry." the right can be claimed only when the specific
question, incriminatory in character, is actually put to the witness. It cannot be claimed at any
other time. It does not give a witness the right to disregard a subpoena, to decline to appear
before the court at the time appointed, or to refuse to testify altogether. It is a right that a
witness knows or should know. He must claim it and could be waived.

Rights in custodial interrogation as laid down in miranda v. Arizona: the rights of the accused
include:

1) he shall have the right to remain silent and to counsel, and to be informed of such right.
2) nor force, violence, threat, intimidation, or any other means which vitiates the free will
shall be used against him.
3) any confession obtained in violation of these rights shall be inadmissible in evidence.

The individual may knowingly and intelligently waive these rights and agree to answer or make a
statement. But unless and until such rights and waivers are demonstrated by the prosecution at
the trial, no evidence obtained as a result of interrogation can be used against him.

3. GAMBOA VS. CRUZ


[162 SCRA 642;L-56291; 27 JUN 1988]

Facts:
Petitioner was arrested for vagrancy without a warrant. During a line-up of 5 detainees including
petitioner, he was identified by a complainant to be a companion in a robbery, thereafter he was
charged. Petitioner filed a Motion to Acquit on the ground that the conduct of the line-up, without
notice and in the absence of his counsel violated his constitutional rights to counsel and to due
process. The court denied said motion. Hearing was set, hence the petition.

Issue: Whether or Not petitioner’s right to counsel and due process violated.

Held: No. The police line-up was not part of the custodial inquest, hence, petitioner was not yet
entitled, at such stage, to counsel. He had not been held yet to answer for a criminal offense.
The moment there is a move or even an urge of said investigators to elicit admissions or
confessions or even plain information which may appear innocent or innocuous at the time, from
said suspect, he should then and there be assisted by counsel, unless he waives the right, but
the waiver shall be made in writing and in the presence of counsel.

On the right to due process, petitioner was not, in any way, deprived of this substantive and
constitutional right, as he was duly represented by a counsel. He was accorded all the
opportunities to be heard and to present evidence to substantiate his defense; only that he
chose not to, and instead opted to file a Motion to Acquit after the prosecution had rested its
case. What due process abhors is the absolute lack of opportunity to be heard.

4. PEOPLE VS. BANDULA


[232 SCRA 566; G.R. NO. 89223; 27 MAY 1994]

Facts: Six armed men barged into the compound of Polo Coconut Plantation in Tanjay, Negros
Oriental. The armed men were identified by Security Guard, including accused. Salva and
Pastrano, security guards were hogtied and accused proceeded to the Atty. Garay, counsel of
plantation. They ransacked the place and took with them money and other valuables. Atty. Garay
was killed. Accused-appellant is charged with robbery with homicide along with 3 others who
were acquitted for insufficiency of evidence. Appellant was convicted.

Now, appellant argues that the extrajudicial confessions he and accused Dionanao executed
suffer from constitutional infirmities, hence, inadmissible in evidence considering that they were
extracted under duress and intimidation, and were merely countersigned later by the municipal
attorney who, by the nature of his position, was not entirely an independent counsel nor counsel
of their choice. Consequently, without the extrajudicial confessions, the prosecution is left
without sufficient evidence to convict him of the crime charged.

Issue: Whether or Not extrajudicial confessions of appellant is admissible as evidence against


him.

Held: No. When accused-appellant Bandula and accused Dionanao were investigated
immediately after their arrest, they had no counsel present. If at all, counsel came in only a day
after the custodial investigation with respect to accused Dionanao, and two weeks later with
respect to appellant Bandula. And, counsel who supposedly assisted both accused was Atty.
Ruben Zerna, the Municipal Attorney of Tanjay. On top of this, there are telltale signs that
violence was used against the accused. Certainly, these are blatant violations of the Constitution
which mandates in
Sec. 12, Art. III. Irregularities present include:

1. The investigators did not inform the accused of their right to remain silent and to have
competent and independent counsel, preferably of their own choice, even before
attempting to elicit statements that would incriminate them.
2. Investigators continuously disregard the repeated requests of the accused for medical
assistance. Reason for Accused Sedigo’s "black eye" which even
Pat. Baldejera admitted is not established, as well as Bandula’s fractured rib.
3. Counsel must be independent. He cannot be a special counsel, public or private
prosecutor, counsel of the police, or a municipal attorney whose interest is admittedly
adverse to the accused.

5. PEOPLE VS. DY
[158 SCRA 111; G.R. 74517; 23 FEB 1988]

Facts: Pat. Padilla reported along with Benny Dy, with caliber .38 as suspect to the shooting
incident at "Benny's Bar," at Sitio Angol, Manoc-Manoc Malay, Aklan (Boracay) situated on the
Island which caused the death of Christian Langel Philippe, tourist, 24 years old and a Swiss
nationale. He was charged with the Murder With the Use of Unlicensed firearms. Appellant
alleges that he carried the victim to the shore to be brought to the hospital to save the latter,
and who facilitated the surrender to Pat. Padilla a gun which his helper found the following
morning while cleaning the bar. Accused posted bail which was granted. The accused denied
having made any oral confession alleging that he went to Pat. Padilla not to report the incident
but to state that a boy helper in the bar had found a gun on the sand floor while cleaning and
that Pat. Padilla picked up the gun from the bar at his request. The Accused argues that even if
he did make such a confession, the same would be inadmissible in evidence. He was found guilty
in the RTC. Hence the appeal.

Issue: Whether or Not the lower court correct in saying that the constitutional procedure on
custodial interrogation is not applicable in the instant case.

Held: YES. Appellant's assertion that the gun he had surrendered was merely found by a boy
helper while cleaning the bar deserves no credence for, if it were so, it would have been absurd
for him to have placed himself under police custody in the early morning after the incident.
Sworn Complaint for "Murder with Use of Unlicensed Firearm" signed by the Chief of Police also
attests to Appellant's oral confession. That Complaint forms part of the record of the proceedings
before the Municipal Circuit Trial Court of Buruanga, Aklan, and is prima facie evidence of the
facts therein stated. Appellant's voluntary surrender implies no violation as "no warrant of arrest
is issued for the apprehension of the accused for the reason that he is already under police
custody before the filing of the complaint." What was told by the Accused to Pat, Padilla was a
spontaneous statement not elicited through questioning, but given in ordinary manner. No
written confession was sought to be presented in evidence as a result of formal custodial
investigation.

6. PEOPLE VS. ALICANDO


[251 SCRA 293; G.R. NO. 117487; 2 DEC 1995]

Facts: Appellant was charged with the crime of rape with homicide of Khazie Mae Penecilla, a
minor, four years of age, choking her with his right hand. The incident happened after appellant
drank liquor. A neighbor, Leopoldo Santiago found the victim’s body and the parents and police
were informed. Appellant was living in his uncle's house some five arm's length from Penecilla's
house. Appellant was arrested and interrogated by PO3 Danilo Tan. He verbally confessed his
guilt without the assistance of counsel. On the basis of his uncounselled verbal confession and
follow up interrogations, the police came to know and recovered from appellant's house, Khazie
Mae's green slippers, a pair of gold earrings, a buri mat, a stained pillow and a stained T-shirt all
of which were presented as evidence for the prosecution. He was arraigned with the assistance
of Atty. Rogelio Antiquiera of the PAO. Appellant pleaded guilty. The RTC convicted him. Hence
an automatic review for the imposition of death penalty.

Issue: Whether or Not the death penalty proper.

Held: No. The records do not reveal that the Information against the appellant was read in the
language or dialect known to him. The Information against the appellant is written in the English
language. It is unknown whether the appellant knows the English language. Neither is it known
what dialect is understood by the appellant. Nor is there any showing that the Information
couched in English was translated to the appellant in his own dialect before his plea of guilt. The
RTC violated section 1(a) of Rule 116, the rule implementing the constitutional right of the
appellant to be informed of the nature and cause of the accusation against him. It also denied
appellant his constitutional right to due process of law. It is urged that we must presume that the
arraignment of the appellant was regularly conducted. When life is at stake, we cannot lean on
this rebuttable presumption. There could be no presumption. The court must be sure.

The trial court violated section 3 of Rule 116 when it accepted the plea of guilt of the appellant.
Said section requires that the court shall conduct a searching inquiry the voluntariness and full
comprehension of the consequences of his plea and require the prosecution to prove his guilt
and the precise degree of culpability. The accused may also present evidence in his behalf. The
trial court simply inquired if appellant had physical marks of maltreatment. It did not ask the
appellant when he was arrested, who arrested him, how and where he was interrogated, whether
he was medically examined before and after his interrogation, etc. It limited its efforts trying to
discover late body marks of maltreatment as if involuntariness is caused by physical abuse
alone.

Further, there are physical evidence to prove Khazie was raped. These consists of a pillow with
bloodstains in its center 14 and the T-shirt 15 of the accused colored white with bloodstains on
its bottom. These physical evidence are evidence of the highest order. They strongly corroborate
the testimony of Luisa Rebada that the victim was raped.These are inadmissible evidence for
they were gathered by PO3 Danilo Tan of the Iloilo City PNP as a result of custodial interrogation
where appellant verbally confessed to the crime without the benefit of counsel.

7. COMMENDADOR VS. DE VILLA


[200 SCRA 80; G.R. NO. 93177; 2 AUG 1991]

Facts: The petitioners in G.R. Nos. 93177 and 96948 who are officers of the AFP were directed to
appear in person before the Pre-Trial Investigating Officers for the alleged participation the failed
coup on December 1 to 9, 1989. Petitioners now claim that there was no pre-trial investigation of
the charges as mandated by Article of War 71. A motion for dismissal was denied. Now, their
motion for reconsideration. Alleging denial of due process.

In G.R. No. 95020, Ltc Jacinto Ligot applied for bail on June 5, 1990, but the application was
denied by GCM No.14. He filed with the RTC a petition for certiorari and mandamus with prayer
for provisional liberty and a writ of preliminary injunction. Judge of GCM then granted the
provisional liberty. However he was not released immediately. The RTC now declared that even
military men facing court martial proceedings can avail the right to bail.

The private respondents in G.R. No. 97454 filed with SC a petition for habeas corpus on the
ground that they were being detained in Camp Crame without charges. The petition was referred
to RTC. Finding after hearing that no formal charges had been filed against the petitioners after
more than a year after their arrest, the trial court ordered their release.

Issue: Whether or Not there was a denial of due process.

Whether or not there was a violation of the accused right to bail.

Held: NO denial of due process. Petitioners were given several opportunities to present their
side at the pre-trial investigation, first at the scheduled hearing of February 12, 1990, and then
again after the denial of their motion of February 21, 1990, when they were given until March 7,
1990, to submit their counter-affidavits. On that date, they filed instead a verbal motion for
reconsideration which they were again asked to submit in writing. They had been expressly
warned in the subpoena that "failure to submit counter-affidavits on the date specified shall be
deemed a waiver of their right to submit controverting evidence." Petitioners have a right to pre-
emptory challenge. (Right to challenge validity of members of G/SCM)

It is argued that since the private respondents are officers of the Armed Forces accused of
violations of the Articles of War, the respondent courts have no authority to order their release
and otherwise interfere with the court-martial proceedings. This is without merit. * The Regional
Trial Court has concurrent jurisdiction with the Court of Appeals and the Supreme Court over
petitions for certiorari, prohibition or mandamus against inferior courts and other bodies and on
petitions for habeas corpus and quo warranto.

The right to bail invoked by the private respondents has traditionally not been recognized and is
not available in the military, as an exception to the general rule embodied in the Bill of Rights.
The right to a speedy trial is given more emphasis in the military where the right to bail does not
exist.

On the contention that they had not been charged after more than one year from their arrest,
there was substantial compliance with the requirements of due process and the right to a speedy
trial. The AFP Special Investigating Committee was able to complete the pre-charge investigation
only after one year because hundreds of officers and thousands of enlisted men were involved in
the failed coup.

Accordingly, in G.R. No. 93177, the petition is dismissed for lack of merit. In G.R. No. 96948, the
petition is granted, and the respondents are directed to allow the petitioners to exercise the right
of peremptory challenge under article 18 of the articles of war. In G.R. Nos. 95020 and 97454,
the petitions are also granted, and the orders of the respondent courts for the release of the
private respondents are hereby reversed and set aside. No costs.

8. PEOPLE VS. JUDGE DONATO


[198 SCRA 130; G.R. NO.79269; 5 JUN 1991]

Facts: Private respondent and his co-accused were charged of rebellion on October 2, 1986 for
acts committed before and after February 1986. Private respondent filed with a Motion to Quash
alleging that: (a) the facts alleged do not constitute an offense; (b) the Court has no jurisdiction
over the offense charged; (c) the Court has no jurisdiction over the persons of the defendants;
and (d) the criminal action or liability has been extinguished. This was denied. May 9, 1987
Respondent filed a petition for bail, which was opposed that the respondent is not entitled to bail
anymore since rebellion became a capital offense under PD 1996, 942 and 1834 amending ART.
135 of RPC. On 5 June 1987 the President issued Executive Order No. 187 repealing, among
others, P.D. Nos. 1996, 942 and 1834 and restoring to full force and effect Article 135 of the
Revised Penal Code as it existed before the amendatory decrees. Judge Donato now granted the
bail, which was fixed at P30,000.00 and imposed a condition that he shall report to the court
once every two months within the first ten days of every period thereof. Petitioner filed a
supplemental motion for reconsideration indirectly asking the court to deny bail to and to allow it
to present evidence in support thereof considering the "inevitable probability that the accused
will not comply with this main condition of his bail. It was contended that:

1. The accused has evaded the authorities for thirteen years and was an escapee from
detention when arrested; (Chairman of CPP-NPA)
2. He was not arrested at his residence as he had no known address;
3. He was using the false name "Manuel Mercado Castro" at the time of his arrest and
presented a Driver's License to substantiate his false identity;
4. The address he gave "Panamitan, Kawit, Cavite," turned out to be also a false address;
5. He and his companions were on board a private vehicle with a declared owner whose
identity and address were also found to be false;
6. Pursuant to Ministry Order No. 1-A dated 11 January 1982 , a reward of P250,000.00 was
offered and paid for his arrest.

This however was denied. Hence the appeal.

Issue: Whether or Not the private respondent has the right to bail.

Held: Yes. Bail in the instant case is a matter of right. It is absolute since the crime is not a
capital offense, therefore prosecution has no right to present evidence. It is only when it is a
capital offense that the right becomes discretionary. However it was wrong for the Judge to
change the amount of bail from 30K to 50K without hearing the prosecution.

Republic Act No. 6968 approved on 24 October 1990, providing a penalty of reclusion perpetua
to the crime of rebellion, is not applicable to the accused as it is not favorable to him.

Accused validly waived his right to bail in another case(petition for habeas corpus). Agreements
were made therein: accused to remain under custody, whereas his co-detainees Josefina Cruz
and Jose Milo Concepcion will be released immediately, with a condition that they will submit
themselves in the jurisdiction of the court. Said petition for HC was dismissed. Bail is the security
given for the release of a person in custody of the law. Ergo, there was a waiver. We hereby rule
that the right to bail is another of the constitutional rights which can be waived. It is a right which
is personal to the accused and whose waiver would not be contrary to law, public order, public
policy, morals, or good customs, or prejudicial to a third person with a right recognized by law.

9.PEOPLE VS. FORTES


[223 SCRA 619; G.R. NO. 90643; 25 JUN 1993]

Facts: Agripino Gine of Barangay Naburacan, Municipality of Matnog, Province of Sorsogon,


accompanied his 13-year old daughter, Merelyn, to the police station of the said municipality to
report a rape committed against the latter by the accused. Following this, the accused was
apprehended and charged. A bond of P25000 was granted for accused’s provisional release. The
MCTC found him guilty. An appeal to RTC was filed, the request for the fixing of bond was denied.
Now accused assails denial of bail on the ground that the same amounted to an undue denial of
his constitutional right to bail.

Issue: Whether or Not the accused’s right to bail violated.

Held: No. It is clear from Section 13, Article III of the 1987 Constitution and Section 3, Rule 114
of the Revised Rules of Court, as amended, that before conviction bail is either a matter of right
or of discretion. It is a matter of right when the offense charged is punishable by any penalty
lower than reclusion perpetua. To that extent the right is absolute. If the offense charged is
punishable by reclusion perpetua bail becomes a matter of discretion. It shall be denied if the
evidence of guilt is strong. The court's discretion is limited to determining whether or not
evidence of guilt is strong. But once it is determined that the evidence of guilt is not strong, bail
also becomes a matter of right. If an accused who is charged with a crime punishable by
reclusion perpetua is convicted by the trial court and sentenced to suffer such a penalty, bail is
neither a matter of right on the part of the accused nor of discretion on the part of the court.

10. MANOTOC VS. COURT OF APPEALS


[142 SCRA 149; G.R. NO. L-62100; 30 MAY 1986]

Facts: Petitioner was charged with estafa. He posted bail. Petitioner filed before each of the trial
courts a motion entitled, "motion for permission to leave the country," stating as ground therefor
his desire to go to the United States, "relative to his business transactions and opportunities."
The prosecution opposed said motion and after due hearing, both trial judges denied the same.
Petitioner thus filed a petition for certiorari and mandamus before the then Court of Appeals
seeking to annul the orders dated March 9 and 26, 1982, of Judges Camilon and Pronove,
respectively, as well as the communication-request of the Securities and Exchange Commission,
denying his leave to travel abroad. He likewise prayed for the issuance of the appropriate writ
commanding the Immigration Commissioner and the Chief of the Aviation Security Command
(AVSECOM) to clear him for departure. The Court of Appeals denied the petition.
Petitioner contends that having been admitted to bail as a matter of right, neither the courts
which granted him bail nor the Securities and Exchange Commission which has no jurisdiction
over his liberty could prevent him from exercising his constitutional right to travel.

Issue: Whether or Not the right to bail a matter of right.

Held: The right to bail is a matter of right if the charge is not a capital offense or punishable by
reclusion perpetua to death.

A court has the power to prohibit a person admitted to bail from leaving the Philippines. This is a
necessary consequence of the nature and function of a bail bond.

The condition imposed upon petitioner to make himself available at all times whenever the court
requires his presence operates as a valid restriction on his right to travel.

Indeed, if the accused were allowed to leave the Philippines without sufficient reason, he may be
placed beyond the reach of the courts.

Petitioner has not shown the necessity for his travel abroad. There is no indication that the
business transactions cannot be undertaken by any other person in his behalf.

11. DEFENSOR-SANTIAGO VS. VASQUEZ


[217 SCRA 633; G.R. NOS. 99289-90; 27 JAN 1993]

Facts: An information was filed against petitioner with the Sandiganbayan for violation of the
Anti Graft and Corrupt Practices Act. The order of arrest was issued with bail for release fixed at
Php. 15,000 so she filed a motion for acceptance of cash bail bond. On the same day the
Sandiganbayan issued a resolution authorizing the petitioner to post cash bond which the later
filed in the amount of Php.15, 000. Her arraignment was set, but petitioner asked for the
cancellation of her bail bond and that she be allowed provisional release on recognizance. The
Sandiganbayan deferred it. The Sandiganbayan issued a hold departure order against petitioner,
by reason of the announcement she made that she would be leaving for the U.S. to accept a
fellowship a Harvard. In the instant motion she submitted before the S.C. she argues that her
right to travel is impaired.

Issue: Whether or Not the petitioner’s right to travel is impaired.

Held: The petitioner does not deny and as a matter of fact even made a public statement, that
she has every intension of leaving the country to pursue higher studies abroad. The court
upholds the course of action of the Sandiganbayan in taking judicial notice of such fact of
petitioners pal to go abroad and in thereafter issuing a sua sponte the hold departure order is
but an exercise of respondent court’s inherent power to preserve and to maintain effectiveness
of its jurisdiction over the case and the person of the accused.

Also, the petitioner assumed obligations, when she posted bail bond. She holds herself amenable
at all times to the orders and process of the court. She may legally be prohibited from leaving
the country during the pendency of the case. (Manotoc v. C.A.)

12. PEOPLE VS. DRAMAYO


[42 SCRA 60; G.R. L-21325; 29 OCT 1971]

Facts: Dramayo brought up the idea of killing Estelito Nogaliza so that he could not testify in the
robbery case where he is an accused. The idea was for Dramayo and Ecubin to ambush Estelito,
who was returning from Sapao. The others were to station themselves nearby. Only Dramayo
and Ecubin were convicted in the RTC for murder. Hence the appeal

Issue: Whether or not the accused’s criminal liability proved beyond reasonable doubt.

Held: Yes. It is to be admitted that the starting point is the Presumption of innocence. So it must
be, according to the Constitution. That is a right safeguarded both appellants. Accusation is not,
according to the fundamental law, synonymous with guilt. It is incumbent on the prosecution
demonstrate that culpability lies. Appellants were not even called upon then to offer evidence on
their behalf. Their freedom is forfeit only if the requisite quantum of proof necessary for
conviction be in existence. Their guilt be shown beyond reasonable doubt. What is required then
is moral certainty. "By reasonable doubt is meant that which of possibility may arise, but it is
doubt engendered by an investigation of the whole proof and an inability, after such
investigation, to let the mind rest easy upon the certainty of guilt. Absolute certain of guilt is not
demanded by the law to convict of any carnal charge but moral certainty is required, and this
certainty is required as to every proposition of proof regular to constitute the offense."

The judgment of conviction should not have occasioned any surprise on the part of the two
appellants, as from the evidence deserving of the fullest credence, their guilt had been more
than amply demonstrated. The presumption of innocence could not come to their rescue as it
was more than sufficiently overcome by the proof that was offered by the prosecution. The
principal contention raised is thus clearly untenable. It must be stated likewise that while
squarely advanced for the first time, there had been cases where this Court, notwithstanding a
majority of the defendants being acquitted, the element of conspiracy likewise being allegedly
present, did hold the party or parties, responsible for the offense guilty of the crime charged, a
moral certainty having arisen as to their capability.

13. DUMLAO VS. COMELEC


[95 SCRA 392; L-52245; 22 JAN 1980]

Facts: Petitioner Dumlao questions the constitutionality of Sec. 4 of Batas Pambansa Blg 52 as
discriminatory and contrary to equal protection and due process guarantees of the Constitution.
Sec. 4 provides that any retired elective provicial or municipal official who has received
payments of retirement benefits and shall have been 65 years of age at the commencement of
the term of office to which he seeks to be elected, shall not be qualified to run for the same
elective local office from which he has retired. According to Dumlao, the provision amounts to
class legislation. Petitioners Igot and Salapantan Jr. also assail the validity of Sec. 4 of Batas
Pambansa Blg 52, which states that any person who has committed any act of disloyalty to the
State, including those amounting to subversion, insurrection, rebellion, or other similar crimes,
shall not be qualified for any of the offices covered by the act, or to participate in any partisan
activity therein: provided that a judgment of conviction of those crimes shall be conclusive
evidence of such fact and the filing of charges for the commission of such crimes before a civil
court or military tribunal after preliminary investigation shall be prima facie evidence of such
fact.

Issue: Whether or not the aforementioned statutory provisions violate the Constitution and thus,
should be declared null and void

Held: In regards to the unconstitutionality of the provisions, Sec. 4 of BP Blg 52 remains


constitutional and valid. The constitutional guarantee of equal protection of the laws is subject to
rational classification. One class can be treated differently from another class. In this case,
employees 65 years of age are classified differently from younger employees. The purpose of the
provision is to satisfy the “need for new blood” in the workplace. In regards to the second
paragraph of Sec. 4, it should be declared null and void for being violative of the constitutional
presumption of innocence guaranteed to an accused. “Explicit is the constitutional provision that,
in all criminal prosecutions, the accused shall be presumed innocent until the contrary is proved,
and shall enjoy the right to be heard by himself and counsel (Article IV, section 19, 1973
Constitution). An accusation, according to the fundamental law, is not synonymous with guilt.
The challenged proviso contravenes the constitutional presumption of innocence, as a candidate
is disqualified from running for public office on the ground alone that charges have been filed
against him before a civil or military tribunal. It condemns before one is fully heard. In ultimate
effect, except as to the degree of proof, no distinction is made between a person convicted of
acts of dislotalty and one against whom charges have been filed for such acts, as both of them
would be ineligible to run for public office. A person disqualified to run for public office on the
ground that charges have been filed against him is virtually placed in the same category as a
person already convicted of a crime with the penalty of arresto, which carries with it the
accessory penalty of suspension of the right to hold office during the term of the sentence (Art.
44, Revised Penal Code).”

And although the filing of charges is considered as but prima facie evidence, and therefore, may
be rebutted, yet. there is "clear and present danger" that because of the proximity of the
elections, time constraints will prevent one charged with acts of disloyalty from offering contrary
proof to overcome the prima facie evidence against him.
Additionally, it is best that evidence pro and con of acts of disloyalty be aired before the Courts
rather than before an administrative body such as the COMELEC. A highly possible conflict of
findings between two government bodies, to the extreme detriment of a person charged, will
thereby be avoided. Furthermore, a legislative/administrative determination of guilt should not
be allowed to be substituted for a judicial determination.
Being infected with constitutional infirmity, a partial declaration of nullity of only that
objectionable portion is mandated. It is separable from the first portion of the second paragraph
of section 4 of Batas Pambansa Big. 52 which can stand by itself.

Wherefore, the first paragraph of section 4 of Batas pambansa Bilang 52 is hereby declared valid
and that portion of the second paragraph of section 4 of Batas Pambansa Bilang 52 is hereby
declared null and void, for being violative of the constitutional presumption of innocence
guaranteed to an accused.

14. CORPUZ VS. REPUBLIC


[194 SCRA 73; G.R. NO. 74259; 14 FEB 1991]

Facts: Generoso Corpuz is the Supervising Accounting Clerk in the Office of the Provincial
Treasurer of Nueva Viscaya. He was designated Acting Supervising Cashier in the said office. In
this capacity, he received collections, disbursed funds and made bank deposits and withdrawals
pertaining to government accounts. On April 13, 1981 his designation as Acting Supervising
Cashier was terminated and a transfer of accountabilities was effected between him and his
successor. The Certificate of turnover revealed a shortage of P72,823.00. He was able to pay
only P10,159.50. After a final demand letter for the total of P50,596.07 which was not met, a
case of malversation was filed against him. Corpuz did not deny such facts but he insists that
the shortage was malversed by other persons. He alleged that Paymaster Diosdado Pineda
through 1 of 4 separate checks (PNB) issued and encashed such checks while he was of leave.
Also, Acting Deputy Provincial Treasurer Bernardo Aluning made to post the amount on his
cashbook although he had not received the said amount. He was convicted in Sandiganbayan.

Issue: Whether or Not Corpuz is guilty of malversation.

Held: It is a subtle way of camouflaging the embezzlement of the money equivalent when 1 of
the 4 checks issued and encashed in the same day was entered in the accused’s cash book 3
months after such encashments. Also, Corpuz claim that he was absent when Paymaster
Diosdado Pineda through 1 of 4 separate checks (PNB) issued and encashed such checks, was
not proven.

Post-Audit is not a preliminary requirement to filing a malversation case. The failure of the public
officer to have duly forthcoming any public funds with which he is chargeable, upon demand by
an authorized officer shall be a prima facie evidence that he has put such missing funds to
personal use.

The equipoise rule(balancing test) which is the presumption of innocence is applicable only
where the evidence of the parties is evenly balance, in which case the scale of justice should be
tilt in favor of the accused. There is no such balance in the case at bar. The evidence of the
prosecution is overwhelming and has not been overcome by the petitioner with his claims. The
presumed innocence must yield to the positive finding that he is guilty of malversation.

Wherefore his petition is denied. He is guilty as principal of Malversation of Public Funds.

15. PEOPLE VS. HOLGADO


[85 PHIL 752; G.R.L-2809; 22 MAR 1950]

Facts: Appellant Frisco Holgado was charged in the court of First Instance of Romblon with slight
illegal detention because according to the information, being a private person, he did
"feloniously and without justifiable motive, kidnap and detain one Artemia Fabreag in the house
of Antero Holgado for about eight hours thereby depriving said Artemia Fabreag of her personal
liberty. He pleaded guilty (without a counsel) and said that he was just instructed by Mr.
Ocampo, which no evidence was presented to indict the latter.

Issue: Whether or Not there was any irregularity in the proceedings in the trial court.

Held: Yes. Rule 112, section 3 of ROC that : “If the defendant appears without attorney, he must
be informed by the court that it is his right to have attorney being arraigned., and must be asked
if he desires the aid of attorney, the Court must assign attorney de oficio to defend him. A
reasonable time must be allowed for procuring attorney.” This was violated. Moreso the
guarantees of our Constitution that "no person shall be held to answer for a criminal offense
without due process of law", and that all accused "shall enjoy the right to be heard by himself
and counsel." In criminal cases there can be no fair hearing unless the accused be given the
opportunity to be heard by counsel.

The trial court failed to inquire as to the true import of the qualified plea of accused. The record
does not show whether the supposed instructions of Mr. Ocampo was real and whether it had
reference to the commission of the offense or to the making of the plea guilty. No investigation
was opened by the court on this matter in the presence of the accused and there is now no way
of determining whether the supposed instruction is a good defense or may vitiate the
voluntariness of the confession. Apparently the court became satisfied with the fiscal's
information that he had investigated Mr. Ocampo and found that the same had nothing to do
with this case. Such attitude of the court was wrong for the simple reason that a mere statement
of the fiscal was not sufficient to overcome a qualified plea of the accused. But above all, the
court should have seen to it that the accused be assisted by counsel especially because of the
qualified plea given by him and the seriousness of the offense found to be capital by the court.

16. PEOPLE VS. MAGSI


[124 SCRA 64; G.R. NO.L-32888; 12 AUG 1983]

Facts: Soon after appellant was apprehended on August 20, 1970, his arraignment was
scheduled before the Criminal Circuit Court of San Fernando, La Union. The case was actually set
and rescheduled for six (6) times, first of which was on August 1, 1970. On that date, despite
appointment by the court of Atty. Mario Rivera as de officio counsel for the accused, hearing was
re-set to September 8, 1970 on motion of Atty. Rivera, who was prompted to ask for it because
of accused desire to be represented by a de parte counsel. Prior to the next hearing, Atty. Rivera
moved to withdraw as de officio counsel and it was favorably acted on by the court on
September 7, 1970. At the second hearing on September 8, 1970, for failure of the de officio and
de parte counsels to appear, despite a second call of the case, the hearing was re-set for the
next day and the court appointed Atty. Dominador Cariaso de officio counsel for the accused. On
the third hearing date, neither the de parte nor the de officio counsel was in Court, so Atty.
Rivera was reappointed that day as de officio counsel for arraignment purposes only. The
accused del Rosario entered a plea of guilty but qualified it with the allegation that he committed
the crime out of fear of his co-accused Eloy Magsi and the other coaccused. Appellant was found
guilty of murder and made to suffer the death penalty.

Issue: Whether or not there was a violation of the rights of the accused.

Held: YES. The desire to speed up the disposition of cases should not be effected at the sacrifice
of the basic rights of the accused. Citing People vs. Domingo (55 SCRA 243-244): the trial courts
should exercise solicitous care before sentencing the accused on a plea of guilty especially in
capital offenses by first insuring that the accused fully understands the gravity of the offense,
the severity of the consequences attached thereto as well as the meaning and significance of his
plea of guilty; and that the prudent and proper thing to do in capital cases is to take testimony,
to assure the court that the accused has not misunderstood the nature and effect of his plea of
guilty. Mere pro-forma appointment of de officio counsel, who fails to genuinely protect the
interests of the accused, resetting of hearing by the court for alleged reception of evidence when
in fact none was conducted, perfunctory queries addressed to the accused whether he
understands the charges and the gravity of the penalty, are not sufficient compliance.

17. BORJA VS. MENDOZA


[77 SCRA 422; G.R. NO.L-45667; 20 JUN 1977]

Facts: Borja was accused of slight physical injuries in the City of Cebu. However, he was not
arraigned. That not withstanding, respondent Judge Senining proceeded with the trial in absentia
and rendered a decision finding petitioner guilty of the crime charged. The case was appealed to
the Court o First Instance in Cebu presided by respondent Judge Mendoza. It was alleged that the
failure to arraign him is a violation of his constitutional rights. It was also alleged that without
any notice to petitioner and without requiring him to submit his memorandum, a decision on the
appealed case was rendered The Solicitor General commented that the decision should be
annulled because there was no arraignment.

Issue: Whether or Not petitioner’s constitutional right was violated when he was not arraigned.

Held: Yes. Procedural due process requires that the accused be arraigned so that he may be
informed as to why he was indicted and what penal offense he has to face, to be convicted only
on a showing that his guilt is shown beyond reasonable doubt with full opportunity to disprove
the evidence against him. It is also not just due process that requires an arraignment. It is
required in the Rules that an accused, for the first time, is granted the opportunity to know the
precise charge that confronts him. It is imperative that he is thus made fully aware of possible
loss of freedom, even of his life, depending on the nature of the crime imputed to him. At the
very least then, he must be fully informed of why the prosecuting arm of the state is mobilized
against him. Being arraigned is thus a vital aspect of the constitutional rights guaranteed him.
Also, respondent Judge Senining convicted petitioner notwithstanding the absence of an
arraignment. With the violation of the constitutional right to be heard by himself and counsel
being thus manifest, it is correct that the Solicitor General agreed with petitioner that the
sentence imposed on him should be set aside for being null. The absence of an arraignment can
be invoked at anytime in view of the requirements of due process to ensure a fair and impartial
trial.

Wherefore, the petition for certiorari is granted. The decision of respondent Judge Romulo R.
Senining dated December 28, 1973, finding the accused guilty of the crime of slight physical
injuries, is nullified and set aside. Likewise, the decision of respondent Judge Rafael T. Mendoza
dated November 16, 1976, affirming the aforesaid decision of Judge Senining, is nullified and set
aside. The case is remanded to the City Court of Cebu for the prosecution of the offense of slight
physical injuries, with due respect and observance of the provisions of the Rules of Court,
starting with the arraignment of petitioner.

18. CONDE VS. RIVERA


[45 PHIL 650; G.R. NO. 21741; 25 JAN 1924]

Facts: Aurelia Conde, formerly a municipal midwife in Lucena, Tayabas, has been forced to
respond to no less the five information for various crimes and misdemeanors, has appeared with
her witnesses and counsel at hearings no less than on eight different occasions only to see the
cause postponed, has twice been required to come to the Supreme Court for protection, and
now, after the passage of more than one year from the time when the first information was filed,
seems as far away from a definite resolution of her troubles as she was when originally charged.

Issue: Whether or Not petitioner has been denied her right to a speedy and impartial trial.

Held: Philippine organic and statutory law expressly guarantee that in all criminal prosecutions
the accused shall enjoy the right to have a speedy trial. Aurelia Conde, like all other accused
persons, has a right to a speedy trial in order that if innocent she may go free, and she has been
deprived of that right in defiance of law. We lay down the legal proposition that, where a
prosecuting officer, without good cause, secures postponements of the trial of a defendant
against his protest beyond a reasonable period of time, as in this instance for more than a year,
the accused is entitled to relief by a proceeding in mandamus to compel a dismissal of the
information, or if he be restrained of his liberty, by habeas corpus to obtain his freedom.

19. PEOPLE VS. SALAS


[143 SCRA 163; G.R. NO. L-66469; 29 JUL 1986]

Facts: At about 6:00 o'clock in the morning of March 6, 1992, a 60 year old woman, identified as
Virginia Talens was found lying dead in a canal at Bo. San Nicolas, Mexico, Pampanga; she was
last seen alive at about 3:00 o'clock early morning of March 6, 1992 by Orlando Pangan and
Richard Pangan who were with her going home coming from the wake of one Leonardo Flores;
both Orlando and Richard Pangan testified that accused was with them in going home at about
3:00 o'clock in the morning of March 6, 1992; Orlando and Richard Pangan reached first their
house and left the two on the way and that was the last time Virginia was seen alive; just a few
minutes after reaching his house and while inside his house, Orlando Pangan heard a shout;
another woman, one Serafia Gutierrez, testified that she likewise was awakened by a shout at
about 3:00 in the morning; Dr. Aguda who autopsied the victim found hematoma on the head
and chest, an abrasion on the left chin and stabwound on the neck which stabwound, the doctor
claims, was the cause of death of the victim; Police Investigator Gonzales who immediately
responded upon report, recovered at the scene a pin, the victim's wristwatch, earring, a ring and
P135.00 money; he likewise found on March 9, 1992 when he continued his investigation
bloodstain on the front door of the house of the accused which bloodstain when submitted for
examination was found to be of human blood; one Resultay was with Virginia Talens at about
5:00 afternoon of March 5, 1992 in going to the wake, who claims that Virginia had money on a
purse as while they were on the way Virginia bet on a jueteng she saw Virginia got money from
her purse a P500.00 bill but as she had no change she instead took P8.00 from her other pocket;
one Ramil Talens, a son of the victim corroborated the claim of Resultay that Virginia had with
her at that time money worth P2,000.00 as in the morning of March 5, 1992 he gave her mother
for safekeeping the sum of P1,500.00 which he claims his mother placed in her purse and claims
further that at the wake, he asked and was given P50.00 by his mother as he also participated in
the gambling thereat, however, the purse of Virginia containing about P2,000.00 was no longer
to be found when she was found dead; Orlando Pangan saw the accused gambled in the wake;
Virginia likewise gambled at the wake; accused had been working for three days before March 6
at Sta. Ana, Pampanga and up to March 5, 1992, but the following day, he did not anymore
report for work at Sta. Ana, Pampanga, was no longer to be found and was last seen at about
3:00 morning together with Virginia Talens on their way home coming from the wake; the
parents of [the] accused were informed by Investigator Gonzales that their son was the suspect
and adviced them to surrender him, but since March 6, 1992 when accused left Mexico,
Pampanga, he returned only on September 19, 1992 at Arayat, Pampanga, not at Mexico,
Pampanga where he was ultimately apprehended by the Mexico Police on September 22, 1992
after chancing on a radio message by the police of Arayat to their Provincial commander that a
vehicular incident occurred at Arayat, Pampanga where one Elmer Salas was the victim and was
hospitalized at the district hospital at Arayat, Pampanga where he used the name of Rommel
Salas and not Elmer Salas. The trial court rendered convicting Salas for Robbery with Homicide

Issue: Whether or Not there is evidence sufficient to sustain a conviction of the appellant of the
crime of Robbery with Homicide.

Whether or Not the appellant’s crime homicide or robbery with homicide.

Held: There was no eyewitness or direct evidence; either to the robbery or to the homicide and
none of the things allegedly stolen were ever recovered. However, direct evidence is not the only
matrix from which the trial court may draw its findings and conclusion of culpability. Resort to
circumstantial evidence is essential when to insist on direct testimony would result in setting
felons free.

For circumstantial evidence to be sufficient to support a conviction, all the circumstances must
be consistent with each other, consistent with the theory that the accused is guilty of the offense
charged, and at the same time inconsistent with the hypothesis that he is innocent and with
every other possible, rational hypothesis excepting that of guilt. All the circumstances
established must constitute an unbroken chain which leads to one and fair and reasonable
conclusion pointing solely to the accused, to the exclusion of all other persons, as the author of
the crime. The facts and circumstances consistent with the guilt of the accused and inconsistent
with his innocence can constitute evidence which, in weight and probative value, may be
deemed to surpass even direct evidence in its effect on the court.

The fatal stabbing of Virginia Talens occurred at around 3:00 a.m. of March 6, 1992. Appellant
hastily abandoned his house in Barrio San Nicolas, Mexico, Pampanga, his residence since
childhood, on that very date. Appellant was nowhere when his co-worker and barrio mate,
Eduardo Bagtas, came to appellant's house to fetch him for work at around 6:30 to 7:00 a.m. of
March 6, 1992. Appellant also abandoned his job as a painter in Sta. Ana, Pampanga, on March 6,
1992, the date of the crime, leaving behind an unfinished painting project. He was not seen
again from said date. Police investigators found human bloodstains on the front door of
appellant's house, on his clothing, and on his yellow slippers after the victim was killed. Despite
efforts of the police to find appellant as the principal suspect, a fact known to appellant's family
and neighbors, appellant did not present himself to the authorities. Appellant was apprehended
only a full six months after the date of the crime, following his confinement in a hospital in
Arayat, Pampanga because he was sideswiped by a Victory Liner bus in Arayat. When
hospitalized, appellant used the alias Rommel Salas, instead of his true name Elmer Salas. These
circumstances denote flight, which when unexplained, has always been considered by the courts
as indicative of guilt.

Both appellant and victim gambled at the wake they attended. The victim was, in fact, enjoying a
winning streak when her son, Ramil Talens, came to fetch her but which he failed to do because
his mother was winning, and she refused to leave. The purse of Talens containing cash was gone
when her corpse was found in the canal with a stab wound and bruises. What was left was a
safety pin which victim used to fasten the missing purse to her clothes.

Denial is an inherently weak defense which must be buttressed by strong evidence of non-
culpability to merit credibility. Denial is negative and self-serving and cannot be given greater
evidentiary weight over the testimonies of credible witnesses who positively testified that
appellant was at the locus criminis and was the last person seen with the victim alive.

The absence of evidence showing any improper motive on the part of the principal witness for
the prosecution to falsely testify against the appellant strongly tends to buttress the conclusion
that no such improper motive exists and that the testimony of said witnesses deserve full faith
and credit.

The essence of voluntary surrender is spontaneity and the intent of the accused to give himself
up and submit himself unconditionally to the authorities either because he acknowledges his
guilt or he wants to save the State the trouble of having to effect his arrest. Spontaneity and
intent to give one's self up are absent where the accused went into hiding for six months after
the incident and had to resort to an alias when he was involved in an accident being investigated
by the police authorities.

Robbery with Homicide is a special complex crime against property. Homicide is incidental to the
robbery which is the main purpose of the criminal. In charging Robbery with Homicide, the onus
probandi is to establish: "(a) the taking of personal property with the use of violence or
intimidation against a person; (b) the property belongs to another; (c) the taking is characterized
with animus lucrandi; and (d) on the occasion of the robbery or by reason thereof, the crime of
homicide, which is used in the generic sense, was committed." Although there was no witness as
to the actual robbing of the victim, there is testimony that the victim had more or less P2,000.00;
and wore gold earrings valued at P750.00. These were never recovered.

While there is indeed no direct proof that Virginia Talens was robbed at the time she was killed,
we may conclude from four circumstances that the robbery occasioned her killing: (1) Both
appellant and victim gambled at the wake. (2) The appellant knew that victim was winning. (3)
The victim was last seen alive with appellant. (4) The victim's purse containing her money and
earrings were missing from her body when found.

The decision of the regional trial court is affirmed. Costs against appellant. So ordered.

20. PEOPLE VS. ACABAL


[226 SCRA 694 ; G.R. NO. 103604, 23 SEP 1993]

Facts: The accusatory portion in the information for murder. Facts are as follows:

"That sometime in the evening of the 28th of January, 1980, at Nagbinlod, Municipality of Sta.
Catalina, Province of Negros Oriental, Philippines, and within the jurisdiction of this Honorable
Court, the accused, including several 'John Does', conspiring and confederating with one another,
with intent to kill, and with treachery and evident premeditation and being then armed with
bolos and 'pinuti', did then and there willfully, unlawfully and feloniously attack, assault and use
personal violence on the person of one Rizalina Apatan Silvano while the latter was about to
leave her house and inflicting upon her injuries, to wit: 'right leg amputated below the knee; left
leg hacked behind the knee; abdomen hacked with viscerae evacerated,' and did then and there
set the house on fire while the aforementioned Rizalina Apatan Silvano was inside said house
trying to escape therefrom, and allowing her to be burned inside said house which was burned to
the ground, thereby causing upon said Rizalina Apatan Silvano her death and burning her
beyond recognition.

But on 16 May 1987, a fire gutted the building where Branch 37 was located and the records of
these two cases were burned. The records were subsequently reconstituted upon petition of the
prosecuting fiscal. The testimonies of the witnesses were retaken, however, before it could
commence, accused Engracio Valeriano jumped bail and the warrant for his arrest issued on 16
November 1987 was returned unserved because he could not be found. An alias warrant for his
arrest was issued on 26 June 1989, but he remains at large up to the present.

After the completion of the re-taking of the testimonies of the witnesses in Branch 37, Criminal
Cases Nos. 4584 and 4585 were re-raffled to Branch 33 of the trial court, then presided over by
Judge Pacifico S. Bulado.

The decision of the trial court, per Judge Pacifico S. Bulado, dated 31 October 1991 but
promulgated on 20 December 1991, contained no specific dispositive portion. Its rulings are
found in the last two paragraphs which read as follows:

"The elements of murder in this case, Criminal Case No. 4585 for the killing of Rizalina Apatan-
Silvano having been proved by the prosecution beyond doubt, the accused JUANITO RISMUNDO,
MACARIO ACABAL and ABUNDIO NAHID, considering the attendant qualifying aggravating
circumstances of nighttime, use of fire by burning the house of victim Rizalina Apatan-Silvano in
order to forcibly drive her out of her house and hack her to death, the abuse of superior strength,
the penalty impossable [sic] here will be in its maximum degree, that is reclusion perpetua
taking into account Article 248 of the Revised Penal Code, the penalty now for murder is
Reclusion Temporal to Reclusion Perpetua, and for all the accused to indemnify the heirs of the
victim the sum of Thirty Thousand (P30,000.00) Pesos since this case occurred [sic] in 1980. For
the wounding of the victim Wilson A. Silvano, this Court believes that simple frustrated homicide
only is committed by the accused Engracio Valeriano only.

But since the person who actually inflicted the injuries of victim Wilson Silvano, accused Engracio
Valeriano only is nowhere to be found, hence, not brought to the bar of justice, he being a
fugitive or at large, no penalty could be imposed on him since he is beyond the jurisdiction of this
court to reach. All the other two (2) accused, JUANITO RISMUNDO and ABUNDIO NAHID are
hereby ordered and declared absolved from any criminal responsibility from frustrated homicide.

The bail bond put up by the three accused, namely: Juanito Rismundo, Macario Acabal and
Abundio Nahid are hereby ordered cancelled and let a warrant of arrest be issued for their
immediate confinement."

Issue: Whether or not the cancellation of the bail bonds of the accused is valid.

Whether or not the accused may be tried in absentia.

Whether or not the accused is guilty of the crime of frustrated murder.

Held: The trial court further erred in holding that no penalty could be imposed on accused
Engracio Valeriano in Criminal Case No. 4584 because he "is nowhere to be found, hence, not
brought to the bar of justice, he being a fugitive or at large." The court ignored the fact that
Engracio jumped bail after he had been arraigned, just before the retaking of evidence
commenced. Paragraph (2), Section 14, Article III of the Constitution permits trial in absentia
after the accused has been arraigned provided he has been duly notified of the trial and his
failure to appear thereat is unjustified. One who jumps bail can never offer a justifiable reason for
his non-appearance during the trial.

Accordingly, after the trial in absentia, the court can render judgment in the case and
promulgation may be made by simply recording the judgment in the criminal docket with a copy
thereof served upon his counsel, provided that the notice requiring him to be present at the
promulgation is served through his bondsmen or warden and counsel.

In conclusion, because of reasonable doubt as to their guilt, the accused-appellants must be


acquitted. Every accused is presumed innocent until the contrary is proved; that presumption is
solemnly guaranteed by the Bill of Rights. The contrary requires proof beyond reasonable doubt,
or that degree of proof which produces conviction in an unprejudiced mind. Short of this, it is not
only the right of the accused to be freed; it is even the constitutional duty of the court to acquit
him.

21. VILLAFLOR VS. SUMMERS


[41 PHIL 62; G.R. NO. 16444; 8 SEP 1920]

Facts: Petitioner Villaflor was charged with the crime of adultery. The trial judge ordered the
petitioner to subject herself into physical examination to test whether or not she was pregnant to
prove the determine the crime of adultery being charged to her. Herein petitioner refused to
such physical examination interposing the defense that such examination was a violation of her
constitutional rights against self-incrimination.

Issue: Whether or Not the physical examination was a violation of the petitioner’s constitutional
rights against self-incrimination.

Held: No. It is not a violation of her constitutional rights. The rule that the constitutional
guaranty, that no person shall be compelled in any criminal case to be a witness against himself,
is limited to a prohibition against compulsory testimonial self-incrimination. The corollary to the
proposition is that, an ocular inspection of the body of the accused is permissible.

22. BELTRAN VS. SAMSON


[53 PHIL 570; G.R. NO. 32025; 23 SEPT 1929]

Facts: Beltran, as a defendant for the crime of Falsification, refused to write a sample of his
handwriting as ordered by the respondent Judge. The petitioner in this case contended that such
order would be a violation of his constitutional right against self-incrimination because such
examination would give the prosecution evidence against him, which the latter should have
gotten in the first place. He also argued that such an act will make him furnish evidence against
himself.

Issue: Whether or not the writing from the fiscal's dictation by the petitioner for the
purpose of comparing the latter's handwriting and determining whether he wrote certain
documents supposed to be falsified, constitutes evidence against himself within the scope and
meaning of the constitutional provision under examination.

Held: The court ordered the respondents and those under their orders desist and abstain
absolutely and forever from compelling the petitioner to take down dictation in his handwriting
for the purpose of submitting the latter for comparison. Writing is something more than moving
the body, or the hands, or the fingers; writing is not a purely mechanical act, because it requires
the application of intelligence and attention; and in the case at bar writing means that the
petitioner herein is to furnish a means to determine whether or not he is the falsifier, as the
petition of the respondent fiscal clearly states. Except that it is more serious, we believe the
present case is similar to that of producing documents or chattels in one's possession. We say
that, for the purposes of the constitutional privilege, there is a similarity between one who is
compelled to produce a document, and one who is compelled to furnish a specimen of his
handwriting, for in both cases, the witness is required to furnish evidence against himself. It
cannot be contended in the present case that if permission to obtain a specimen of the
petitioner's handwriting is not granted, the crime would go unpunished. Considering the
circumstance that the petitioner is a municipal treasurer, it should not be a difficult matter for
the fiscal to obtained genuine specimens of his handwriting. But even supposing it is impossible
to obtain specimen or specimens without resorting to the means complained herein, that is no
reason for trampling upon a personal right guaranteed by the constitution. It might be true that
in some cases criminals may succeed in evading the hand of justice, but such cases are
accidental and do not constitute the raison d' etre of the privilege. This constitutional privilege
exists for the protection of innocent persons.

23. PASCUAL VS. BME


[28 SCRA 345; G.R. NO. 25018; 26 MAY 1969]

Facts: Petitioner Arsenio Pascual, Jr. filed an action for prohibition against the Board of Medical
Examiners. It was alleged therein that at the initial hearing of an administrative case for alleged
immorality, counsel for complainants announced that he would present as his first witness the
petitioner. Thereupon, petitioner, through counsel, made of record his objection, relying on the
constitutional right to be exempt from being a witness against himself. Petitioner then alleged
that to compel him to take the witness stand, the Board of Examiners was guilty, at the very
least, of grave abuse of discretion for failure to respect the constitutional right against self-
incrimination.

The answer of respondent Board, while admitting the facts stressed that it could call petitioner to
the witness stand and interrogate him, the right against self-incrimination being available only
when a question calling for an incriminating answer is asked of a witness. They likewise alleged
that the right against self-incrimination cannot be availed of in an administrative hearing.

Petitioner was sustained by the lower court in his plea that he could not be compelled to be the
first witness of the complainants, he being the party proceeded against in an administrative
charge for malpractice. Hence, this appeal by respondent Board.

Issue: Whether or Not compelling petitioner to be the first witness of the complainants violates
the Self-Incrimination Clause.

Held: The Supreme Court held that in an administrative hearing against a medical practitioner
for alleged malpractice, respondent Board of Medical Examiners cannot, consistently with the
self-incrimination clause, compel the person proceeded against to take the witness stand without
his consent. The Court found for the petitioner in accordance with the well-settled principle that
"the accused in a criminal case may refuse, not only to answer incriminatory questions, but, also,
to take the witness stand." If petitioner would be compelled to testify against himself, he could
suffer not the forfeiture of property but the revocation of his license as a medical practitioner.
The constitutional guarantee protects as well the right to silence: "The accused has a perfect
right to remain silent and his silence cannot be used as a presumption of his guilt." It is the right
of a defendant "to forego testimony, to remain silent, unless he chooses to take the witness
stand — with undiluted, unfettered exercise of his own free genuine will."

The reason for this constitutional guarantee, along with other rights granted an accused, stands
for a belief that while crime should not go unpunished and that the truth must be revealed, such
desirable objectives should not be accomplished according to means or methods offensive to the
high sense of respect accorded the human personality. More and more in line with the
democratic creed, the deference accorded an individual even those suspected of the most
heinous crimes is given due weight. The constitutional foundation underlying the privilege is the
respect a government ... must accord to the dignity and integrity of its citizens.

24. TATAD VS. SANDIGANBAYAN


[159 SCRA 70; G.R. NOS. L-72335-39; 21 MAR 1988]

Facts: The complainant, Antonio de los Reyes, originally filed what he termed "a report" with the
Legal Panel of the Presidential Security Command (PSC) on October 1974, containing charges of
alleged violations of Rep. Act No. 3019 against then Secretary of Public Information Francisco S.
Tatad. The "report" was made to "sleep" in the office of the PSC until the end of 1979 when it
became widely known that Secretary (then Minister) Tatad had a falling out with President
Marcos and had resigned from the Cabinet. On December 12, 1979, the 1974 complaint was
resurrected in the form of a formal complaint filed with the Tanodbayan. The Tanodbayan acted
on the complaint on April 1, 1980 which was around two months after petitioner Tatad's
resignation was accepted by Pres. Marcos by referring the complaint to the CIS, Presidential
Security Command, for investigation and report. On June 16, 1980, the CIS report was submitted
to the Tanodbayan, recommending the filing of charges for graft and corrupt practices against
former Minister Tatad and Antonio L. Cantero. By October 25, 1982, all affidavits and counter-
affidavits were in the case was already for disposition by the Tanodbayan. However, it was only
on June 5, 1985 that a resolution was approved by the Tanodbayan. Five criminal informations
were filed with the Sandiganbayan on June 12, 1985, all against petitioner Tatad alone. (1)
Section 3, paragraph (e) of RA. 3019 for giving D' Group, a private corporation controlled by his
brother-in-law, unwarranted benefits, advantage or preference in the discharge of his official
functions; (2) Violation of Section 3, paragraph (b) for receiving a check of P125,000.00 from
Roberto Vallar, President/General Manager of Amity Trading Corporation as consideration for the
release of a check of P588,000.00 to said corporation for printing services rendered for the
Constitutional Convention Referendum in 1973; (3) Violation of Section 7 on three (3) counts for
his failure to file his Statement of Assets and Liabilities for the calendar years 1973, 1976 and
1978. A motion to quash the information was made alleging that the prosecution deprived
accused of due process of law and of the right to a speedy disposition of the cases filed against
him. It was denied hence the appeal.

Issue: Whether or not petitioner was deprived of his rights as an accused.

Held: YES. Due process (Procedural) and right to speedy disposition of trial were violated. Firstly,
the complaint came to life, as it were, only after petitioner Tatad had a falling out with President
Marcos. Secondly, departing from established procedures prescribed by law for preliminary
investigation, which require the submission of affidavits and counter-affidavits by the
complainant and the respondent and their witnesses, the Tanodbayan referred the complaint to
the Presidential Security Command for finding investigation and report. The law (P.D. No. 911)
prescribes a ten-day period for the prosecutor to resolve a case under preliminary investigation
by him from its termination. While we agree with the respondent court that this period fixed by
law is merely "directory," yet, on the other hand, it can not be disregarded or ignored
completely, with absolute impunity. A delay of close to three (3) years can not be deemed
reasonable or justifiable in the light of the circumstance obtaining in the case at bar.

25. CAUNCA VS. SALAZAR


[82 PHIL 851; NO.L-2690; 1 JAN 1949]

Facts: This is an action for habeas corpus brought by Bartolome Caunca in behalf of his cousin
Estelita Flores who was employed by the Far Eastern Employment Bureau, owned by Julia
Salazar, respondent herein. An advanced payment has already been given to Estelita by the
employment agency, for her to work as a maid. However, Estelita wanted to transfer to another
residence, which was disallowed by the employment agency. Further she was detained and her
liberty was restrained. The employment agency wanted that the advance payment, which was
applied to her transportation expense from the province should be paid by Estelita before she
could be allowed to leave.

Issue: Whether or Not an employment agency has the right to restrain and detain a maid
without returning the advance payment it gave?

Held: An employment agency, regardless of the amount it may advance to a prospective


employee or maid, has absolutely no power to curtail her freedom of movement. The fact that no
physical force has been exerted to keep her in the house of the respondent does not make less
real the deprivation of her personal freedom of movement, freedom to transfer from one place to
another, freedom to choose one’s residence. Freedom may be lost due to external moral
compulsion, to founded or groundless fear, to erroneous belief in the existence of an imaginary
power of an impostor to cause harm if not blindly obeyed, to any other psychological element
that may curtail the mental faculty of choice or the unhampered exercise of the will. If the actual
effect of such psychological spell is to place a person at the mercy of another, the victim is
entitled to the protection of courts of justice as much as the individual who is illegally deprived of
liberty by duress or physical coercion.

26. PEOPLE VS. FERRER


[48 SCRA 382; NOS.L-32613-14; 27 DEC 1972]

Facts: Hon. Judge Simeon Ferrer is the Tarlac trial court judge that declared RA1700 or the Anti-
Subversive Act of 1957 as a bill of attainder. Thus, dismissing the information of subversion
against the following: 1.) Feliciano Co for being an officer/leader of the Communist Party of the
Philippines (CPP) aggravated by circumstances of contempt and insult to public officers,
subversion by a band and aid of armed men to afford impunity. 2.) Nilo Tayag and 5 others, for
being members/leaders of the NPA, inciting, instigating people to unite and overthrow the
Philippine Government. Attended by Aggravating Circumstances of Aid or Armed Men, Craft, and
Fraud. The trial court is of opinion that 1.) The Congress usurped the powers of the judge 2.)
Assumed judicial magistracy by pronouncing the guilt of the CPP without any forms of safeguard
of a judicial trial. 3.) It created a presumption of organizational guilt by being members of the
CPP regardless of voluntariness.

The Anti Subversive Act of 1957 was approved 20June1957. It is an act to outlaw the CPP and
similar associations penalizing membership therein, and for other purposes. It defined the
Communist Party being although a political party is in fact an organized conspiracy to overthrow
the Government, not only by force and violence but also by deceit, subversion and other illegal
means. It declares that the CPP is a clear and present danger to the security of the Philippines.
Section 4 provided that affiliation with full knowledge of the illegal acts of the CPP is punishable.
Section 5 states that due investigation by a designated prosecutor by the Secretary of Justice be
made prior to filing of information in court. Section 6 provides for penalty for furnishing false
evidence. Section 7 provides for 2 witnesses in open court for acts penalized by prision mayor to
death. Section 8 allows the renunciation of membership to the CCP through writing under oath.
Section 9 declares the constitutionality of the statute and its valid exercise under freedom if
thought, assembly and association.

Issue: Whether or not RA1700 is a bill of attainder/ ex post facto law.

Whether or Not RA1700 violates freedom of expression.

Held: The court holds the VALIDITY Of the Anti-Subversion Act of 1957.

A bill of attainder is solely a legislative act. It punishes without the benefit of the trial. It is the
substitution of judicial determination to a legislative determination of guilt. In order for a statute
be measured as a bill of attainder, the following requisites must be present: 1.) The statute
specifies persons, groups. 2.) the statute is applied retroactively and reach past conduct. (A bill
of attainder relatively is also an ex post facto law.)

In the case at bar, the statute simply declares the CPP as an organized conspiracy for the
overthrow of the Government for purposes of example of SECTION 4 of the Act. The Act applies
not only to the CPP but also to other organizations having the same purpose and their
successors. The Act’s focus is on the conduct not person.
Membership to this organizations, to be UNLAWFUL, it must be shown that membership was
acquired with the intent to further the goals of the organization by overt acts. This is the
element of MEMBERSHIP with KNOWLEDGE that is punishable. This is the required proof of a
member’s direct participation. Why is membership punished. Membership renders aid and
encouragement to the organization. Membership makes himself party to its unlawful acts.

Furthermore, the statute is PROSPECTIVE in nature. Section 4 prohibits acts committed after
approval of the act. The members of the subversive organizations before the passing of this Act
is given an opportunity to escape liability by renouncing membership in accordance with Section
8. The statute applies the principle of mutatis mutandis or that the necessary changes having
been made.

The declaration of that the CPP is an organized conspiracy to overthrow the Philippine
Government should not be the basis of guilt. This declaration is only a basis of Section 4 of the
Act. The EXISTENCE OF SUBSTANTIVE EVIL justifies the limitation to the exercise of “Freedom of
Expression and Association” in this matter. Before the enactment of the statute and statements
in the preamble, careful investigations by the Congress were done. The court further stresses
that whatever interest in freedom of speech and association is excluded in the prohibition of
membership in the CPP are weak considering NATIONAL SECURITY and PRESERVATION of
DEMOCRACY.

The court set basic guidelines to be observed in the prosecution under RA1700. In addition to
proving circumstances/ evidences of subversion, the following elements must also be
established:

1. Subversive Organizations besides the CPP, it must be proven that the organization
purpose is to overthrow the present Government of the Philippines and establish a
domination of a FOREIGN POWER. Membership is willfully and knowingly done by overt
acts.
2. In case of CPP, the continued pursuance of its subversive purpose. Membership is
willfully and knowingly done by overt acts.

The court did not make any judgment on the crimes of the accused under the Act. The Supreme
Court set aside the resolution of the TRIAL COURT.

27. LACSON VS. EXECUTIVE SECRETARY


[301 SCRA 298; G.R. NO. 128096; 20 JAN 1999]

Facts: Eleven persons believed to be members of the Kuratong Baleleng gang, an organized
crime syndicate involved in bank robberies, were slain by elements of the Anti-Bank Robbery and
Intelligence Task Group (ABRITG). Among those included in the ABRITG were petitioners and
petitioner-intervenors.

Acting on a media expose of SPO2 Eduardo delos Reyes, a member of the Criminal Investigation
Command, that what actually transpired was a summary execution and not a shoot-out between
the Kuratong Baleleng gang members and the ABRITG, Ombudsman Aniano Desierto formed a
panel of investigators to investigate the said incident. Said panel found the incident as a
legitimate police operation. However, a review board modified the panel’s finding and
recommended the indictment for multiple murder against twenty-six respondents including
herein petitioner, charged as principal, and herein petitioner-intervenors, charged as
accessories. After a reinvestigation, the Ombudsman filed amended informations before the
Sandiganbayan, where petitioner was charged only as an accessory.

The accused filed separate motions questioning the jurisdiction of the Sandiganbayan, asserting
that under the amended informations, the cases fall within the jurisdiction of the Regional Trial
Court pursuant to Section 2 of R.A. 7975. They contend that the said law limited the jurisdiction
of the Sandiganbayan to cases where one or ore of the “principal accused” are government
officals with Salary Grade 27 or higher, or PNP officials with rank of Chief Superintendent or
higher. Thus, they did not qualify under said requisites. However, pending resolution of their
motions, R.A. 8249 was approved amending the jurisdiction of the Sandiganbayan by deleting
the word “principal” from the phrase “principal accused” in Section 2 of R.A. 7975.

Petitioner questions the constitutionality of Section 4 of R.A. 8249, including Section 7 which
provides that the said law shall apply to all cases pending in any court over which trial has not
begun as of the approval hereof.

Issue: Whether or not said statute may be considered as an ex-post facto statute.

Whether or not the multiple murder of the alleged members of the Kuratong Baleleng was
committed in relation to the office of the accused PNP officers which is essential to the
determination whether the case falls within the Sandiganbayan’s or Regional Trial Court’s
jurisdiction.

Held: There is nothing ex post facto in R.A. 8249. Ex post facto law, generally, provides
retroactive effect of penal laws. R.A. 8249 is not a penal law. It is a substantive law on
jurisdiction which is not penal in character. Penal laws are those acts of the Legislature which
prohibit certain acts and establish penalties for their violations or those that define crimes and
provide for their punishment. R.A. 7975, as regards the Sandiganbayan’s jurisdiction, its mode of
appeal and other procedural matters, has been declared by the Court as not a penal law, but
clearly a procedural statute, one which prescribes rules of procedure by which courts applying
laws of all kinds can properly administer justice. Not being a penal law, the retroactive
application of R.A. 8249 cannot be challenged as unconstitutional.

In People vs. Montejo, it was held that an offense is said to have been committed in relation to
the office if it is intimately connected with the office of the offender and perpetrated while he
was in the performance of his official functions. Such intimate relation must be alleged in the
information which is essential in determining the jurisdiction of the Sandiganbayan. However,
upon examination of the amended information, there was no specific allegation of facts that the
shooting of the victim by the said principal accused was intimately related to the discharge of
their official duties as police officers. Likewise, the amended information does not indicate that
the said accused arrested and investigated the victim and then killed the latter while in their
custody. The stringent requirement that the charge set forth with such particularity as will
reasonably indicate the exact offense which the accused is alleged to have committed in relation
to his office was not established.

Consequently, for failure to show in the amended informations that the charge of murder was
intimately connected with the discharge of official functions of the accused PNP officers, the
offense charged in the subject criminal cases is plain murder and, therefore, within the exclusive
original jurisdiction of the Regional Trial Court and not the Sandiganbayan.

28. PEOPLE VS. RELOVA


[149 SCRA 292; G.R. NO.L-45129; 6 MAR 1987]
FACTS: In this petition for certiorari and mandamus, People of the Philippines seeks to set aside
the orders of Respondent Judge Hon. Relova quashing an information for theft filed against Mr.
Opulencia on the ground of double jeopardy and denying the petitioner’s motion for
reconsideration.. On Feb.1 1975, Batangas police together with personnel of Batangas Electric
Light System, equipped with a search warrant issued by a city judge of Batangas to search and
examine the premises of the Opulencia Carpena Ice Plant owned by one Manuel Opulencia. They
discovered electric wiring devices have been installed without authority from the city
government and architecturally concealed inside the walls of the building. Said devices are
designed purposely to lower or decrease the readings of electric current consumption in the
plant’s electric meter. The case was dismissed on the ground of prescription for the complaint
was filed nine months prior to discovery when it should be 2months prior to discovery that the
act being a light felony and prescribed the right to file in court. On Nov 24, 1975, another case
was filed against Mr. Opulencia by the Assistant City Fiscal of Batangas for a violation of a
Batangas Ordinance regarding unauthorized electrical installations with resulting damage and
prejudice to City of Batangas in the amount of P41,062.16. Before arraignment, Opulencia filed a
motion to quash on the ground of double jeopardy. The Assistant fiscal’s claim is that it is not
double jeopardy because the first offense charged against the accused was unauthorized
installation of electrical devices without the approval and necessary authority from the City
Government which was punishable by an ordinance, where in the case was dismissed, as
opposed to the second offense which is theft of electricity which is punishable by the Revised
Penal Code making it a different crime charged against the 1st complaint against Mr.Opulencia.

Issue: Whether or Not the accused Mr. Opulencia can invoke double jeopardy as defense to the
second offense charged against him by the assistant fiscal of Batangas on the ground of theft of
electricity punishable by a statute against the Revised Penal Code.

Held: Yes, Mr. Opulencia can invoke double jeopardy as defense for the second offense because
as tediously explained in the case of Yap vs Lutero, the bill of rights give two instances or kinds
of double jeopardy. The first would be that “No person shall be twice put in jeopardy of
punishment for the same offense and the second sentence states that “If an act is punishable by
a law or an ordinance, the conviction or acquittal shall bar to another prosecution for the same
act”. In the case at bar, it was very evident that the charges filed against Mr. Opulencia will fall
on the 2nd kind or definition of double jeopardy wherein it contemplates double jeopardy of
punishment for the same act. It further explains that even if the offenses charged are not the
same, owing that the first charge constitutes a violation of an ordinance and the second charge
was a violation against the revised penal code, the fact that the two charges sprung from one
and the same act of conviction or acquittal under either the law or the ordinance shall bar a
prosecution under the other thus making it against the logic of double jeopardy. The fact that
Mr. Opulencia was acquitted on the first offense should bar the 2 nd complaint against him coming
from the same identity as that of the 1st offense charged against Mr.Opulencia.

29. LANSANG VS. GARCIA


[42 SCRA 448; L-33964; 11 Dec 1971]

Facts: In the evening of August 21, 1971, at about 9 p.m., while the Liberal Party of the
Philippines was holding a public meeting at Plaza Miranda, Manila, for the presentation of its
candidates in the general elections scheduled for November 8, 1971, two hand grenades were
thrown at the platform where said candidates and other persons were. Eight persons were killed
and many more injured. Proclamation 889 was issued by the President suspending privilege of
writ of habeas corpus stating that there is a conspiracy of rebellion and insurrection in order to
forcibly seize political power. Petitions for writ of habeas corpus were filed by persons (13) who
have been arrested without a warrant.

It was stated that one of the safeguards of the proclamation was that it is to be applied to
persons caught in flagrante delicto. Incidentally, Proc. 889-A was issued as an amendment,
inserting the word “actually staging”. Proc. 889-B was also issued lifting the suspension of
privilege in 27 provinces, 3 sub-provinces and 26 cities. Proc. 889-C was issued restoring the
suspension in 13 provinces and cities(mostly in Mindanao). Proc. 889-D further lifted the
suspension in 7 provinces and 4 cities. Only 18 provinces and sub-provinces and 2 cities whose
privilege was suspended. Petitioners maintained that Proclamation No. 889 did not declare the
existence of actual "invasion insurrection or rebellion or imminent danger thereof, however it
became moot and academic since it was amended. Petitioners further contend that public safety
did not require the issuance of proclamations stating: (a) that there is no rebellion; (b) that, prior
to and at the time of the suspension of the privilege, the Government was functioning normally,
as were the courts; (c) that no untoward incident, confirmatory of an alleged July-August Plan,
has actually taken place after August 21, 1971; (d) that the President's alleged apprehension,
because of said plan, is non-existent and unjustified; and (e) that the Communist forces in the
Philippines are too small and weak to jeopardize public safety to such extent as to require the
suspension of the privilege of the writ of habeas corpus.

A resolution was issued by majority of the Court having tentatively arrived at a consensus that it
may inquire in order to satisfy itself of the existence of the factual bases for the proclamations.
Now the Court resolves after conclusive decision reached by majority.

Issue: Whether or Not the authority to decide whether the exigency has arisen requiring
suspension (of the privilege of the writ of habeas corpus) belongs to the President and his
decision is final and conclusive upon the courts and upon all other persons.

Whether or Not public safety require the suspension of the privilege of the writ of habeas corpus
decreed in Proclamation No. 889-A.

Held: The President has authority however it is subject to judicial review. Two conditions must
concur for the valid exercise of the authority to suspend the privilege to the writ (a) there must
be "invasion, insurrection, or rebellion" or "imminent danger thereof," and (b) "public safety"
must require the suspension of the privilege. President has three (3) courses of action: (a) to call
out the armed forces; (b) to suspend the privilege of the writ of habeas corpus; and (c) to place
the Philippines or any part thereof under martial law. He had, already, called out the armed
forces, proved inadequate. Of the two other alternatives, the suspension of the privilege is the
least harsh.

Petitioners contention that CPP-NPA has no ability, is negatived by the killing of 5 mayors, 20
barrio captains and 3 chiefs of police; that there were fourteen (14) meaningful bombing
incidents in the Greater Manila Area in 1970. CPP has managed to infiltrate or establish and
control nine major labor organizations; has exploited the (11) major student or youth
organizations; about thirty (30) mass organizations actively advancing the CPP.

30. US VS. BUSTOS


[37 PHIL. 731; G.R. L-12592; 8 MAR 1918]

Facts: In the latter part of 1915, numerous citizens of the Province of Pampanga assembled, and
prepared and signed a petition to the Executive Secretary(privileged communication) through
the law office of Crossfield and O'Brien, and five individuals signed affidavits, charging Roman
Punsalan, justice of the peace of Macabebe and Masantol, Pampanga, with malfeasance in office
and asking for his removal. The specific charges against the justice of the peace include the
solicitation of money from persons who have pending cases before the judge. Now, Punsalan
alleged that accused published a writing which was false, scandalous, malicious, defamatory,
and libelous against him.

Issue: Whether or Not accused is entitled to constitutional protection by virtue of his right to
free speech and free press.
Held: Yes. The guaranties of a free speech and a free press include the right to criticize judicial
conduct. The administration of the law is a matter of vital public concern. Whether the law is
wisely or badly enforced is, therefore, a fit subject for proper comment. If the people cannot
criticize a justice of the peace or a judge the same as any other public officer, public opinion will
be effectively suppressed. It is a duty which every one owes to society or to the State to assist in
the investigation of any alleged misconduct. It is further the duty of all who know of any official
dereliction on the part of a magistrate or the wrongful act of any public officer to bring the facts
to the notice of those whose duty it is to inquire into and punish them.

The right to assemble and petition is the necessary consequence of republican institutions and
the complement of the part of free speech. Assembly means a right on the part of citizens to
meet peaceably for consultation in respect to public affairs. Petition means that any person or
group of persons can apply, without fear of penalty, to the appropriate branch or office of the
government for a redress of grievances. The persons assembling and petitioning must, of course,
assume responsibility for the charges made. All persons have an interest in the pure and efficient
administration of justice and of public affairs.

Public policy, the welfare of society, and the orderly administration of government have
demanded protection for public opinion. The inevitable and incontestable result has been the
development and adoption of the doctrine of privilege. All persons have an interest in the pure
and efficient administration of justice and of public affairs. The duty under which a party is
privileged is sufficient if it is social or moral in its nature and this person in good faith believes he
is acting in pursuance thereof although in fact he is mistaken. Although the charges are probably
not true as to the justice of the peace, they were believed to be true by the petitioners. Good
faith surrounded their action. Probable cause for them to think that malfeasance or misfeasance
in office existed is apparent. The ends and the motives of these citizens— to secure the removal
from office of a person thought to be venal — were justifiable. In no way did they abuse the
privilege.

In the usual case malice can be presumed from defamatory words. Privilege destroys that
presumption. A privileged communication should not be subjected to microscopic examination to
discover grounds of malice or falsity.

31. BURGOS, SR. V. CHIEF OF STAFF, AFP


[133 SCRA 800; G.R. NO. 64261; 26 DEC 1984]

Facts: Petitioners assail the validity of 2 search warrants issued on December 7, 1982 by
respondent Judge Cruz-Pano of the then Court of First Instance of Rizal, under which the
premises known as No. 19, Road 3, Project 6, Quezon City, and 784 Units C & D, RMS Building,
Quezon Avenue, Quezon City, business addresses of the "Metropolitan Mail" and "We Forum"
newspapers, respectively, were searched, and office and printing machines, equipment,
paraphernalia, motor vehicles and other articles used in the printing, publication and distribution
of the said newspapers, as well as numerous papers, documents, books and other written
literature alleged to be in the possession and control of petitioner Jose Burgos, Jr. publisher-editor
of the "We Forum" newspaper, were seized. As a consequence of the search and seizure, these
premises were padlocked and sealed, with the further result that the printing and publication of
said newspapers were discontinued. Respondents contend that petitioners should have filed a
motion to quash said warrants in the court that issued them before impugning the validity of the
same before this Court. Respondents also assail the petition on ground of laches (Failure or
negligence for an unreasonable and unexplained length of time to do that which, by exercising
due diligence, could or should have been done earlier. It is negligence or omission to assert a
right within a reasonable time, warranting a presumption that the party entitled to assert it
either has abandoned it or declined to assert it). Respondents further state that since petitioner
had already used as evidence some of the documents seized in a prior criminal case, he is
stopped from challenging the validity of the search warrants.
Petitioners submit the following reasons to nullify the questioned warrants:
1. Respondent Judge failed to conduct an examination under oath or affirmation of the
applicant and his witnesses, as mandated by the above-quoted constitutional provision as
well as Sec. 4, Rule 126 of the Rules of Court.
2. The search warrants pinpointed only one address which would be the former
abovementioned address.
3. Articles belonging to his co-petitioners were also seized although the warrants were only
directed against Jose Burgos, Jr.
4. Real properties were seized.
5. The application along with a joint affidavit, upon which the warrants were issued, from the
Metrocom Intelligence and Security Group could not have provided sufficient basis for the
finding of a probable cause upon which a warrant may be validly issued in accordance with
Section 3, Article IV of the 1973 Constitution.

Respondents justify the continued sealing of the printing machines on the ground that they have
been sequestered under Section 8 of Presidential Decree No. 885, as amended, which authorizes
sequestration of the property of any person engaged in subversive activities against the
government in accordance with implementing rules and regulations as may be issued by the
Secretary of National Defense.

Issue: Whether or Not the 2 search warrants were validly issued and executed.

Held: In regard to the quashal of warrants that petitioners should have initially filed to the lower
court, this Court takes cognizance of this petition in view of the seriousness and urgency of the
constitutional Issue raised, not to mention the public interest generated by the search of the "We
Forum" offices which was televised in Channel 7 and widely publicized in all metropolitan dailies.
The existence of this special circumstance justifies this Court to exercise its inherent power to
suspend its rules. With the contention pertaining to laches, the petitioners gave an explanation
evidencing that they have exhausted other extra-judicial efforts to remedy the situation,
negating the presumption that they have abandoned their right to the possession of the seized
property.

On the enumerated reasons:


1. This objection may properly be considered moot and academic, as petitioners themselves
conceded during the hearing on August 9, 1983, that an examination had indeed been
conducted by respondent judge of Col. Abadilla and his witnesses.
2. The defect pointed out is obviously a typographical error. Precisely, two search warrants
were applied for and issued because the purpose and intent were to search two distinct
premises. It would be quite absurd and illogical for respondent judge to have issued two
warrants intended for one and the same place.
3. Section 2, Rule 126, of the Rules of Court, does not require that the property to be seized
should be owned by the person against whom the search warrant is directed. It may or
may not be owned by him.
4. Petitioners do not claim to be the owners of the land and/or building on which the
machineries were placed. This being the case, the machineries in question, while in fact
bolted to the ground, remain movable property susceptible to seizure under a search
warrant.
5. The broad statements in the application and joint affidavit are mere conclusions of law and
does not satisfy the requirements of probable cause. Deficient of such particulars as would
justify a finding of the existence of probable cause, said allegation cannot serve as basis
for the issuance of a search warrant and it was a grave error for respondent judge to have
done so. In Alvarez v. Court of First Instance, this Court ruled that "the oath required must
refer to the truth of the facts within the personal knowledge of the petitioner or his
witnesses, because the purpose thereof is to convince the committing magistrate, not the
individual making the affidavit and seeking the issuance of the warrant, of the existence of
probable cause." Another factor which makes the search warrants under consideration
constitutionally objectionable is that they are in the nature of general warrants. The
description of the articles sought to be seized under the search warrants in question are
too general.

With regard to the respondents invoking PD 885, there is an absence of any implementing rules
and regulations promulgated by the Minister of National Defense. Furthermore, President Marcos
himself denies the request of military authorities to sequester the property seized from
petitioners. The closure of the premises subjected to search and seizure is contrary to the
freedom of the press as guaranteed in our fundamental law. The search warrants are declared
null and void.

32. ZALDIVAR VS. SANDIGANBAYAN


[170 SCRA 1; G.R. NO. 79690-707; 1 FEB 1989]

Facts: The case stemmed from the resolution of the Supreme Court stopping the respondent
from investigating graft cases involving Antique Gov. Enrique Zaldivar. The Court ruled that since
the adoption of the 1987 Constitution, respondent’s powers as Tanodbayan have been
superseded by the creation of the Office of the Ombudsman, he however becomes the Special
Prosecutor of the State, and can only conduct an investigation and file cases only when so
authorized by the Ombudsman. A motion for reconsideration was filed by the respondent
wherein he included statements which were unrelated in the Issue raised in the Court. This
include: (a)That he had been approached twice by a leading member of the court and he was
asked to 'go slow on Zaldivar and 'not to be too hard on him; (b) That he "was approached and
asked to refrain from investigating the COA report on illegal disbursements in the Supreme Court
because 'it will embarass the Court;" and (c) that in several instances, the undersigned
respondent was called over the phone by a leading member of the Court and was asked to
dismiss the cases against two Members of the Court." Statements of the respondent saying that
the SC’s order '"heightens the people's apprehension over the justice system in this country,
especially because the people have been thinking that only the small fly can get it while big
fishes go scot-free” was publicized in leading newspapers.

Now, the Court Resolved to require respondent to explain in writing why he should not be
punished for contempt of court for making such public statements reported in the media.
Respondent then sought to get some members of the Court to inhibit themselves in the
resolution of the Zaldivar case for alleged bias and prejudice against him. A little later, he in
effect asked the whole Court to inhibit itself from passing upon the Issue involved in proceeding
and to pass on responsibility for this matter to the Integrated Bar of the Philippines, upon the
ground that respondent cannot expect due process from this Court, that the Court has become
incapable of judging him impartially and fairly. The Court found respondent guilty of contempt of
court and indefinitely suspended from the practice of law. Now, he assails said conviction,
invoking his freedom of speech. Counsel for respondent urges that it is error "for this Court to
apply the "visible tendency" rule rather than the "clear and present danger" rule in disciplinary
and contempt charges."

Issue: Whether or Not there was a violation of the freedom of speech/expression.

Held: There was no violation. The Court did not purport to announce a new doctrine of "visible
tendency," it was simply paraphrasing Section 3 (d) of Rule 71 of the Revised Rules of Court
which penalizes a variety of contumacious conduct including: "any improper conduct tending,
directly or indirectly, to impede, obstruct or degrade the administration of justice."

Under either the "clear and present danger" test or the "balancing-of-interest test," the Court
held that the statements made by respondent Gonzalez are of such a nature and were made in
such a manner and under such circumstances, as to transcend the permissible limits of free
speech. What is here at stake is the authority of the Supreme Court to confront and prevent a
"substantive evil" consisting not only of the obstruction of a free and fair hearing of a particular
case but also the avoidance of the broader evil of the degradation of the judicial system of a
country and the destruction of the standards of professional conduct required from members of
the bar and officers of the courts, which has some implications to the society.

33. AYER PRODUCTIONS VS. CAPULONG


[160 SCRA 861; G.R. NO. L-82380; 29 APR 1988]

Facts: Petitioner McElroy an Australian film maker, and his movie production company, Ayer
Productions, envisioned, sometime in 1987, for commercial viewing and for Philippine and
international release, the historic peaceful struggle of the Filipinos at EDSA. The proposed motion
picture entitled "The Four Day Revolution" was endorsed by the MTRCB as and other government
agencies consulted. Ramos also signified his approval of the intended film production.

It is designed to be viewed in a six-hour mini-series television play, presented in a "docu-drama"


style, creating four fictional characters interwoven with real events, and utilizing actual
documentary footage as background. David Williamson is Australia's leading playwright and
Professor McCoy (University of New South Wales) is an American historian have developed a
script.

Enrile declared that he will not approve the use, appropriation, reproduction and/or exhibition of
his name, or picture, or that of any member of his family in any cinema or television production,
film or other medium for advertising or commercial exploitation. petitioners acceded to this
demand and the name of Enrile was deleted from the movie script, and petitioners proceeded to
film the projected motion picture. However, a complaint was filed by Enrile invoking his right to
privacy. RTC ordered for the desistance of the movie production and making of any reference to
plaintiff or his family and from creating any fictitious character in lieu of plaintiff which
nevertheless is based on, or bears substantial or marked resemblance to Enrile. Hence the
appeal.

Issue: Whether or Not freedom of expression was violated.

Held: Yes. Freedom of speech and of expression includes the freedom to film and produce
motion pictures and exhibit such motion pictures in theaters or to diffuse them through
television. Furthermore the circumstance that the production of motion picture films is a
commercial activity expected to yield monetary profit, is not a disqualification for availing of
freedom of speech and of expression.

The projected motion picture was as yet uncompleted and hence not exhibited to any audience.
Neither private respondent nor the respondent trial Judge knew what the completed film would
precisely look like. There was, in other words, no "clear and present danger" of any violation of
any right to privacy. Subject matter is one of public interest and concern. The subject thus
relates to a highly critical stage in the history of the country.

At all relevant times, during which the momentous events, clearly of public concern, that
petitioners propose to film were taking place, Enrile was a "public figure:" Such public figures
were held to have lost, to some extent at least, their right to privacy.

The line of equilibrium in the specific context of the instant case between the constitutional
freedom of speech and of expression and the right of privacy, may be marked out in terms of a
requirement that the proposed motion picture must be fairly truthful and historical in its
presentation of events.

34. Alvero vs. Dizon [GR L-342, 4 May 1946]


En Banc, de Joya (J): 4 concur, 4 acting justices concur
Facts: On 12 February 1945, while the battle for Manila was raging, soldiers of the United States
Army, accompanied by men of Filipino Guerrilla Forces, placed Aurelio S. Alvero under arrest,
having been suspected of collaboration with the enemy, and seized and took certain papers from
his house in Pasay, Rizal. On or about 4 October 1945, Alvero was accused of treason, in criminal
case 3 of the People’s Court; after which, on 1 December 1945, he filed a petition, demanding
the return of the papers allegedly seized and taken from his house. Alvero also filed a petition for
bail, at the hearing of which the prosecution presented certain papers and documents, which
were admitted as part of its evidence, and said petition was denied. At the trial of the case on
the merits, the prosecution again presented said papers and documents, which were admitted as
part of its evidence, and were marked as exhibits. On 26 February 1946, the judges issued an
order denying the petition for the return of the documents, and admitted as competent evidence
the documents presented by the prosecution. On the same date that said order was issued,
denying the petition for the return of said documents, Alvero asked for the reconsideration of
said order, which was also denied. Alvero filed a petition for certiorari with injunction with the
Supreme Court.

Issue: Whether the documents seized by United States Army personnel at Alvero’s home can be
used as evidence against the latter.

Held: The right of officers and men of the United States Army to arrest Alvero, as a
collaborationist suspect, and to seize his personal papers, without any search warrant, in the
zone of military operations, is unquestionable, under the provisions of article 4, Chapter II,
Section I, of the Regulations relative to the Laws and Customs of War on Land of the Hague
Conventions of 1907, authorizing the seizure of military papers in the possession of prisoners of
war; and also under the proclamation, dated 29 December 1944, issued by Gen. Douglas
MacArthur, as Commander in Chief of the United States Army, declaring his purpose to remove
certain citizens of the Philippines, who had voluntarily given aid and comfort to the enemy, in
violation of the allegiance due the Governments of the United States and the Commonwealth of
the Philippines, when apprehended, from any position of political and economic influence in the
Philippines and to hold them in restraint for the duration of the war. The purpose of the
constitutional provisions against unlawful searches and seizures is to prevent violations of
private security in person and property, and unlawful invasions of the sanctity of the home, by
officers of the law acting under legislative or judicial sanction, and to give remedy against such
usurpations when attempted. But it does not prohibit the Government from taking advantage of
unlawful searches made by a private person or under authority of state law. Herein, as the
soldiers of the United States Army, that took and seized certain papers and documents from the
residence of Alvero, were not acting as agents or on behalf of the Government of the
Commonwealth of the Philippines; and that those papers and documents came into the
possession of the authorities of the Commonwealth Government, through the Office of the CIC of
the United States Army in Manila, the use and presentation of said papers and documents, as
evidence for the prosecution against Alvero, at the trial of his case for treason, before the
People’s Court, cannot now be legally attacked, on the ground of unlawful or unreasonable
searches and seizures, or on any other constitutional ground, as declared by the Supreme Court
of the United States in similar cases. (See Burdeau vs. McDowell, 256 U. S., 465; Gambino vs.
United States, 275 U. S., 310.)

35. People vs. Andre Marti [GR 81561, 18 January 1991]


Third Division, Bidin (J): 3 concur

Facts: On 14 August 1987, Andre Marti and his common-law wife, Shirley Reyes, went to the
booth of the Manila Packing and Export Forwarders in the Pistang Pilipino Complex, Ermita,
Manila, carrying with them 4 gift-wrapped packages. Anita Reyes (the proprietress and no
relation to Shirley Reyes) attended to them. Marti informed Anita Reyes that he was sending the
packages to a friend in Zurich, Switzerland. Marti filled up the contract necessary for the
transaction, writing therein his name, passport number, the date of shipment and the name and
address of the consignee, namely, “WALTER FIERZ, Mattacketr II, 8052 Zurich, Switzerland.”
Anita Reyes did not inspect the packages as Marti refused, who assured the former that the
packages simply contained books, cigars, and gloves and were gifts to his friend in Zurich. In
view of Marti’s representation, the 4 packages were then placed inside a brown corrugated box,
with styro-foam placed at the bottom and on top of the packages, and sealed with masking tape.
Before delivery of Marti’s box to the Bureau of Customs and/or Bureau of Posts, Mr. Job Reyes
(proprietor) and husband of Anita (Reyes), following standard operating procedure, opened the
boxes for final inspection, where a peculiar odor emitted therefrom. Job pulled out a cellophane
wrapper protruding from the opening of one of the gloves, and took several grams of the
contents thereof. Job Reyes forthwith prepared a letter reporting the shipment to the NBI and
requesting a laboratory examination of the samples he extracted from the cellophane wrapper.
At the Narcotics Section of the National Bureau of Investigation (NBI), the box containing Marti’s
packages was opened, yielding dried marijuana leaves, or cake-like (bricks) dried marijuana
leaves. The NBI agents made an inventory and took charge of the box and of the contents
thereof, after signing a “Receipt” acknowledging custody of the said effects. Thereupon, the NBI
agents tried to locate Marti but to no avail, inasmuch as the latter’s stated address was the
Manila Central Post Office. Thereafter, an Information was filed against Marti for violation of RA
6425, otherwise known as the Dangerous Drugs Act. After trial, the Special Criminal Court of
Manila (Regional Trial Court, Branch XLIX) rendered the decision, convicting Marti of violation of
Section 21 (b), Article IV in relation to Section 4, Article 11 and Section 2 (e)(i), Article 1 of
Republic Act 6425, as amended, otherwise known as the Dangerous Drugs Act. Marti appealed.

Issue: Whether an act of a private individual, allegedly in violation of the accused’s


constitutional rights, be invoked against the State.

Held: In the absence of governmental interference, the liberties guaranteed by the Constitution
cannot be invoked against the State. The contraband herein, having come into possession of the
Government without the latter transgressing the accused’s rights against unreasonable search
and seizure, the Court sees no cogent reason why the same should not be admitted against him
in the prosecution of the offense charged. The mere presence of the NBI agents did not convert
the reasonable search effected by Reyes into a warrantless search and seizure proscribed by the
Constitution. Merely to observe and look at that which is in plain sight is not a search. Having
observed that which is open, where no trespass has been committed in aid thereof, is not search.
Where the contraband articles are identified without a trespass on the part of the arresting
officer, there is not the search that is prohibited by the constitution. The constitutional
proscription against unlawful searches and seizures therefore applies as a restraint directed only
against the government and its agencies tasked with the enforcement of the law. Thus, it could
only be invoked against the State to whom the restraint against arbitrary and unreasonable
exercise of power is imposed. If the search is made upon the request of law enforcers, a warrant
must generally be first secured if it is to pass the test of constitutionality. However, if the search
is made at the behest or initiative of the proprietor of a private establishment for its own and
private purposes, as in the case at bar, and without the intervention of police authorities, the
right against unreasonable search and seizure cannot be invoked for only the act of private
individual, not the law enforcers, is involved. In sum, the protection against unreasonable
searches and seizures cannot be extended to acts committed by private individuals so as to
bring it within the ambit of alleged unlawful intrusion by the government.

36. Bache & Co. (Phil.) Inc. vs. Ruiz [GR L-32409, 27 February 1971]
En Banc, Villamor (J): 7 concur, 1 filed a separate concurring opinion to which 1 concurs, 1
concurs in result

Facts: On 24 February 1970, Misael P. Vera, Commissioner of Internal Revenue, wrote a letter
addressed to Judge Vivencio M. Ruiz requesting the issuance of a search warrant against Bache
& Co. (Phil.), Inc. and Frederick E. Seggerman for violation of Section 46(a) of the National
Internal Revenue Code (NIRC), in relation to all other pertinent provisions thereof, particularly
Sections 53, 72, 73, 208 and 209, and authorizing Revenue Examiner Rodolfo de Leon to make
and file the application for search warrant which was attached to the letter. In the afternoon of
the following day, De Leon and his witness, Arturo Logronio, went to the Court of First Instance
(CFI) of Rizal. They brought with them the following papers: Vera’s letter-request; an application
for search warrant already filled up but still unsigned by De Leon; an affidavit of Logronio
subscribed before De Leon; a deposition in printed form of Logronio already accomplished and
signed by him but not yet subscribed; and a search warrant already accomplished but still
unsigned by Judge. At that time the Judge was hearing a certain case; so, by means of a note, he
instructed his Deputy Clerk of Court to take the depositions of De Leon and Logronio. After the
session had adjourned, the Judge was informed that the depositions had already been taken. The
stenographer, upon request of the Judge, read to him her stenographic notes; and thereafter, the
Judge asked Logronio to take the oath and warned him that if his deposition was found to be
false and without legal basis, he could be charged for perjury. The Judge signed de Leon’s
application for search warrant and Logronio’s deposition. Search Warrant 2-M-70 was then
signed by Judge and accordingly issued. 3 days later (a Saturday), the BIR agents served the
search warrant to the corporation and Seggerman at the offices of the corporation on Ayala
Avenue, Makati, Rizal. The corporation’s lawyers protested the search on the ground that no
formal complaint or transcript of testimony was attached to the warrant. The agents
nevertheless proceeded with their search which yielded 6 boxes of documents. On 3 March 1970,
the corporation and Seggerman filed a petition with the Court of First Instance (CFI) of Rizal
praying that the search warrant be quashed, dissolved or recalled, that preliminary prohibitory
and mandatory writs of injunction be issued, that the search warrant be declared null and void,
and that Vera, Logronio, de Leon, et. al., be ordered to pay the corporation and Seggerman,
jointly and severally, damages and attorney’s fees. After hearing and on 29 July 1970, the court
issued an order dismissing the petition for dissolution of the search warrant. In the meantime, or
on 16 April 1970, the Bureau of Internal Revenue made tax assessments on the corporation in
the total sum of P2,594,729.97, partly, if not entirely, based on the documents thus seized. The
corporation and Seggerman filed an action for certiorari, prohibition, and mandamus.

Issue: Whether the corporation has the right to contest the legality of the seizure of documents
from its office.

Held: The legality of a seizure can be contested only by the party whose rights have been
impaired thereby, and that the objection to an unlawful search and seizure is purely personal and
cannot be availed of by third parties. In Stonehill, et al. vs. Diokno, et al. (GR L-19550, 19 June
1967; 20 SCRA 383) the Supreme Court impliedly recognized the right of a corporation to object
against unreasonable searches and seizures; holding that the corporations have their respective
personalities, separate and distinct from the personality of the corporate officers, regardless of
the amount of shares of stock or the interest of each of them in said corporations, whatever, the
offices they hold therein may be; and that the corporate officers therefore may not validly object
to the use in evidence against them of the documents, papers and things seized from the offices
and premises of the corporations, since the right to object to the admission of said papers in
evidence belongs exclusively to the corporations, to whom the seized effects belong, and may
not be invoked by the corporate officers in proceedings against them in their individual capacity.
The distinction between the Stonehill case and the present case is that: in the former case, only
the officers of the various corporations in whose offices documents, papers and effects were
searched and seized were the petitioners; while in the latter, the corporation to whom the seized
documents belong, and whose rights have thereby been impaired, is itself a petitioner. On that
score, the corporation herein stands on a different footing from the corporations in Stonehill.
Moreover, herein, the search warrant was void inasmuch as First, there was no personal
examination conducted by the Judge of the complainant (De Leon) and his witness (Logronio).
The Judge did not ask either of the two any question the answer to which could possibly be the
basis for determining whether or not there was probable cause against Bache & Co. and
Seggerman. The participation of the Judge in the proceedings which led to the issuance of Search
Warrant 2-M-70 was thus limited to listening to the stenographer’s readings of her notes, to a
few words of warning against the commission of perjury, and to administering the oath to the
complainant and his witness. This cannot be consider a personal examination. Second, the
search warrant was issued for more than one specific offense. The search warrant was issued for
at least 4 distinct offenses under the Tax Code. The first is the violation of Section 46(a), Section
72 and Section 73 (the filing of income tax returns), which are interrelated. The second is the
violation of Section 53 (withholding of income taxes at source). The third is the violation of
Section 208 (unlawful pursuit of business or occupation); and the fourth is the violation of
Section 209 (failure to make a return of receipts, sales, business or gross value of output actually
removed or to pay the tax due thereon). Even in their classification the 6 provisions are
embraced in 2 different titles: Sections 46(a), 53, 72 and 73 are under Title II (Income Tax); while
Sections 208 and 209 are under Title V (Privilege Tax on Business and Occupation). Lastly, the
search warrant does not particularly describe the things to be seized. Search Warrant No. 2-M-70
tends to defeat the major objective of the Bill of Rights, i.e., the elimination of general warrants,
for the language used therein is so all-embracing as to include all conceivable records of the
corporation, which, if seized, could possibly render its business inoperative. Thus, Search
Warrant 2-M-70 is null and void.

37. Burgos v. Chief of Staff, AFP [GR 64261, 26 December 1984]


En Banc, Escolin (J): 10 concur, 1 took no part

Facts: On 7 December 1982, Judge Ernani Cruz-Paño, Executive Judge of the then CFI Rizal
[Quezon City], issued 2 search warrants where the premises at 19, Road 3, Project 6, Quezon
City, and 784 Units C & D, RMS Building, Quezon Avenue, Quezon City, business addresses of the
“Metropolitan Mail” and “We Forum” newspapers, respectively, were searched, and office and
printing machines, equipment, paraphernalia, motor vehicles and other articles used in the
printing, publication and distribution of the said newspapers, as well as numerous papers,
documents, books and other written literature alleged to be in the possession and control of Jose
Burgos, Jr. publisher-editor of the “We Forum” newspaper, were seized. A petition for certiorari,
prohibition and mandamus with preliminary mandatory and prohibitory injunction was filed after
6 months following the raid to question the validity of said search warrants, and to enjoin the
Judge Advocate General of the AFP, the city fiscal of Quezon City, et.al. from using the articles
seized as evidence in Criminal Case Q-022782 of the RTC Quezon City (People v. Burgos).

Issue: Whether allegations of possession and printing of subversive materials may be the basis
of the issuance of search warrants.

Held: Section 3 provides that no search warrant or warrant of arrest shall issue except upon
probable cause to be determined by the judge, or such other responsible officer as may be
authorized by law, after examination under oath or affirmation of the complainant and the
witnesses he may produce, and particularly describing the place to be searched and the persons
or things to be seized. Probable cause for a search is defined as such facts and circumstances
which would lead a reasonably discreet and prudent man to believe that an offense has been
committed and that the objects sought in connection with the offense are in the place sought to
be searched. In mandating that “no warrant shall issue except upon probable cause to be
determined by the judge, after examination under oath or affirmation of the complainant and the
witnesses he may produce”; the Constitution requires no less than personal knowledge by the
complainant or his witnesses of the facts upon which the issuance of a search warrant may be
justified. Herein, a statement in the effect that Burgos “is in possession or has in his control
printing equipment and other paraphernalia, news publications and other documents which were
used and are all continuously being used as a means of committing the offense of subversion
punishable under PD 885, as amended” is a mere conclusion of law and does not satisfy the
requirements of probable cause. Bereft of such particulars as would justify a finding of the
existence of probable cause, said allegation cannot serve as basis for the issuance of a search
warrant. Further, when the search warrant applied for is directed against a newspaper publisher
or editor in connection with the publication of subversive materials, the application and/or its
supporting affidavits must contain a specification, stating with particularity the alleged
subversive material he has published or is intending to publish. Mere generalization will not
suffice.

38. People vs. Molina [GR 133917, 19 February 2001]


En Banc, Ynares-Santiago (J): 14 concur

Facts: Sometime in June 1996, SPO1 Marino Paguidopon, then a member of the Philippine
National Police (PNP) detailed at Precinct No. 3, Matina, Davao City, received an information
regarding the presence of an alleged marijuana pusher in Davao City. The first time he came to
see the said marijuana pusher in person was during the first week of July 1996. SPO1 Paguidopon
was then with his informer when a motorcycle passed by. His informer pointed to the motorcycle
driver, Gregorio Mula y Malagura (@”Boboy”), as the pusher. As to Nasario Molina y Manamat (@
“Bobong”), SPO1 Paguidopon had no occasion to see him prior to 8 August 1996. At about 7:30
a.m. of 8 August 1996, SPO1 Paguidopon received an information that the alleged pusher will be
passing at NHA, Maa, Davao City any time that morning. Consequently, at around 8:00 a.m. he
called for assistance at the PNP, Precinct 3, Matina, Davao City, which immediately dispatched
the team of SPO4 Dionisio Cloribel (team leader), SPO2 Paguidopon (brother of SPO1 Marino
Paguidopon), and SPO1 Pamplona, to proceed to the house of SPO1 Marino Paguidopon where
they would wait for the alleged pusher to pass by. At around 9:30 a.m., while the team were
positioned in the house of SPO1 Paguidopon, a “trisikad” carrying Mula and Molina passed by. At
that instance, SPO1 Paguidopon pointed to Mula and Molina as the pushers. Thereupon, the team
boarded their vehicle and overtook the “trisikad.” SPO1 Paguidopon was left in his house, 30
meters from where Mula and Molina were accosted. The police officers then ordered the
“trisikad” to stop. At that point, Mula, who was holding a black bag, handed the same to Molina.
Subsequently, SPO1 Pamplona introduced himself as a police officer and asked Molina to open
the bag. Molina replied, “Boss, if possible we will settle this.” SPO1 Pamplona insisted on opening
the bag, which revealed dried marijuana leaves inside. Thereafter, Mula and Molina were
handcuffed by the police officers. On 6 December 1996, the accused Mula and Molina, through
counsel, jointly filed a Demurrer to Evidence, contending that the marijuana allegedly seized
from them is inadmissible as evidence for having been obtained in violation of their
constitutional right against unreasonable searches and seizures. The demurrer was denied by
the trial court. A motion for reconsideration was filed by the accused, but this was likewise
denied. The accused waived presentation of evidence and opted to file a joint memorandum. On
25 April 1997, the trial court rendered the decision, finding the accused guilty of the offense
charged, and sentenced both to suffer the penalty of death by lethal injection. Pursuant to Article
47 of the Revised Penal Code and Rule 122, Section 10 of the Rules of Court, the case was
elevated to the Supreme Court on automatic review.

Issue: Whether Mula and Molina manifested outward indication that would justify their arrest,
and the seizure of prohibited drugs that were in their possession.

Held: The fundamental law of the land mandates that searches and seizures be carried out in a
reasonable fashion, that is, by virtue or on the strength of a search warrant predicated upon the
existence of a probable cause. Complementary to the foregoing provision is the exclusionary rule
enshrined under Article III, Section 3, paragraph 2, which bolsters and solidifies the protection
against unreasonable searches and seizures. The foregoing constitutional proscription, however,
is not without exceptions. Search and seizure may be made without a warrant and the evidence
obtained therefrom may be admissible in the following instances: (1) search incident to a lawful
arrest; (2) search of a moving motor vehicle; (3) search in violation of customs laws; (4) seizure
of evidence in plain view; (5) when the accused himself waives his right against unreasonable
searches and seizures; and (6) stop and frisk situations (Terry search). The first exception
(search incidental to a lawful arrest) includes a valid warrantless search and seizure pursuant to
an equally valid warrantless arrest which must precede the search. Still, the law requires that
there be first a lawful arrest before a search can be made — the process cannot be reversed.
Herein, Mula and Molina manifested no outward indication that would justify their arrest. In
holding a bag on board a trisikad, they could not be said to be committing, attempting to commit
or have committed a crime. It matters not that Molina responded “Boss, if possible we will settle
this” to the request of SPO1 Pamplona to open the bag. Such response which allegedly
reinforced the “suspicion” of the arresting officers that Mula and Molina were committing a
crime, is an equivocal statement which standing alone will not constitute probable cause to
effect an in flagrante delicto arrest. Note that were it not for SPO1 Marino Paguidopon, Mula and
Molina could not be the subject of any suspicion, reasonable or otherwise. Further, it would
appear that the names and addresses of Mula and Molina came to the knowledge of SPO1
Paguidopon only after they were arrested, and such cannot lend a semblance of validity on the
arrest effected by the peace officers. Withal, the Court holds that the arrest of Mula and Molina
does not fall under the exceptions allowed by the rules. Hence, the search conducted on their
person was likewise illegal. Consequently, the marijuana seized by the peace officers could not
be admitted as evidence against them.

39. Yee Sue Koy vs. Almeda [GR 47021, 15 June 1940]
Laurel (J): 3 concur, 1 concurs in result

Facts: In response to a sworn application of Mariano G. Almeda, chief agent of the Anti-Usury
Board, dated 5 May 1938, the justice of the peace of Sagay, Occidental Negros, after taking the
testimony of applicant’s witness, Jose Estrada, special agent of the Anti-Usury Board, issued on
the same date a search warrant commanding any peace officer to search during day time the
store and premises occupied by Sam Sing & Co., situated at Sagay, Occidental Negros, as well as
the person of said Sam Sing & Co., and to seize the documents, notebooks, lists, receipts and
promissory notes being used by said Sam Sing & Co. in connection with their activities of lending
money at usurious rates of interest in violation of law, or such as may be found, and to bring
them forthwith before the aforesaid justice of the peace of Sagay. On the same date, at 10:30 a.
m., search was accordingly made by Mariano G. Almeda, Jose Estrada, 2 internal revenue agents
and 2 members of the Philippine Army, who seized certain receipt books, vales or promissory
notes, chits, notebooks, journal book, and collection list belonging to Sam Sing & Co. and
enumerated in the inventory receipt issued by Mariano G. Almeda to the owner of the
documents, papers and articles seized. Immediately after the search and seizure thus effected,
Mariano G. Almeda filed a return with the justice of the peace of Sagay together. With a request
that the office of the Anti-Usury Board be allowed to retain possession of the articles seized for
examination, pursuant to section 4 of Act 4109, which request was granted. Under the date of 11
March 1939, Godofredo P. Escalona, counsel for Sam Sing & Co. filed a motion with the Court of
First Instance (CFI) of Occidental Negros praying that the search warrant and the seizure effected
thereunder be declared illegal and set aside and that the articles in question be ordered returned
to Sam Sing & Co., which motion was denied in the order dated 24 July 1939. A similar motion
was presented to the justice of the peace of Sagay on 27 October 1939 but was denied the next
day. Meanwhile, an information dated 30 September 1939 had been filed in the CFI Occidental
Negros, charging Yee Fock alias Yee Sue Koy, Y. Tip and A. Sing, managers of Sam Sing & Co.,
with a violation of Act 2655. Before the criminal case could be tried, Yee Sue Koy and Yee Tip
filed the petition with the Supreme Court on 6 November 1939. The petition is grounded on the
propositions (1) that the search warrant issued on 2 May 1938, by the justice of the peace of
Sagay and the seizure accomplished thereunder are illegal, because the warrant was issued
three days ahead of the application therefor and of the affidavit of the Jose Estrada which is
insufficient in itself to justify the issuance of a search warrant, and because the issuance of said
warrant manifestly contravenes the mandatory provisions both of section 1, paragraph 3, of
Article III of the Constitution and of section 97 of General Orders 58, and (2) that the seizure of
the aforesaid articles by means of a search warrant for the purpose of using them as evidence in
the criminal case against the accused, is unconstitutional because the warrant thereby becomes
unreasonable and amounts to a violation of the constitutional prohibition against compelling the
accused to testify against themselves.
Issue: Whether the application of the search warrant is supported by the personal knowledge of
the witness, besides the applicant, for the judge to determine probable cause in issuing the
warrant.

Held: Strict observance of the formalities under section 1, paragraph 3, of Article III of the
Constitution and of section 97 of General Orders 58 was followed. The applicant Mariano G.
Almeda, in his application, swore that “he made his own personal investigation and ascertained
that Sam Sing & Co. is lending money without license, charging usurious rate of interest and is
keeping, utilizing and concealing in the store and premises occupied by it situated at Sagay,
Occidental Negros, documents, notebooks, lists, receipts, promissory notes, and book of
accounts and records, all of which are being used by it in connection with its activities of lending
money at usurious rate of interest in violation of the Usury Law.” In turn, the witness Jose
Estrada, in his testimony before the justice of the peace of Sagay, swore that he knew that Sam
Sing & Co. was lending money without license and charging usurious rate of interest, because he
personally investigated the victims who had secured loans from said Sam Sing & Co. and were
charged usurious rate of interest; that he knew that the said Sam Sing & Co. was keeping and
using books of accounts and records containing its transactions relative its activities as money
lender and the entries of the interest paid by its debtors, because he saw the said Sam Sing & d
make entries and records of their debts and the interest paid thereon. As both Mariano G.
Almeda and Jose Estrada swore that they had personal knowledge, their affidavits were sufficient
for, thereunder, they could be held liable for perjury if the facts would turn out to be not as their
were stated under oath. That the existence of probable cause had been determined by the
justice of the peace of Sagay before issuing the search warrant complained of, is shown by the
following statement in the warrant itself, to wit: “After examination under oath of the
complainant, Mariano G. Almeda, Chief Agent of the Anti-Usury Board, Department of Justice and
Special Agent of the Philippine Army, Manila, and the witness he presented, . . . and this Court,
finding that there is just and probable cause to believe as it does believe, that the above
described articles, relating to the activities of said Sam Sing & Co. of lending money at usurious
rate of interest, are being utilized and kept and concealed at its store and premises occupied by
said Sam Sing & Co., all in violation of law.”

40. Mata vs. Bayona [GR 50720, 26 March 1984]


Second Division, de Castro (J): 3 concur, 2 concur in result, 1 took no part

Facts: Soriano Mata was accused under Presidential Decree (PD) 810, as amended by PD 1306,
the information against him alleging that Soriano Mata offered, took and arranged bets on the Jai
Alai game by “selling illegal tickets known as ‘Masiao tickets’ without any authority from the
Philippine Jai Alai & Amusement Corporation or from the government authorities concerned.”
Mata claimed that during the hearing of the case, he discovered that nowhere from the records
of the said case could be found the search warrant and other pertinent papers connected to the
issuance of the same, so that he had to inquire from the City Fiscal its whereabouts, and to
which inquiry Judge Josephine K. Bayona, presiding Jufe of the City Court of Ormoc replied, “it is
with the court”. The Judge then handed the records to the Fiscal who attached them to the
records. This led Mata to file a motion to quash and annul the search warrant and for the return
of the articles seized, citing and invoking, among others, Section 4 of Rule 126 of the Revised
Rules of Court. The motion was denied by the Judge on 1 March 1979, stating that the court has
made a thorough investigation and examination under oath of Bernardo U. Goles and Reynaldo
T. Mayote, members of the Intelligence Section of 352nd PC Co./Police District II INP; that in fact
the court made a certification to that effect; and that the fact that documents relating to the
search warrant were not attached immediately to the record of the criminal case is of no
moment, considering that the rule does not specify when these documents are to be attached to
the records. Mata’s motion for reconsideration of the aforesaid order having been denied, he
came to the Supreme Court, with the petition for certiorari, praying, among others, that the
Court declare the search warrant to be invalid for its alleged failure to comply with the requisites
of the Constitution and the Rules of Court, and that all the articles confiscated under such
warrant as inadmissible as evidence in the case, or in any proceedings on the matter.

Issue: Whether the judge must before issuing the warrant personally examine on oath or
affirmation the complainant and any witnesses he may produce and take their depositions in
writing, and attach them to the record, in addition to any affidavits presented to him.

Held: Under the Constitution “no search warrant shall issue but upon probable cause to be
determined by the Judge or such other responsible officer as may be authorized by law after
examination under oath or affirmation of the complainant and the witnesses he may produce”.
More emphatic and detailed is the implementing rule of the constitutional injunction, The Rules
provide that the judge must before issuing the warrant personally examine on oath or affirmation
the complainant and any witnesses he may produce and take their depositions in writing, and
attach them to the record, in addition to any affidavits presented to him. Mere affidavits of the
complainant and his witnesses are thus not sufficient. The examining Judge has to take
depositions in writing of the complainant and the witnesses he may produce and to attach them
to the record. Such written deposition is necessary in order that the Judge may be able to
properly determine the existence or nonexistence of the probable cause, to hold liable for perjury
the person giving it if it will be found later that his declarations are false. We, therefore, hold that
the search warrant is tainted with illegality by the failure of the Judge to conform with the
essential requisites of taking the depositions in writing and attaching them to the record,
rendering the search warrant invalid.

41. Olaez vs. People of the Philippines [GR 78347-49, 9 November 1987]
First Division, Cruz (J): 4 concur

Facts: Adolfo Olaes and Linda M. Cruz were charged for violation of the Dangerous Drugs Act.
Olaes and Cruz filed a petition for certiorari and prohibition with preliminary injunction,
challenging the admission by Judge Alicia L. Santos (in her capacity as Presiding Judge of the
Regional Trial Court of Olongapo City, Branch 73) of evidence seized by virtue of an allegedly
invalid search warrant and of an extrajudicial confession taken from them without according
them the right to assistance of counsel; and thus seek to restrain further proceedings in the
criminal case against them and ask that they be acquitted with the setting aside of the
questioned orders (the facts do not provide the disposition of the said orders). Olaes and Cruz
claim that the search warrant issued by the judge is unconstitutional because it does not indicate
the specific offense they are supposed to have committed. There is, therefore, according to
them, no valid finding of probable cause as a justification for the issuance of the said warrant in
conformity with the Bill of Rights.

Issue: Whether the lack of specific section of the Dangerous Drugs Act renders the caption
vague, and negate the claim that the specific offense was committed to serve as basis for the
finding of probable cause.

Held: No. The search warrant issued does not come under the strictures of the Stonehill
doctrine. While in the case cited, there was a bare reference to the laws in general, without any
specification of the particular sections thereof that were alleged to have been violated out of the
hundreds of prohibitions contained in such codifications, there is no similar ambiguity herein.
While it is true that the caption of the search warrant states that it is in connection with
“Violation of RA 6425, otherwise known as the Dangerous Drugs Acts of 1972,” it is clearly
recited in the text thereof that “There is probable cause to believe that Adolfo Olaes alias ‘Debie’
and alias ‘Baby’ of No. 628 Comia St., Filtration, Sta. Rita, Olongapo City, has in their possession
and control and custody of marijuana dried stalks/leaves/seeds/cigarettes and other
regulated/prohibited and exempt narcotics preparations which is the subject of the offense
stated above.” Although the specific section of the Dangerous Drugs Act is not pinpointed, there
is no question at all of the specific offense alleged to have been committed as a basis for the
finding of probable cause. The search warrant also satisfies the requirement in the Bill of Rights
of the particularity of the description to be made of the “place to be searched and the persons or
things to be seized.”

42. 20th Century Fox Film Corporation vs. Court of Appeals [GR L-76649-51, 19 August
1988]
Third Division, Gutierrez J. (J): 4 concur

Facts: In a letter-complaint dated 26 August 1985, 20th Century Fox Film Corporation through
counsel sought the National Bureau of Investigation’s (NBI) assistance in the conduct of searches
and seizures in connection with the NBI’s anti-film piracy campaign. Specifically, the letter-
complaint alleged that certain videotape outlets all over Metro Manila are engaged in the
unauthorized sale and renting out of copyrighted films in videotape form which constitute a
flagrant violation of Presidential Decree 49 (Decree on the Protection of Intellectual Property).
Acting on the letter-complaint, the NBI conducted surveillance and investigation of the outlets
pinpointed by the film corporation and subsequently filed 3 applications for search warrants
against the video outlets owned by Eduardo M. Barreto, Raul Sagullo, and Fortune Ledesma. The
applications were consolidated and heard by the Regional Trial Court (RTC) of Makati, Branch
132. On 4 September 1985, the lower court issued the desired search warrants, describing the
articles sought to be seized as”(c) Television sets, Video Cassettes Recorders, rewinders, tape
head cleaners, accessories, equipments and other machines used or intended to be used in the
unlawful reproduction, sale, rental/lease, distribution of the above-mentioned video tapes which
she is keeping and concealing in the premises above-described.”. Armed with the search
warrants, the NBI accompanied by the film corporation’s agents, raided the video outlets and
seized the items described therein. An inventory of the items seized was made and left with
Barreto, et. al. Acting on a motion to lift search warrants and release seized properties filed by
Barreto, et. al., the lower court issued an order dated 8 October 1985, lifting the 3 search
warrants issued earlier against them by the court, due to the failure of the NBI to deliver the
articles to the Court, and thus ordered the return of the articles to their respective owners. The
lower court denied a motion for reconsideration filed by the film corporation in its order dated 2
January 1986. The film corporation filed a petition for certiorari with the Court of Appeals to annul
the orders of the lower court. The petition was dismissed. The 20th Century Fox Film Corporation
filed the petition for review with the Supreme Court.

Issue: Whether the inclusion of certain articles of property which are usually connected to
legitimate business, and not involving piracy of intellectual property or infringement of copyright
laws, renders the warrant to be unreasonable.

Held: Television sets, video cassette recorders, rewinders and tape cleaners are articles which
can be found in a video tape store engaged in the legitimate business of lending or renting out
betamax tapes. In short, these articles and appliances are generally connected with, or related to
a legitimate business not necessarily involving piracy of intellectual property or infringement of
copyright laws. Hence, including these articles without specification and/or particularity that they
were really instruments in violating an Anti-Piracy law makes the search warrant too general
which could result in the confiscation of all items found in any video store. In fact, this actually
happened in the present case. Although the applications and warrants themselves covered
certain articles of property usually found in a video store, the Court believes that the search
party should have confined themselves to articles that are according to them, evidence
constitutive of infringement of copyright laws or the piracy of intellectual property, but not to
other articles that are usually connected with, or related to, a legitimate business, not involving
piracy of intellectual property, or infringement of copyright laws. So that a television set, a
rewinder, and a whiteboard listing Betamax tapes, video cassette cleaners video cassette
recorders as reflected in the Returns of Search Warrants, are items of legitimate business
engaged in the video tape industry, and which could not be the subject of seizure. The applicant
and his agents therefore exceeded their authority in seizing perfectly legitimate personal
property usually found in a video cassette store or business establishment. The search and
seizure is unreasonable.

43. Yousef Al-Ghoul vs. Court of Appeals [GR 126859, 4 September 2001]
Second Division, Quisumbing (J): 4 concur

Facts: On 31 March 1995, Judge Geronimo S. Mangay, presiding judge of the Regional Trial
Court, National Capital Judicial Region, Branch 125, Kalookan City, issued search warrants 54-95
and 55-95 for the search and seizure of certain items in Apartment 2 at 154 Obiniana Compound,
Deparo Road, Kalookan City. On 1 April 1995, the police searched Apartment 8, in the same
compound and found one (1) .45 caliber pistol. Found in Apartment 2 were 2 M-16 rifles with 2
magazines and 20 live M-16 ammunitions, 1 Bar of demolition charge, 1 Caliber Pistol with no.
634 and other nos. were placed with magazine of Caliber .45 and 3 live 45 ammunitions, 1 22
Caliber handgun with 5 live ammunitions in its cylinder, 1 Box containing 40 pieces of .25 caliber
ammunitions, 2 pieces of fragmentation grenade, 1 roll of detonating cord color yellow, 2 big
bags of ammonium nitrate suspected to be explosives substance, 22 detonating cords with
blasting caps, ½ and ¼ pound of high explosives TNT, 1 timer alarm clock, 2 bags of suspected
gun powder, 2 small plastic bag of suspected explosive substance, 1 small box of plastic bag of
suspected dynamites, One weighing scale, and 2 batteries 9 volts with blasting caps and
detonating cord. The firearms, ammunitions, explosives and other incendiary devices seized at
the apartments were acknowledged in the receipt signed by SPO2 Melanio de la Cruz. Yousef Al
Ghoul, Isam Mohammad Abdulhadi, Wail Rashid Al-Khatib, Nabeel Nasser Al-Riyami, Ashraf
Hassam Al-Yazori, and Mohammad Abushendi were charged before the Regional Trial Court of
Kalookan City, Branch 123, in informations (Criminal Cases C-48666-67) accusing them with
illegal possession of firearms, ammunitions and explosives, pursuant to Presidential Decree
1866. Thereafter, they were arrested and detained. They filed a motion for bail on 24 May 1995,
the resolution of which was held in abeyance by the RTC pending the presentation of evidence
from the prosecution to determine whether or not the evidence presented is strong. On 7
February 1996, at the hearing for bail, the RTC “admitted all exhibits being offered for whatever
purpose that they maybe worth” after the prosecution had finished adducing its evidence despite
the objection by the petitioners on the admissibility of said evidence. On 19 February 1996, the
RTC denied their motion for bail earlier filed. As their action before appellate court also proved
futile, with the appellate court dismissing their special civil action for certiorari, they filed the
petition for review before the Supreme Court.

Issue: Whether the search and seizure orders are valid, and the objects seized admissible in
evidence.

Held: As held in PICOP v. Asuncion, the place to be searched cannot be changed, enlarged nor
amplified by the police. Policemen may not be restrained from pursuing their task with vigor, but
in doing so, care must be taken that constitutional and legal safeguards are not disregarded.
Exclusion of unlawfully seized evidence is the only practical means of enforcing the constitutional
injunction against unreasonable searches and seizures. Hence, the search made at Apartment
No. 8 is illegal and the .45 caliber pistol taken thereat is inadmissible in evidence against Al-
Ghoul, et. al. In contrast, the search conducted at Apartment 2 could not be similarly faulted. The
search warrants specifically mentioned Apartment 2. The search was done in the presence of its
occupants, in accordance with Section 7 of Rule 126, Revised Rules of Court. The articles seized
during the search of Apartment 2 are of the same kind and nature as those items enumerated in
the search warrant. The items seized from Apartment 2 were described with specificity in the
warrants in question. The nature of the items ordered to be seized did not require a technical
description. Moreover, the law does not require that the things to be seized must be described in
precise and minute details as to leave no room for doubt on the part of the searching authorities,
otherwise, it would be virtually impossible for the applicants to obtain a search warrant as they
would not know exactly what kind of things they are looking for. Once described, however, the
articles subject of the search and seizure need not be so invariant as to require absolute
concordance between those seized and those described in the warrant. Substantial similarity of
those articles described as a class or species would suffice.

44. People v. Omaweng [GR 99050, 2 September 1992]


Third Division, Davide (J): 3 concur, 1 on leave

Facts: In the morning of 12 September 1988, PC constables with the Mt. Province PC Command
put up a checkpoint at the junction of the roads, one going to Sagada and the other to Bontoc.
They stopped and checked all vehicles that went through the checkpoint. At 9:15 a.m., they
flagged down a cream-colored Ford Fiera (ABT-634) coming from the Bontoc Poblacion and
headed towards Baguio. The vehicle was driven by Conway Omaweng and had no passengers.
The Constables (Layong, et.al.) asked permission to inspect the vehicle to which Omaweng
acceded to. When they peered into the rear of the vehicle, they saw a travelling bag which was
partially covered by the rim of a spare tire under the passenger seat on the right side of the
vehicle. They asked permission to see the contents of the bag to which Omaweng consented to.
When they opened the bag, they found that it contained 41 plastic packets of different sizes
containing pulverized substances. The constable gave a packet to his team leader, who, after
sniffing the stuff concluded that it was marijuana. The Constables thereafter boarded the
vehicles and proceeded to the Bontoc poblacion to report the incident to the PC Headquarters.
The prohibited drugs were surrendered to the evidence custodian. The PC Forensic Chemist at
Camp Dangwa, La Trinidad, Benguet conducted 2 chemistry examinations of the substance
contained in the plastic packets taken from appellant and found them to be positive for hashish
or marijuana. Omaweng was indicted for the violation of Section 4, Article II of RA 6425
(Dangerous Drugs Act of 1972), as amended, in a criminal complaint filed with the MTC Bontoc,
Mountain Province on 12 September 1988. Upon his failure to submit counter-affidavits despite
the granting of an extension of time to do so, the court declared that he had waived his right to a
preliminary investigation and, finding probable cause against Omaweng, ordered the elevation of
the case to the proper court. On 14 November 1988, the Office of the Provincial Fiscal of
Mountain Province filed an Information charging Omaweng with the violation of Section 47 Article
II of the Dangerous Drugs Act of 1972, as amended (Crim Case 713). After his motion for
reinvestigation was denied by the Provincial Fiscal, Omaweng entered a plea of not guilty during
his arraignment on 20 June 1989. During the trial on the merits, the prosecution presented 4
witnesses. Omaweng did not present any evidence other than portions of the Joint Clarificatory
Sworn Statement, dated 23 December 1988, of prosecution witnesses Joseph Layong and David
Fomocod. On 21 March 1991, the trial court promulgated its Judgment convicting Omaweng of
the crime of transporting prohibited drugs (Section 4, Article II of RA 6425, as amended).
Omaweng appealed to the Supreme Court.

Issue: Whether Omaweng was subjected to search which violates his Constitutional right against
unreasonable searches and seizures.

Held: Omaweng was not subjected to any search which may be stigmatized as a violation of his
Constitutional right against unreasonable searches and seizures. He willingly gave prior consent
to the search and voluntarily agreed to have it conducted on his vehicle and travelling bag. The
testimony of the PC Constable (Layung) was not dented on cross-examination or rebutted by
Omaweng for he chose not to testify on his own behalf. Omaweng waived his right against
unreasonable searches and seizures when he voluntarily submitted to a search or consents to
have it made in his person or premises. He is precluded from later complaining thereof right to
be secure from unreasonable search may, like every right, be waived and such waiver may be
made either expressly or impliedly. Since in the course of the valid search 41 packages of drugs
were found, it behooved the officers to seize the same; no warrant was necessary for such
seizure.

45. People vs. Correa [GR 119246, 30 January 1998]


En Banc, Martinez (J): 12 concur
Facts: A week before 18 June 1994, Leonardo Dulay was placed under surveillance by the Police
Operatives from the Drug Enforcement Unit of the Western Police District Command (DEU-WPDC)
on account of confidential and intelligence reports received in said Unit about his drug trafficking
around Bambang Street, Tondo, Manila. The police surveillance brought forth positive results and
confirmed Dulay’s illegal drug trade. On 17 June 1994, operatives were alerted that Dulay would
transport and deliver a certain quantity of drugs that night on board a owner-type jeep
(FMR948). Thereafter, the operatives, together with the informer proceeded to A. Bonifacio
Street on board 3 vehicles, and inconspicuously parked along the side of North Cemetery and
waited for the suspect. The police informant spotted Dulay’s vehicle at 3:00 am. The operatives
tailed the subject jeepney until they reached Bambang extension and Jose Abad Santos Avenue,
where they accosted the passengers of said jeepney. The team inspected a cylindrical tin can of
El Cielo Vegetable Cooking Lard, about two feet high, loaded in the vehicle of the appellants. The
can contained 8 bundles of suspected dried marijuana flowering tops wrapped in pieces of paper
and plastic tapes. The team seized the suspected contrabands and marked each bundle
consecutively. The 3 suspects were brought to the police headquarters at DEU-WPDC for
investigation. The packages of suspected marijuana were submitted to the NBI for laboratory
analysis to determine their chemical composition. The tests confirmed that the confiscated stuff
were positive for marijuana and weighed 16.1789 kilograms. The defense, however, contends
that the 3 accused were arrested without warrant in Camarin D, Caloocan City, enroute to
Dulay’s house to get the things of his child allegedly rushed previously to the Metropolitan
Hospital, for an alleged charge of trafficking on ’shabu,’ and were brought to the WPDC
headquarters at U.N. Avenue, where they were detained. On 12 July 1994, an Information was
filed with the RTC Manila (Branch 35) indicting Antonio Correa y Cayton @ “Boyet,” Rito Gunida y
Sesante @ “Dodong,” and Leonardo Dulay y Santos @ “Boy Kuba” for having violated Section 4,
Article II of RA 6425, as amended. When arraigned, the 3 accused pleaded not guilty. After trial
and on 3 March 1995, the lower court found the appellants guilty as charged and were sentenced
to death and a fine of P10 million.

Issue: Whether the accused are precluded from assailing the warrantless search and seizure,
due to waiver on their part.

Held: Antonio Correa y Cayton @ “Boyet,” Rito Gunida y Sesante @ “Dodong,” and Leonardo
Dulay y Santos @ “Boy Kuba” are precluded from assailing the warrantless search and seizure
when they voluntarily submitted to it as shown by their actuation during the search and seizure.
They never protested when the police officer opened the tin can loaded in their vehicle, nor when
he opened one of the bundles, nor when they, together with their cargo of drugs and their
vehicle, were brought to the police station for investigation and subsequent prosecution. When
one voluntarily submits to a search or consents to have it made on his person or premises, he is
precluded from later complaining thereof The right to be secure from unreasonable search may,
like every right, be waived and such waiver may be made either expressly or impliedly.” Further,
they effectively waived their constitutional right against the search and seizure by their
voluntary submission to the jurisdiction of the trial court, when they entered a plea of not guilty
upon arraignment and by participating in the trial.

46. People v. Ramos [GR 85401-02, 4 June 1990]


Third Division, Gutierrez Jr. (J): 3 concur, 1 took no part

Facts: On 29 November 1982, a civilian informer came to the Narcotics Command Office in
Olongapo City and reported that a cigarette vendor by the name of “Mama Rose” (Rosalinda
Ramos) was selling marijuana at the corner of 3rd Street and Rizal Avenue in Olongapo City.
Tests buys were made using marked money. The Narcotics Command (NARCOM) team
proceeded to the place where appellant was selling cigarettes, and arrested the latter for illegal
peddling of marijuana. Ramos was requested to take out the contents of her wallet. The four
marked five-peso bills used in the test buys were found among her possessions and were
confiscated after the serial numbers were confirmed. Search of Ramos’ stall yielded 20 sticks of
marijuana cigarettes in a trash can placed under the small table where Ramos displayed the
wares she was selling. Ramos was thereafter brought to the station. At the station, Ramos
executed a statement confessing to her crimes which she swore to before Assistant City Fiscal.
The marijuana sticks confiscated were sent to the Philippine Constabulary Crime Laboratory
(PCCL) for analysis, and thereafter were confirmed to be marijuana. The defense contends
however that she assented to the invitation of the NARCOM operatives for investigation, after
search of her buri bags (which she stores the fruits that she sells) were fruitless. She claimed
that she was forced to affix her signature on the four 5-peso bills by one Sgt. Sudiacal,
purportedly to be the same money which was used to buy marijuana from her, but which she
insists was her money being saved for the rentals. She was later brought to the Fiscal’s Office
after investigation, where she signed a document. She claimed she was not assisted by any
counsel during the investigation, neither during the time she signed the document at the Fiscal’s
Office. Two informations were filed against Ramos, one for sale (Criminal Case 5991) and the
other for possession of marijuana (Criminal Case 5990). After trial, the RTC Olongapo City
(Branch 73) found her guilty beyond reasonable doubt in Criminal Case 5990 for violating Section
8 of RA 6425 and sentenced her to imprisonment of 6 years and 1 day and a fine of P6,000. She
was likewise found guilty beyond reasonable doubt in Criminal Case 5991 for violating Section 4
of RA 6425 and was sentenced to life imprisonment and a fine of P20,000. Ramos sought
reversal of the decisions with the Supreme Court.

Issue: Whether Ramos waived her right against the warrantless search of the trash can, where
illegal drugs were found, under her control.

Held: The trash can (where the contraband were found) was found under the table where her
legitimate wares were being sold. Ramos he was the only person who had access to the trash
can. The same was under her immediate physical control. She had complete charge of the
contents of the trash can under the table to the exclusion of all other persons. In law, actual
possession exists when the thing is in the immediate occupancy and control of the party. But this
is not to say that the law requires actual possession. In criminal law, possession necessary for
conviction of the offense of possession of controlled substances with intent to distribute may be
constructive as well as actual. It is only necessary that the defendant must have dominion and
control over the contraband. These requirements are present in the situation described, where
the prohibited drugs were found inside the trash can placed under the stall owned by Ramos. In
fact, the NARCOM agents who conducted the search testified that they had to ask Ramps to
stand so that they could look inside the trash can under Ramos’ papag. The trash can was
positioned in such a way that it was difficult for another person to use the trash can. The trash
can was obviously not for use by her customers. Therefore, the twenty sticks of marijuana are
admissible in evidence and the trial court’s finding that Ramos is guilty of possession is correct.

47. People vs. dela Cruz [GR 83260, 18 April 1990]


Second Division, Regalado (J): 4 concur

Facts: After receiving a confidential report from Arnel, their informant, a “buy-bust” operation
was conducted by the 13th Narcotics Regional Unit through a team composed of T/Sgt. Jaime
Raposas as Team Leader, S/Sgt. Rodelito Oblice, Sgt. Dante Yang, Sgt. Vicente Jimenez, P/Pfc.
Adolfo Arcoy as poseur-buyer and Pat. Deogracias Gorgonia at Maliclic St., Tondo, Manila at
around 2:30 p.m. of 4 May 1987 to catch the pusher/s. P/Pfc. Adolfo Arcoy acted as the poseur-
buyer with Arnel as his companion to buy marijuana worth P10.00 from the two accused, Juan de
la Cruz and Reynaldo Beltran. At the scene, it was Juan de la Cruz whom Arcoy first negotiated
with on the purchase and when Arcoy told De la Cruz that he was buying P10.00 worth of
marijuana, De la Cruz instructed Reynaldo Beltran to give one aluminum foil of marijuana which
Beltran got from his pants’ pocket and delivered it to Arcoy. After ascertaining that the foil of
suspected marijuana was really marijuana, Arcoy gave the prearranged signal to his teammates
by scratching his head and his teammates who were strategically positioned in the vicinity,
converged at the place, identified themselves as NARCOM agents and effected the arrest of De la
Cruz and Beltran. The P10.00 marked bill used by Arcoy was found in the possession of Juan de la
Cruz together with two aluminum foils and containing marijuana. Juan de la Cruz y Gonzales and
Reynaldo Beltran y Aniban were charged in Criminal Case 87-54417 of the Regional Trial Court
(RTC) of Manila with violation of Section 4, Art. II, in relation to Section 21, Article IV of Republic
Act 6425, as amended. The court, on 15 March 1988, found Dela Cruz and Beltran guilty beyond
reasonable doubt and sentenced each of them to suffer the penalty of reclusion perpetua, with
the accessory penalties provided by law; to pay a fine of P20,000.00, without subsidiary
imprisonment in case of insolvency, and each to pay one-half of the costs. From this decision, de
la Cruz and Beltran appealed. In a letter of the Warden, Manila City Jail, dated 3 March 1989, the
Court was informed of the death of de la Cruz on 21 February 1989. Thus, the criminal case
against de la Cruz was dismissed in the Supreme Court resolution of 25 September 1989. The
present appellate proceeding is limited only to Beltran.

Issue: Whether the warrantless seizure incidental to the buy-bust operation violates Beltran’s
constitutional rights against unreasonable search and seizure.

Held: A buy-bust operation is the method employed by peace officers to trap and catch a
malefactor in flagrante delicto. It is essentially a form of entrapment since the peace officer
neither instigates nor induces the accused to commit a crime. Entrapment is the employment of
such ways and means for the purpose of trapping or capturing a lawbreaker from whose mind
the criminal intent originated. Oftentimes, it is the only effective way of apprehending a criminal
in the act of the commission of the offense. While it is conceded that in a buy-bust operation,
there is seizure of evidence from one’s person without a search warrant, needless to state a
search warrant is not necessary, the search being incident to a lawful arrest. A peace officer
may, without a warrant, arrest a person when, in his presence, the person to be arrested has
committed, is actually committing or is attempting to commit an offense. It is a matter of judicial
experience that in the arrest of violators of the Dangerous Drugs Act in a buy-bust operation, the
malefactors were invariably caught red-handed. There being no violation of the constitutional
right against unreasonable search and seizure, the confiscated articles are admissible in
evidence.

48. Espano vs. Court of Appeals [GR 120431, 1 April 1998]


Third Division, Romero (J): 3 concur

Facts: On 14 July 1991, at about 12:30 a.m., Pat. Romeo Pagilagan and other police officers,
namely, Pat. Wilfredo Aquilino, Simplicio Rivera, and Erlindo Lumboy of the Western Police
District (WPD), Narcotics Division went to Zamora and Pandacan Streets, Manila to confirm
reports of drug pushing in the area. They saw Rodolfo Espano selling “something” to another
person. After the alleged buyer left, they approached Espano, identified themselves as
policemen, and frisked him. The search yielded two plastic cellophane tea bags of marijuana .
When asked if he had more marijuana, he replied that there was more in his house. The
policemen went to his residence where they found ten more cellophane tea bags of marijuana.
Espano was brought to the police headquarters where he was charged with possession of
prohibited drugs. On 24 July 1991, Espano posted bail and the trial court issued his order of
release on 29 July 1991. On 14 August 1992, the trial court rendered a decision, convicting
Espano of the crime charged. Espano appealed the decision to the Court of Appeals. The
appellate court, however, on 15 January 1995 affirmed the decision of the trial court in toto.
Espano filed a petition for review with the Supreme Court.

Issue: Whether the search of Espano’s home after his arrest does not violate against his right
against unreasonable search and seizure.

Held: Espano’s arrest falls squarely under Rule 113 Section 5(a) of the Rules of Court. He was
caught in flagranti as a result of a buy-bust operation conducted by police officers on the basis of
information received regarding the illegal trade of drugs within the area of Zamora and Pandacan
Streets, Manila. The police officer saw Espano handing over something to an alleged buyer. After
the buyer left, they searched him and discovered two cellophanes of marijuana. His arrest was,
therefore, lawful and the two cellophane bags of marijuana seized were admissible in evidence,
being the fruits of the crime. As for the 10 cellophane bags of marijuana found at Espano’s
residence, however, the same inadmissible in evidence. The articles seized from Espano during
his arrest were valid under the doctrine of search made incidental to a lawful arrest. The
warrantless search made in his house, however, which yielded ten cellophane bags of marijuana
became unlawful since the police officers were not armed with a search warrant at the time.
Moreover, it was beyond the reach and control of Espano. The right of the people to be secure in
their persons, houses, papers and effects against unreasonable searches and seizures of
whatever nature and for any purposes shall be inviolable, and no search warrant or warrant of
arrest shall issue except upon probable cause to be determined personally by the judge after
examination under oath or affirmation of the complainant and the witnesses he may produce,
and particularly describing the place to be searched and the persons or things to be seized.” An
exception to the said rule is a warrantless search incidental to a lawful arrest for dangerous
weapons or anything which may be used as proof of the commission of an offense. It may extend
beyond the person of the one arrested to include the premises or surroundings under his
immediate control. Herein, the ten cellophane bags of marijuana seized at petitioner’s house
after his arrest at Pandacan and Zamora Streets do not fall under the said exceptions.

49. Arizona v. Hicks [480 US 321, 3 March 1987]


Scalia (J): 4 concur, 1 filed a separate concurring opinion, 2 filed separate dissenting opinions to
which 1 concurs.

Facts: On 18 April 1984, a bullet was fired through the floor of Hicks’ apartment, striking and
injuring a man in the apartment below. Police officers arrived and entered Hicks’ apartment to
search for the shooter, for other victims, and for weapons. They found and seized three weapons,
including a sawed-off rifle, and in the course of their search also discovered a stocking-cap mask.
One of the policemen, Officer Nelson, noticed two sets of expensive stereo components, which
seemed out of place in the squalid and otherwise ill-appointed four-room apartment. Suspecting
that they were stolen, he read and recorded their serial numbers - moving some of the
components, including a Bang and Olufsen turntable, in order to do so - which he then reported
by phone to his headquarters. On being advised that the turntable had been taken in an armed
robbery, he seized it immediately. It was later determined that some of the other serial numbers
matched those on other stereo equipment taken in the same armed robbery, and a warrant was
obtained and executed to seize that equipment as well. Hicks was subsequently indicted for the
robbery. The state trial court granted Hicks’ motion to suppress the evidence that had been
seized. The Court of Appeals of Arizona affirmed. It was conceded that the initial entry and
search, although warrantless, were justified by the exigent circumstance of the shooting. The
Court of Appeals viewed the obtaining of the serial numbers, however, as an additional search,
unrelated to that exigency. Both courts - the trial court explicitly and the Court of Appeals by
necessary implication - rejected the State’s contention that Officer Nelson’s actions were justified
under the “plain view” doctrine of Coolidge v. New Hampshire. The Arizona Supreme Court
denied review, and the State filed this petition.

Issue: Whether the policeman’s actions come within the purview of the Fourth Amendment.

Held: The policeman’s actions come within the purview of the Fourth Amendment. The mere
recording of the serial numbers did not constitute a “seizure” since it did not meaningfully
interfere with Hicks’ possessory interest in either the numbers or the stereo equipment.
However, the moving of the equipment was a “search” separate and apart from the search that
was the lawful objective of entering the apartment. The fact that the search uncovered nothing
of great personal value to Hicks is irrelevant. The “plain view” doctrine does not render the
search “reasonable” under the Fourth Amendment. The policeman’s action directed to the stereo
equipment was not ipso facto unreasonable simply because it was unrelated to the justification
for enteringthe apartment. That lack of relationship always exists when the “plain view” doctrine
applies. In saying that a warrantless search must be “strictly circumscribed by the exigencies
which justify its initiation,” Mincey was simply addressing the scope of the primary search itself,
and was not overruling the “plain view” doctrine by implication. However, the search was invalid
because, as the State concedes, the policeman had only a “reasonable suspicion” - i. e., less
than probable cause to believe - that the stereo equipment was stolen. Probable cause is
required to invoke the “plain view” doctrine as it applies to seizures. It would be illogical to hold
that an object is seizable on lesser grounds, during an unrelated search and seizure, than would
have been needed to obtain a warrant for it if it had been known to be on the premises. Probable
cause to believe the equipment was stolen was also necessary to support the search here,
whether legal authority to move the equipment could be found only as the inevitable
concomitant of the authority to seize it, or also as a consequence of some independent power to
search objects in plain view. The policeman’s action cannot be upheld on the ground that it was
not a “full-blown search” but was only a “cursory inspection” that could be justified by
reasonable suspicion instead of probable cause. A truly cursory inspection - one that involves
merely looking at what is already exposed to view, without disturbing it - is not a “search” for
Fourth Amendment purposes, and therefore does not even require reasonable suspicion. This
Court is unwilling to create a subcategory of “cursory” searches under the Fourth Amendment.

50. People vs. Gatward [GRs 119772-73, 7 February 1997]


Second Division, Regalado (J): 4 concur

Facts: At about 3:30 p.m. of 30 August 1994, U Aung Win, a Passenger of TG Flight 620 of the
Thai Airways which had just arrived from Bangkok, Thailand, presented his luggage, a travelling
bag for examination to Customs Examiner Busran Tawano, who was assigned at the Arrival Area
of the Ninoy Aquino International Airport (NAIA) in Pasay City. U Aung Win also handed to
Tawano his Customs Declaration 128417 stating that he had no articles to declare. When Tawano
was about to inspect his luggage, U Aung Win suddenly left, proceeding towards the direction of
Carousel 1, the conveyor for the pieces of luggage of the passengers of Flight 620, as if to
retrieve another baggage from it. After having inspected the luggages of the other incoming
passengers, Tawano became alarmed by the failure of U Aung Win to return and suspected that
the bag of U Aung Win contained illegal articles. The Customs Examiner reported the matter to
his superiors. Upon their instructions, the bag was turned over to the office of the Customs Police
in the NAIA for x-ray examination where it was detected that it contained some powdery
substance. When opened, the bag revealed two packages containing the substance neatly
hidden in between its partitions. Representative samples of the substance were examined by
Elizabeth Ayonon, a chemist of the Crime Laboratory Service of the Philippine National Police
(PNP) assigned at the Arrival Area of the NAIA, and by Tita Advincula, another chemist of the PNP
Crime Laboratory Service at Camp Crame, and found to be positive for heroin. The two chemists
concluded that the entire substance, with a total weight of 5,579.80 grams, contained in the two
packages found in the bag of U Aung Win, is heroin. A manhunt was conducted to locate U Aung
Win. At about 7:45 p.m. of the same date, Rey Espinosa, an employee of the Lufthansa Airlines,
notified the commander of the NAIA Customs Police District Command that a certain Burmese
national by the name of U Aung Win appeared at the check-in counter of the airline as a
departing passenger. Immediately, a team of law enforcers proceeded to the Departure Area and
apprehended the accused after he had been identified through his signatures in his Customs
Declaration and in his Bureau of Immigration and Deportation Arrival Card. Customs Examiner
Tawano also positively identified U Aung Win as the person who left his bag with him at the
Arrival Area of the NAIA. During the investigation of U Aung Win, the agents of the Customs
Police and the Narcotics Command (NARCOM) gathered the information that U Aung Win had a
contact in Bangkok and that there were other drug couriers in the Philippines. Following the lead,
a team of lawmen, together with U Aung Win, was dispatched to the City Garden Hotel in Mabini
St., Ermita, Manila, to enable U Aung Win to communicate with his contact in Bangkok for further
instructions. While the police officers were standing by, they noticed two persons, a Caucasian
and an oriental, alight from a car and enter the hotel. U Aung Win whispered to Customs Police
Special Agent Edgar Quiñones that he recognized the two as drug couriers whom he saw talking
with his contact in Bangkok named Mau Mau. The members of the team were able to establish
the identity of the two persons as Nigel Richard Gatward and one Zaw Win Naing, a Thailander,
from the driver of the hotel service car used by the two when they arrived in the hotel. It was
gathered by the law enforcers that Gatward and Zaw Win Naing were scheduled to leave for
Bangkok on board a KLM flight. On 31 August 1994, operatives of the NAIA Customs Police
mounted a surveillance operation at the Departure Area for Gatward and Zaw Win Naing who
might be leaving the country. At about 7:45 p.m., Special Agent Gino Minguillan of the Customs
Police made a verification on the passenger manifest of KLM Royal Dutch Airlines Flight 806,
bound for Amsterdam via Bangkok, which was scheduled to depart at about 7:55 p.m. He found
the name “GATWARD/NRMR” listed therein as a passenger for Amsterdam and accordingly
informed his teammates who responded immediately Customs Police Captain Juanito Algenio
requested Victorio Erece, manager of the KLM airline at the NAIA, to let passenger Gatward
disembark from the aircraft and to have his checked-in luggage, if any, unloaded. The manager
acceded to the request to off-load Gatward but not to the unloading of his check-in bag as the
plane was about to depart and to do so would unduly delay the flight. However, Erece made an
assurance that the bag would be returned immediately to the Philippines on the first available
flight from Bangkok. Upon his disembarkment. Gatward was invited by the police officers for
investigation. At about 3:00 p.m. of 1 September 1994, Gatward’s luggage, was brought back to
the NAIA from Bangkok through the Thai airways, pursuant to the request of Erece. Upon its
retrieval, the law enforcers subjected the bag to x-ray examinations in the presence of Gatward
and some Customs officials. It was observed to contain some powdery substance. Inside the bag
were two improvised envelopes made of cardboard each containing the powdery substance,
together with many clothes. The envelopes were hidden inside the bag, one at the side in
between a double-wall, the other inside a partition in the middle. Upon its examination by
Chemists Ayonon and Advincula pursuant to the request of Police Senior Inspector John Campos
of the NARCOM, the powdery substance contained in the two cardboard envelopes, with a net
weight of 5,237.70 grams, was found to be heroin. Nigel Richard Gatward was charged with
violating Section 4 of Republic Act 6425, the Dangerous Drugs Act of 1972 (transporting); while U
Aung Win was charged for transgressing Section 3 of the Dangerous Drugs Act of 1972
(importing). Gatward pleaded not guilty of the charge when arraigned, while U Aung Win pleaded
guilty of the crime charged upon his arraignment. On 3 March 1995, the trial court found both
guilty of the crime charged.

Issue: Whether Gatward’s and U Aung Win’s suitcases may be searched without warrant.

Held: While no search warrant had been obtained for that purpose, when Gatward checked in
his bag as his personal luggage as a passenger of KLM Flight 806 he thereby agreed to the
inspection thereof in accordance with customs rules and regulations, an international practice of
strict observance, and waived any objection to a warrantless search. His subsequent arrest,
although likewise without a warrant, was justified since it was effected upon the discovery and
recovery of the heroin in his bag, or in flagrante delicto. The conviction of U Aung Win is likewise
unassailable. His culpability was not based only upon his plea of guilty but also upon the
evidence of the prosecution, the presentation of which was required by the lower court despite
said plea. The evidence thus presented convincingly proved his having imported into this country
the heroin found in his luggage which he presented for customs examination upon his arrival at
the international airport. There was, of course, no showing that he was authorized by law to
import such dangerous drug, nor did he claim or present any authority to do so.

51. Terry vs. Ohio [392 US 1, 10 June 1968]


Warren (J)

Facts: Martin McFadden, a Cleveland police detective, on a downtown beat which he had been
patrolling for many years, observed two strangers (Terry and Richard Chilton) on a street corner.
He saw them proceed alternately back and forth along an identical route, strolling down Huron
Road, pausing to stare in the same store window, which they did for a total of about 24 times.
Each completion of the route was followed by a conference between the two on a corner, at one
of which they were joined by a third man (Katz) who left swiftly. Suspecting the two men of
“casing a job, a stick-up,” the officer followed them and saw them rejoin the third man a couple
of blocks away in front of a store. The officer approached the three, identified himself as a
policeman, and asked their names. The men “mumbled something,” whereupon McFadden spun
Terry around, patted down his outside clothing, and found in his overcoat pocket, but was unable
to remove, a pistol. The officer ordered the three into the store. He removed Terry’s overcoat,
took out a revolver, and ordered the three to face the wall with their hands raised. He patted
down the outer clothing of Chilton and Katz and seized a revolver from Chilton’s outside overcoat
pocket. He did not put his hands under the outer garments of Katz (since he discovered nothing
in his pat-down which might have been a weapon), or under Terry’s or Chilton’s outer garments
until he felt the guns. The three were taken to the police station. Terry and Chilton were charged
with carrying concealed weapons. The defense moved to suppress the weapons. Though the trial
court rejected the prosecution theory that the guns had been seized during a search incident to a
lawful arrest, the court denied the motion to suppress and admitted the weapons into evidence
on the ground that the officer had cause to believe that Terry and Chilton were acting
suspiciously, that their interrogation was warranted, and that the officer for his own protection
had the right to pat down their outer clothing having reasonable cause to believe that they might
be armed. The court distinguished between an investigatory “stop” and an arrest, and between a
“frisk” of the outer clothing for weapons and a full-blown search for evidence of crime. Terry and
Chilton were found guilty, an intermediate appellate court affirmed, and the State Supreme Court
dismissed the appeal on the ground that “no substantial constitutional question” was involved.

Issue: Whether it is always unreasonable for a policeman to seize a person and subject him to a
limited search for weapons unless there is probable cause for an arrest.

Held: The Fourth Amendment right against unreasonable searches and seizures, made
applicable to the States by the Fourteenth Amendment, “protects people, not places,” and
therefore applies as much to the citizen on the streets as well as at home or elsewhere. The
issue in this case is not the abstract propriety of the police conduct but the admissibility against
petitioner of the evidence uncovered by the search and seizure. The exclusionary rule cannot
properly be invoked to exclude the products of legitimate and restrained police investigative
techniques; and this Court’s approval of such techniques should not discourage remedies other
than the exclusionary rule to curtail police abuses for which that is not an effective sanction. The
Fourth Amendment applies to “stop and frisk” procedures such as those followed here.
Whenever a police officer accosts an individual and restrains his freedom to walk away, he has
“seized” that person within the meaning of the Fourth Amendment. A careful exploration of the
outer surfaces of a person’s clothing in an attempt to find weapons is a “search” under that
Amendment. Where a reasonably prudent officer is warranted in the circumstances of a given
case in believing that his safety or that of others is endangered, he may make a reasonable
search for weapons of the person believed by him to be armed and dangerous regardless of
whether he has probable cause to arrest that individual for crime or the absolute certainty that
the individual is armed. Though the police must whenever practicable secure a warrant to make
a search and seizure, that procedure cannot be followed where swift action based upon on-the-
spot observations of the officer on the beat is required. The reasonableness of any particular
search and seizure must be assessed in light of the particular circumstances against the
standard of whether a man of reasonable caution is warranted in believing that the action taken
was appropriate. The officer here was performing a legitimate function of investigating
suspicious conduct when he decided to approach Terry and his companions. An officer justified in
believing that an individual whose suspicious behavior he is investigating at close range is armed
may, to neutralize the threat of physical harm, take necessary measures to determine whether
that person is carrying a weapon. A search for weapons in the absence of probable cause to
arrest must be strictly circumscribed by the exigencies of the situation. An officer may make an
intrusion short of arrest where he has reasonable apprehension of danger before being
possessed of information justifying arrest. The officer’s protective seizure of Terry and his
companions and the limited search which he made were reasonable, both at their inception and
as conducted. The actions of Terry and his companions were consistent with the officer’s
hypothesis that they were contemplating a daylight robbery and were armed. The officer’s
search was confined to what was minimally necessary to determine whether the men were
armed, and the intrusion, which was made for the sole purpose of protecting himself and others
nearby, was confined to ascertaining the presence of weapons. Herein, Officer McFadden patted
down the outer clothing of Terry and his two companions. He did not place his hands in their
pockets or under the outer surface of their garments until he had felt weapons, and then he
merely reached for and removed the guns. He did not conduct a general exploratory search for
whatever evidence of criminal activity he might find. Thus, the revolver seized from Terry was
properly admitted in evidence against him. At the time McFadden seized Terry and searched him
for weapons, Officer McFadden had reasonable grounds to believe that Terry was armed and
dangerous, and it was necessary for the protection of himself and others to take swift measures
to discover the true facts and neutralize the threat of harm if it materialized. The policeman
carefully restricted his search to what was appropriate to the discovery of the particular items
which he sought. Each case of this sort will, of course, have to be decided on its own facts.
Where a police officer observes unusual conduct which leads him reasonably to conclude in light
of his experience that criminal activity may be afoot and that the persons with whom he is
dealing may be armed and presently dangerous, where in the course of investigating this
behavior he identifies himself as a policeman and makes reasonable inquiries, and where
nothing in the initial stages of the encounter serves to dispel his reasonable fear for his own or
others’ safety, he is entitled for the protection of himself and others in the area to conduct a
carefully limited search of the outer clothing of such persons in an attempt to discover weapons
which might be used to assault him. Such a search is a reasonable search under the Fourth
Amendment, and any weapons seized may properly be introduced in evidence against the
person from whom they were taken.

52. People v. Solayao [GR 119220, 20 September 1996]


Second Division, Romero (J): 4 concur

Facts: On 9 June 1992, CAFGU members, headed by SPO3 Nino, were conducting an intelligence
patrol to verify reports on the presence of armed persons roaming around the barangays of
Caibiran. In Baragay Onion, they met the 5-man group of accused Nilo Solayao, who was also
wearing a camouflage uniform. His companions, upon seeing the government agents, fled. SPO3
Niño told Salayao not to run away and introduced himself as “PC,” after which he seized the
dried coconut leaves which the latter was carrying and found wrapped in it a 49-inch long
homemade firearm locally known as “latong.” When he asked Salayao who issued him a license
to carry said firearm or whether he was connected with the military or any intelligence group,
the latter answered that he had no permission to possess the same. Thereupon, SPO3 Niño
confiscated the firearm and turned him over to the custody of the policemen of Caibiran who
subsequently investigated him and charged him with illegal possession of firearm. Salayao did
not contest the confiscation of the shotgun but averred that this was only given to him by one of
his companions, Hermogenes Cenining, when it was still wrapped in coconut leaves, which they
were using the coconut leaves as a torch. Salayao’s claim was corroborated by one Pedro
Balano. On 15 August 1994, the RTC of Naval Biliran (Branch 16) found Salayao guilty of illegal
possession of firearm under Section 1 of PD 1866 and imposed upon him the penalty of
imprisonment ranging from reclusion temporal maximum to reclusion perpetua. The trial court,
having found no mitigating but one aggravating circumstance of nighttime, sentenced accused-
appellant to suffer the prison term of reclusion perpetua with the accessory penalties provided
by law. Salayao appealed to the Supreme Court.

Issue: Whether the search upon Solayao, yielding the firearm wrapped in coconut leaves, is
valid.

Held: Nilo Solayao and his companions’ drunken actuations aroused the suspicion of SPO3
Niño’s group, as well as the fact that he himself was attired in a camouflage uniform or a jungle
suit and that upon espying the peace officers, his companions fled. It should be noted that the
peace officers were precisely on an intelligence mission to verify reports that armed persons
were roaming around the barangays of Caibiran. The circumstances are similar to those
obtaining in Posadas v. Court of Appeals where this Court held that “at the time the peace
officers identified themselves and apprehended the petitioner as he attempted to flee, they did
not know that he had committed, or was actually committing the offense of illegal possession of
firearm and ammunitions. They just suspected that he was hiding something in the buri bag.
They did not know what its contents were. The said circumstances did not justify an arrest
without a warrant.” As with Posadas, the case herein constitutes an instance where a search and
seizure may be effected without first making an arrest. There was justifiable cause to “stop and
frisk” Solayao when his companions fled upon seeing the government agents. Under the
circumstances, the government agents could not possibly have procured a search warrant first.
Thus, there was no violation of the constitutional guarantee against unreasonable searches and
seizures. Nor was there error on the part of the trial court when it admitted the homemade
firearm as evidence.

53. Papa vs. Mago [GR L-27360, 28 February 1968]


En Banc, Zaldivar (J): 9 concur

Facts: Martin Alagao, head of the counter-intelligence unit of the Manila Police Department,
acting upon a reliable information received on 3 November 1966 to the effect that a certain
shipment of personal effects, allegedly misdeclared and undervalued, would be released the
following day from the customs zone of the port of Manila and loaded on two trucks, and upon
orders of Ricardo Papa, Chief of Police of Manila and a duly deputized agent of the Bureau of
Customs, conducted surveillance at gate 1 of the customs zone. When the trucks left gate 1 at
about 4:30 p.m. of 4 November 1966, elements of the counter-intelligence unit went after the
trucks and intercepted them at the Agrifina Circle, Ermita, Manila. The load of the two trucks,
consisting of nine bales of goods, and the two trucks, were seized on instructions of the Chief of
Police. Upon investigation, a person claimed ownership of the goods and showed to the
policemen a “Statement and Receipts of Duties Collected on Informal Entry No. 147-5501″,
issued by the Bureau of Customs in the name of a certain Bienvenido Naguit. Claiming to have
been prejudiced by the seizure and detention of the two trucks and their cargo, Remedios Mago
and Valentin B. Lanopa filed with the Court of First Instance (CFI) of Manila a petition “for
mandamus with restraining order or preliminary injunction (Civil Case 67496), praying for the
issuance of a restraining order, ex parte, enjoining the police and customs authorities, or their
agents, from opening the bales and examining the goods, and a writ of mandamus for the return
of the goods and the trucks, as well as a judgment for actual, moral and exemplary damages in
their favor. On 10 November 1966, Judge Hilarion Jarencio issued an order ex parte restraining
Ricardo Papa (as Chief of Police of Manila) and Juan Ponce Enrile (as Commissioner of Customs)
in Civil Case 67496. However, when the restraining order was received by Papa. et. al., some
bales had already been opened by the examiners of the Bureau of Customs in the presence of
officials of the Manila Police Department, an assistant city fiscal and a representative of
Remedios Mago. Under date of 15 November 1966, Mago filed an amended petition, including as
party defendants Collector of Customs Pedro Pacis of the Port of Manila and Lt. Martin Alagao of
the Manila Police Department. At the hearing on 9 December 1966, the lower court, with the
conformity of the parties, ordered that an inventory of the goods be made by its clerk of court in
the presence of the representatives of the claimant of the goods, the Bureau of Customs, and the
Anti- Smuggling Center of the Manila Police Department. On 23 December 1966, Mago filed an
ex parte motion to release the goods, alleging that since the inventory of the goods seized did
not show any article of prohibited importation, the same should be released as per agreement of
the parties upon her posting of the appropriate bond that may be determined by the court. On 7
March 1967, the Judge issued an order releasing the goods to Mago upon her filing of a bond in
the amount of P40,000.00. On 13 March 1967, Papa, on his own behalf, filed a motion for
reconsideration of the order of the court releasing the goods under bond, upon the ground that
the Manila Police Department had been directed by the Collector of Customs of the Port of Manila
to hold the goods pending termination of the seizure proceedings. Without waiting for the court’s
action on the motion for reconsideration, and alleging that they had no plain, speedy and
adequate remedy in the ordinary course of law, Papa, et. al. filed the action for prohibition and
certiorari with preliminary injunction before the Supreme Court.

Held: The Chief of the Manila Police Department, Ricardo G. Papa, having been deputized in
writing by the Commissioner of Customs, could, for the purposes of the enforcement of the
customs and tariff laws, effect searches, seizures, and arrests, and it was his duty to make
seizure, among others, of any cargo, articles or other movable property when the same may be
subject to forfeiture or liable for any fine imposed under customs and tariff laws. He could
lawfully open and examine any box, trunk, envelope or other container wherever found when he
had reasonable cause to suspect the presence therein of dutiable articles introduced into the
Philippines contrary to law; and likewise to stop, search and examine any vehicle, beast or
person reasonably suspected of holding or conveying such article as aforesaid. It cannot be
doubted, therefore, that Papa, Chief of Police of Manila, could lawfully effect the search and
seizure of the goods in question. The Tariff and Customs Code authorizes him to demand
assistance of any police officer to effect said search and seizure, and the latter has the legal duty
to render said assistance. This was what happened precisely in the case of Lt. Martin Alagao
who, with his unit, made the search and seizure of the two trucks loaded with the nine bales of
goods in question at the Agrifina Circle. He was given authority by the Chief of Police to make the
interception of the cargo. Martin Alagao and his companion policemen had authority to effect the
seizure without any search warrant issued by a competent court. The Tariff and Customs Code
does not require said warrant herein. The Code authorizes persons having police authority under
Section 2203 of the Tariff and Customs Code to enter, pass through or search any land,
inclosure, warehouse, store or building, not being a dwelling house; and also to inspect, search
and examine any vessel or aircraft and any trunk, package, box or envelope or any person on
board, or stop and search and examine any vehicle, beast or person suspected of holding or
conveying any dutiable or prohibited article introduced into the Philippines contrary to law,
without mentioning the need of a search warrant in said cases. But in the search of a dwelling
house, the Code provides that said “dwelling house may be entered and searched only upon
warrant issued by a judge or justice of the peace.” Except in the case of the search of a dwelling
house, persons exercising police authority under the customs law may effect search and seizure
without a search warrant in the enforcement of customs laws. Herein, Martin Alagao and his
companion policemen did not have to make any search before they seized the two trucks and
their cargo. But even if there was a search, there is still authority to the effect that no search
warrant would be needed under the circumstances obtaining herein. The guaranty of freedom
from unreasonable searches and seizures is construed as recognizing a necessary difference
between a search of a dwelling house or other structure in respect of which a search warrant
may readily be obtained and a search of a ship, motorboat, wagon, or automobile for contraband
goods, where it is not practicable to secure a warrant, because the vehicle can be quickly moved
out of the locality or jurisdiction in which the warrant must be sought. Having declared that the
seizure by the members of the Manila Police Department of the goods in question was in
accordance with law and by that seizure the Bureau of Customs had acquired jurisdiction over
the goods for the purposes of the enforcement of the customs and tariff laws, to the exclusion of
the Court of First Instance of Manila.

54. Whren v. United States [ 517 US 806 (No. 95-5841), 10 June 1996]
Scalia (J)

Facts: On the evening of 10 June 1993, plainclothes vice-squad officers of the District of
Columbia Metropolitan Police Department were patrolling a “high drug area” of the city in an
unmarked car. Their suspicions were aroused when they passed a dark Pathfinder truck with
temporary license plates and youthful occupants waiting at a stop sign, the driver looking down
into the lap of the passenger at his right. The truck remained stopped at the intersection for what
seemed an unusually long time–more than 20 seconds. When the police car executed a U-turn in
order to head back toward the truck, the Pathfinder turned suddenly to its right, without
signalling, and sped off at an “unreasonable” speed. The policemen followed, and in a short
while overtook the Pathfinder when it stopped behind other traffic at a red light. They pulled up
alongside, and Officer Ephraim Soto stepped out and approached the driver’s door, identifying
himself as a police officer and directing the driver, James L. Brown, to put the vehicle in park.
When Soto drew up to the driver’s window, he immediately observed two large plastic bags of
what appeared to be crack cocaine in Michael A. Whren’s hands. Whren and Brown were
arrested, and quantities of several types of illegal drugs were retrieved from the vehicle. They
were charged in a four-count indictment with violating various federal drug laws, including 21 U.
S. C. Section(s) 844(a) and 860(a). At a pretrial suppression hearing, they challenged the legality
of the stop and the resulting seizure of the drugs. They argued that the stop had not been
justified by probable cause to believe, or even reasonable suspicion, that they were engaged in
illegal drug-dealing activity; and that Officer Soto’s asserted ground for approaching the vehicle–
to give the driver a warning concerning traffic violations–was pretextual. The District Court
denied the suppression motion, concluding that “the facts of the stop were not controverted,”
and “[t]here was nothing to really demonstrate that the actions of the officers were contrary to a
normal traffic stop.” Whren and Brown were convicted of the counts at issue here. The Court of
Appeals affirmed the convictions, holding with respect to the suppression issue that, “regardless
of whether a police officer subjectively believes that the occupants of an automobile may be
engaging in some other illegal behavior, a traffic stop is permissible as long as a reasonable
officer in the same circumstances could have stopped the car for the suspected traffic violation.”

Issue: Whether the seizure involving possession of drugs valid, when the vehicle was stopped
due to a violation of the traffic code.

Held: The Fourth Amendment guarantees “[t]he right of the people to be secure in their
persons, houses, papers, and effects, against unreasonable searches and seizures.” Temporary
detention of individuals during the stop of an automobile by the police, even if only for a brief
period and for a limited purpose, constitutes a “seizure” of “persons” within the meaning of this
provision. An automobile stop is thus subject to the constitutional imperative that it not be
“unreasonable” under the circumstances. As a general matter, the decision to stop an
automobile is reasonable where the police have probable cause to believe that a traffic violation
has occurred. It is of course true that in principle every Fourth Amendment case, since it turns
upon a “reasonableness” determination, involves a balancing of all relevant factors. With rare
exceptions not applicable here, however, the result of that balancing is not in doubt where the
search or seizure is based upon probable cause. analysis involved searches or seizures
conducted in an extraordinary manner, unusually harmful to an individual’s privacy or even
physical interests–such as, for example, seizure by means of deadly force, unannounced entry
into a home, entry into a home without a warrant, or physical penetration of the body. The
making of a traffic stop out-of-uniform does not remotely qualify as such an extreme practice,
and so is governed by the usual rule that probable cause to believe the law has been broken
“outbalances” private interest in avoiding police contact. Herein, the officers had probable cause
to believe that Whren and Brown had violated the traffic code. That rendered the stop
reasonable under the Fourth Amendment, the evidence thereby discovered admissible.

55. People vs. de Gracia [GR 102009-10, 6 July 1994]


Second Division, Regalado (J): 5 concur

Facts: The incidents took place at the height of the coup d’etat staged in December, 1989 by
ultra-rightist elements headed by the Reform the Armed Forces Movement-Soldiers of the Filipino
People (RAM-SFP) against the Government. At that time, various government establishments and
military camps in Metro Manila were being bombarded by the rightist group with their “tora-tora”
planes. At around midnight of 30 November 1989, the 4th Marine Battalion of the Philippine
Marines occupied Villamor Air Base, while the Scout Rangers took over the Headquarters of the
Philippine Army, the Army Operations Center, and Channel 4, the government television station.
Also, some elements of the Philippine Army coming from Fort Magsaysay occupied the Greenhills
Shopping Center in San Juan, Metro Manila. On 1 December 1989, Maj. Efren Soria of the
Intelligence Division, National Capital Region Defense Command, was on board a brown Toyota
car conducting a surveillance of the Eurocar Sales Office located at Epifanio de los Santos
Avenue (EDSA) in Quezon City, together with his team composed of Sgt. Crispin Sagario, M/Sgt.
Ramon Briones, S/Sgt. Henry Aquino, one S/Sgt. Simon and a Sgt. Ramos. The surveillance, which
actually started on the night of 30 November 1989 at around 10:00 p.m., was conducted
pursuant to an intelligence report received by the division that said establishment was being
occupied by elements of the RAM-SFP as a communication command post. Sgt. Crispin Sagario,
the driver of the car, parked the vehicle around 10 to 15 meters away from the Eurocar building
near P. Tuazon Street, S/Sgt. Henry Aquino had earlier alighted from the car to conduct his
surveillance on foot. A crowd was then gathered near the Eurocar office watching the on-going
bombardment near Camp Aguinaldo. After a while a group of 5 men disengaged themselves from
the crowd and walked towards the car of the surveillance team. At that moment, Maj. Soria, who
was then seated in front, saw the approaching group and immediately ordered Sgt. Sagario to
start the car and leave the area. As they passed by the group, then only 6 meters away, the
latter pointed to them, drew their guns and fired at the team, which attack resulted in the
wounding of Sgt. Sagario on the right thigh. Nobody in the surveillance team was able to
retaliate because they sought cover inside the car and they were afraid that civilians or
bystanders might be caught in the cross-fire. As a consequence, at around 6:30 a.m. of 5
December 1989, searching them composed of F/Lt. Virgilio Babao as team leader, M/Sgt. Lacdao,
Sgt. Magallion, Sgt. Patricio Pacatang, and elements of the 16th Infantry Battalion under one Col.
delos Santos raided the Eurocar Sales Office. They were able to find and confiscate 6 cartons of
M-16 ammunition, five bundles of C-4 dynamites, M-shells of different calibers, and “molotov”
bombs inside one of the rooms belonging to a certain Col. Matillano which is located at the right
portion of the building. St. Oscar Obenia, the first one to enter the Eurocar building, saw Rolando
De Gracia inside the office of Col. Matillano, holding a C-4 and suspiciously peeping through a
door. De Gracia was the only person then present inside the room. A uniform with the nametag
of Col. Matillano was also found. As a result of the raid, the team arrested de Gracia, as well as
Soprieso Verbo and Roberto Jimena who were janitors at the Eurocar building. They were then
made to sign an inventory, written in Tagalog, of the explosives and ammunition confiscated by
the raiding team. No search warrant was secured by the raiding team because, according to
them, at that time there was so much disorder considering that the nearby Camp Aguinaldo was
being mopped up by the rebel forces and there was simultaneous firing within the vicinity of the
Eurocar office, aside from the fact that the courts were consequently closed. The group was able
to confirm later that the owner of Eurocar office is a certain Mr. Gutierrez and that de Gracia is
supposedly a “boy” therein. de Gracia was charged in two separate informations for illegal
possession of ammunition and explosives in furtherance of rebellion, and for attempted homicide
(Criminal Cases Q-90-11755 and Q-90-11756, respectively), which were tried jointly by the
Regional Trial Court of Quezon City, Branch 103. During the arraignment, de Gracia pleaded not
guilty to both charges. However, he admitted that he is not authorized to posses any firearms,
ammunition and/or explosive. The parties likewise stipulated that there was a rebellion during
the period from November 30 up to 9 December 1989. On 22 February 1991, the trial court
rendered judgment acquitting de Gracia of attempted homicide, but found him guilty beyond
reasonable doubt of the offense of illegal possession of firearms in furtherance of rebellion and
sentenced him to serve the penalty of reclusion perpetua. De Gracia appealed.

Issue: Whether the military operatives made a valid search and seizure during the height of the
December 1989 coup d’etat.

Held: It is admitted that the military operatives who raided the Eurocar Sales Office were not
armed with a search warrant at that time. The raid was actually precipitated by intelligence
reports that said office was being used as headquarters by the RAM. Prior to the raid, there was a
surveillance conducted on the premises wherein the surveillance team was fired at by a group of
men coming from the Eurocar building. When the military operatives raided the place, the
occupants thereof refused to open the door despite the requests for them to do so, thereby
compelling the former to break into the office. The Eurocar Sales Office is obviously not a gun
store and it is definitely not an armory or arsenal which are the usual depositories for explosives
and ammunition. It is primarily and solely engaged in the sale of automobiles. The presence of
an unusual quantity of high-powered firearms and explosives could not be justifiably or even
colorably explained. In addition, there was general chaos and disorder at that time because of
simultaneous and intense firing within the vicinity of the office and in the nearby Camp
Aguinaldo which was under attack by rebel forces. The courts in the surrounding areas were
obviously closed and, for that matter, the building and houses therein were deserted. Under the
foregoing circumstances, the case falls under one of the exceptions to the prohibition against a
warrantless search. In the first place, the military operatives, taking into account the facts
obtaining in this case, had reasonable ground to believe that a crime was being committed.
There was consequently more than sufficient probable cause to warrant their action.
Furthermore, under the situation then prevailing, the raiding team had no opportunity to apply
for and secure a search warrant from the courts. The trial judge himself manifested that on 5
December 1989 when the raid was conducted, his court was closed. Under such urgency and
exigency of the moment, a search warrant could lawfully be dispensed with.

56. Valmonte vs. de Villa [GR 83988, 24 May 1990]


En Banc, Padilla (J): 10 concur, 1 on leave

Facts: On 20 January 1987, the National Capital Region District Command (NCRDC) was
activated pursuant to Letter of Instruction 02/87 of the Philippine General Headquarters, AFP,
with the mission of conducting security operations within its area of responsibility and peripheral
areas, for the purpose of establishing an effective territorial defense, maintaining peace and
order, and providing an atmosphere conducive to the social, economic and political development
of the National Capital Region. 1 As part of its duty to maintain peace and order, the NCRDC
installed checkpoints in various parts of Valenzuela, Metro Manila. Ricardo C. Valmonte and the
Union of Lawyers and Advocates for People’s Right (ULAP) filed a petition for prohibition with
preliminary injunction and/or temporary restraining order witht the Supreme Court, seeking the
declaration of checkpoints in Valenzuela, Metro Manila or elsewhere, as unconstitutional and the
dismantling and banning of the same or, in the alternative, to direct the respondents to
formulate guidelines in the implementation of checkpoints, for the protection of the people. They
aver that, because of the installation of said checkpoints, the residents of Valenzuela are worried
of being harassed and of their safety being placed at the arbitrary, capricious and whimsical
disposition of the military manning the checkpoints, considering that their cars and vehicles are
being subjected to regular searches and check-ups, especially at night or at dawn, without the
benefit of a search warrant and/or court order. Their alleged fear for their safety increased when,
at dawn of 9 July 1988, Benjamin Parpon, a supply officer of the Municipality of Valenzuela,
Bulacan, was gunned down allegedly in cold blood by the members of the NCRDC manning the
checkpoint along McArthur Highway at Malinta, Valenzuela, for ignoring and/or refusing to submit
himself to the checkpoint and for continuing to speed off inspire of warning shots fired in the air.
Valmonte also claims that, on several occasions, he had gone thru these checkpoints where he
was stopped and his car subjected to search/check-up without a court order or search warrant.
They further contend that the said checkpoints give Gen. Renato de Villa and the National Capital
Region District Command a blanket authority to make searches and/or seizures without search
warrant or court order in violation of the Constitution. In the Supreme Court’s decision dated 29
September 1989, Valmonte’s and ULAP’s petition for prohibition, seeking the declaration of the
checkpoints as unconstitutional and their dismantling and/or banning, was dismissed. Valmonte
and ULAP filed the motion and supplemental motion for reconsideration of said decision.
Issue: Whether checkpoints serve as a blanket authority for government officials for warrantless
search and seizure and, thus, are violative of the Constitution.

Held: Nowhere in the Supreme Court’s decision of 24 May 1990 did the Court legalize all
checkpoints, i.e. at all times and under all circumstances. What the Court declared is, that
checkpoints are not illegal per se. Thus, under exceptional circumstances, as where the survival
of organized government is on the balance, or where the lives and safety of the people are in
grave peril, checkpoints may be allowed and installed by the government. Implicit in this
proposition is, that when the situation clears and such grave perils are removed, checkpoints will
have absolutely no reason to remain. Recent and on-going events have pointed to the continuing
validity and need for checkpoints manned by either military or police forces. Although no one can
be compelled, under our libertarian system, to share with the present government its ideological
beliefs and practices, or commend its political, social and economic policies or performance; one
must concede to it the basic right to defend itself from its enemies and, while in power, to pursue
its program of government intended for public welfare; and in the pursuit of those objectives, the
government has the equal right, under its police power, to select the reasonable means and
methods for best achieving them. The checkpoint is evidently one of such means it has selected.
Admittedly, the routine checkpoint stop does intrude, to a certain extent, on motorist’s right to
“free passage without interruption”, but it cannot be denied that, as a rule, it involves only a
brief detention of travellers during which the vehicle’s occupants are required to answer a brief
question or two. For as long as the vehicle is neither searched nor its occupants subjected to a
body search, and the inspection of the vehicle is limited to a visual search, said routine checks
cannot be regarded as violative of an individual’s right against unreasonable search. These
routine checks, when conducted in a fixed area, are even less intrusive. Further, vehicles are
generally allowed to pass these checkpoints after a routine inspection and a few questions. If
vehicles are stopped and extensively searched, it is because of some probable cause which
justifies a reasonable belief of the men at the checkpoints that either the motorist is a law-
offender or the contents of the vehicle are or have been instruments of some offense. By the
same token, a warrantless search of incoming and outgoing passengers, at the arrival and
departure areas of an international airport, is a practice not constitutionally objectionable
because it is founded on public interest, safety, and necessity. Lastly, the Court’s decision on
checkpoints does not, in any way, validate nor condone abuses committed by the military
manning the checkpoints. The Court’s decision was concerned with power, i.e. whether the
government employing the military has the power to install said checkpoints. Once that power is
acknowledged, the Court’s inquiry ceases. True, power implies the possibility of its abuse. But
whether there is abuse in a particular situation is a different “ball game” to be resolved in the
constitutional arena. In any situation, where abuse marks the operation of a checkpoint, the
citizen is not helpless. For the military is not above but subject to the law. And the courts exist to
see that the law is supreme. Soldiers, including those who man checkpoints, who abuse their
authority act beyond the scope of their authority and are, therefore, liable criminally and civilly
for their abusive acts.

57. Camara vs. Municipal Court of the City and Country of San Francisco [387 US 523,
5 June 1967]
White (J)

Facts: On 6 November 1963, an inspector of the Division of Housing Inspection of the San
Francisco Department of Public Health entered an apartment building to make a routine annual
inspection for possible violations of the city’s Housing Code. The building’s manager informed
the inspector that Camara, lessee of the ground floor, was using the rear of his leasehold as a
personal residence. Claiming that the building’s occupancy permit did not allow residential use of
the ground floor, the inspector confronted Camara and demanded that he permit an inspection of
the premises. Camara refused to allow the inspection because the inspector lacked a search
warrant. The inspector returned on November 8, again without a warrant, and Camara again
refused to allow an inspection. A citation was then mailed ordering Camara to appear at the
district attorney’s office. When Camara failed to appear, two inspectors returned to his
apartment on November 22. They informed Camara that he was required by law to permit an
inspection under 503 of the Housing Code. Camara nevertheless refused the inspectors access to
his apartment without a search warrant. Thereafter, a complaint was filed charging him with
refusing to permit a lawful inspection in violation of 507 of the Code. Camara was arrested on
December 2nd released on bail. When his demurrer to the criminal complaint was denied,
Camara filed the petition for a writ of prohibition in a California Superior Court alleging that he
was awaiting trial on a criminal charge of violating the San Francisco Housing Code by refusing
to permit a warrantless inspection of his residence, and that a writ of prohibition should issue to
the criminal court because the ordinance authorizing such inspections is unconstitutional on its
face. The Superior Court denied the writ, the District Court of Appeal affirmed, and the Supreme
Court of California denied a petition for hearing.

Issue: Whether Camara can validly refuse the inspection of his dwelling by the Division of
Housing Inspection.

Held: The Fourth Amendment bars prosecution of a person who has refused to permit a
warrantless code-enforcement inspection of his personal residence. The basic purpose of the
Fourth Amendment, which is enforceable against the States through the Fourteenth, through its
prohibition of “unreasonable” searches and seizures is to safeguard the privacy and security of
individuals against arbitrary invasions by governmental officials. With certain carefully defined
exceptions, an unconsented warrantless search of private property is “unreasonable.”
Administrative searches of the kind at issue here are significant intrusions upon the interests
protected by the Fourth Amendment, that such searches when authorized and conducted without
a warrant procedure lack the traditional safeguards which the Fourth Amendment guarantees to
the individual, and that the reasons put forth in Frank v. Maryland and in other cases for
upholding these warrantless searches are insufficient to justify so substantial a weakening of the
Fourth Amendment’s protections. Contrary to the assumption of Frank v. Maryland, Fourth
Amendment interests are not merely “peripheral” where municipal fire, health, and housing
inspection programs are involved whose purpose is to determine the existence of physical
conditions not complying with local ordinances. Those programs, moreover, are enforceable by
criminal process, as is refusal to allow an inspection. Warrantless administrative searches cannot
be justified on the grounds that they make minimal demands on occupants; that warrants in
such cases are unfeasible; or that area inspection programs could not function under reasonable
search-warrant requirements. Probable cause upon the basis of which warrants are to be issued
for area code-enforcement inspections is not dependent on the inspector’s belief that a particular
dwelling violates the code but on the reasonableness of the enforcement agency’s appraisal of
conditions in the area as a whole. The standards to guide the magistrate in the issuance of such
search warrants will necessarily vary with the municipal program being enforced. Nothing here is
intended to foreclose prompt inspections, even without a warrant, that the law has traditionally
upheld in emergency situations. On the other hand, in the case of most routine area inspections,
there is no compelling urgency to inspect at a particular time or on a particular day. Moreover,
most citizens allow inspections of their property without a warrant. Thus, as a practical matter
and in light of the Fourth Amendment’s requirement that a warrant specify the property to be
searched, it seems likely that warrants should normally be sought only after entry is refused
unless there has been a citizen complaint or there is other satisfactory reason for securing
immediate entry. Similarly, the requirement of a warrant procedure does not suggest any change
in what seems to be the prevailing local policy, in most situations, of authorizing entry, but not
entry by force, to inspect. Herein, Camara has been charged with a crime for his refusal to
permit housing inspectors to enter his leasehold without a warrant. There was no emergency
demanding immediate access; in fact, the inspectors made three trips to the building in an
attempt to obtain Camara’s consent to search. Yet no warrant was obtained and thus appellant
was unable to verify either the need for or the appropriate limits of the inspection. No doubt, the
inspectors entered the public portion of the building with the consent of the landlord, through the
building’s manager, but the City/County does not contend that such consent was sufficient to
authorize inspection of Camara’s premises. Assuming the facts to be as the parties have alleged,
camara had a constitutional right to insist that the inspectors obtain a warrant to search and that
appellant may not constitutionally be convicted for refusing to consent to the inspection. It
appears from the opinion of the District Court of Appeal that under these circumstances a writ of
prohibition will issue to the criminal court under California law.

58. In RE: Umil, Umil vs. Ramos [GR 81567, 9 July 1990]; also Roque vs. de Villa [GR
84581-82], In RE: Anonuevo. Anonuevo vs. Ramos [GR 84583-84], In RE: Ocaya. Ocaya
vs. Aguirre [GR 83162], In RE: Espiritu. Espiritu vs. Lim [GR 85727], and In RE:
Nazareno. Nazareno vs. Station Commander of Muntinlupa Police Station [GR 86332]
En Banc, Per Curiam: 11 concur

Facts: [GR 81567] On 1 February 1988, the Regional Intelligence Operations Unit of the Capital
Command (RIOU-CAPCOM) received confidential information about a member of the NPA
Sparrow Unit (liquidation squad) being treated for a gunshot wound at the St. Agnes Hospital in
Roosevelt Avenue, Quezon City. Upon verification, it was found that the wounded person, who
was listed in the hospital records as Ronnie Javelon, is actually Rolando Dural, a member of the
NPA liquidation squad, responsible for the killing of 2 CAPCOM soldiers the day before, or on 31
January 1988, in Macanining Street, Bagong Barrio, Caloocan City. In view of this verification,
Dural was transferred to the Regional Medical Services of the CAPCOM, for security reasons.
While confined thereat, or on 4 February 1988, Dural was positively identified by eyewitnesses as
the gunman who went on top of the hood of the CAPCOM mobile patrol car, and fired at the 2
CAPCOM soldiers seated inside the car identified as T/Sgt. Carlos Pabon and CIC Renato Manligot.
As a consequence of this positive identification, Dural was referred to the Caloocan City Fiscal
who conducted an inquest and thereafter filed with the Regional Trial Court of Caloocan City an
information charging Rolando Dural alias Ronnie Javelon with the crime of “Double Murder with
Assault Upon Agents of Persons in Authority.” (Criminal Case C-30112; no bail recommended).
On 15 February 1988, the information was amended to include, as defendant, Bernardo Itucal, Jr.
who, at the filing of the original information, was still unidentified. Meanwhile, on 6 February
1988, a petition for habeas corpus was filed with the Supreme Court on behalf of Roberto Umil,
Rolando Dural, and Renato Villanueva. The Court issued the writ of habeas corpus on 9 February
1988 and Fidel V. Ramos, Maj. Gen. Renato de Villa, Brig. Gen. Ramon Montano, and Brig. Gen.
Alexander Aguirre filed a Return of the Writ on 12 February 1988. Thereafter, the parties were
heard on 15 February 1988. On 26 February 1988, however, Umil and Villanueva posted bail
before the Regional Trial Court of Pasay City where charges for violation of the Anti-Subversion
Act had been filed against them, and they were accordingly released.

Issue: Whether Dural can be validly arrested without any warrant of arrest for the crime of
rebellion.

Held: Dural, it clearly appears that he was not arrested while in the act of shooting the 2
CAPCOM soldiers nor was he arrested just after the commission of the said offense for his arrest
came a day after the said shooting incident. Seemingly, his arrest without warrant is unjustified.
However, Dural was arrested for being a member of the New Peoples Army (NPA), an outlawed
subversive organization. Subversion being a continuing offense, the arrest of Rolando Dural
without warrant is justified as it can be said that he was committing an offense when arrested.
The crimes of rebellion, subversion, conspiracy or proposal to commit such crimes, and crimes or
offenses committed in furtherance thereof or in connection therewith constitute direct assaults
against the State and are in the nature of continuing crimes. The arrest of persons involved in
the rebellion whether as its fighting armed elements, or for committing non-violent acts but in
furtherance of the rebellion, is more an act of capturing them in the course of an armed conflict,
to quell the rebellion, than for the purpose of immediately prosecuting them in court for a
statutory offense. The arrest, therefore, need not follow the usual procedure in the prosecution
of offenses which requires the determination by a judge of the existence of probable cause
before the issuance of a judicial warrant of arrest and the granting of bail if the offense is
bailable. Obviously, the absence of a judicial warrant is no legal impediment to arresting or
capturing persons committing overt acts of violence against government forces, or any other
milder acts but equally in pursuance of the rebellious movement. The arrest or capture is thus
impelled by the exigencies of the situation that involves the very survival of society and its
government and duly constituted authorities.

59. People vs. Sucro [GR 93239, 18 March 1991]


Third Division, Gutierrez Jr. (J): 4 concur

Facts: On 21 March 1989, Pat. Roy Fulgencio, a member of the INP, Kalibo, Aklan, was instructed
by P/Lt. Vicente Seraspi, Jr. (Station Commander of the INP Kalibo, Aklan) to monitor the activities
of Edison Sucro, because of information gathered by Seraspi that Sucro was selling marijuana. As
planned, at about 5:00 P.M. on said date, Pat. Fulgencio positioned himself under the house of a
certain Arlie Regalado at C. Quimpo Street. Adjacent to the house of Regalado, about 2 meters
away, was a chapel. Thereafter, Pat. Fulgencio saw Sucro enter the chapel, taking something
which turned out later to be marijuana from the compartment of a cart found inside the chapel,
and then return to the street where he handed the same to a buyer, Aldie Borromeo. After a
while Sucro went back to the chapel and again came out with marijuana which he gave to a
group of persons. It was at this instance that Pat. Fulgencio radioed P/Lt. Seraspi and reported
the activity going on P/Lt. Seraspi instructed Pat. Fulgencio to continue monitoring
developments. At about 6:30 P.M., Pat. Fulgencio again called up Seraspi to report that a third
buyer later identified as Ronnie Macabante, was transacting with Sucro. At that point, the team
of P/Lt Seraspi proceeded to the area and while the police officers were at the Youth Hostel at
Maagma St., Pat. Fulgencio told P/Lt. Seraspi to intercept Macabante and Sucro. P/ Lt. Seraspi
and his team caught up with Macabante at the crossing of Mabini and Maagma Sts. in front of the
Aklan Medical Center. Upon seeing the police, Macabante threw something to the ground which
turned out to be a tea bag of marijuana. When confronted, Macabante readily admitted that he
bought the same from Sucro in front of the chapel. The police team was able to overtake and
arrest Sucro at the corner of C. Quimpo and Veterans Sts. The police recovered 19 sticks and 4
teabags of marijuana from the cart inside the chapel and another teabag from Macabante. The
teabags of marijuana were sent to the PC-INP Crime Laboratory Service, at Camp Delgado, Iloilo
City for analysis. The specimens were all found positive of marijuana. Sucro was charged with
violation of Section 4, Article II of the Dangerous Drugs Act. Upon arraignment, Sucro, assisted by
counsel, entered a plea of “not guilty” to the offense charged. Trial ensued and a judgment of
conviction was rendered, finding Sucro guilty of the sale of prohibited drug and sentencing him
to suffer the penalty of life imprisonment, and pay a fine of P20,000, and costs. Sucro appealed.

Issue: Whether the arrest without warrant of the accused is lawful and consequently, whether
the evidence resulting from such arrest is admissible.

Held: Section 5, Rule 113 of the Rules on Criminal Procedure provides for the instances where
arrest without warrant is considered lawful. The rule states that “A peace officer or private
person may, without warrant, arrest a person: (a) When in his presence, the person to be
arrested has committed, is actually committing, or is attempting to commit an offense; (b) When
an offense has in fact just been committed, and he has personal knowledge of facts indicating
that the person to be arrested has committed it;” An offense is committed in the presence or
within the view of an officer, within the meaning of the rule authorizing an arrest without a
warrant, when the officer sees the offense, although at a distance, or hears the disturbances
created thereby and proceeds at once to the scene thereof. The failure of the police officers to
secure a warrant stems from the fact that their knowledge acquired from the surveillance was
insufficient to fulfill the requirements for the issuance of a search warrant. What is paramount is
that probable cause existed. Still, that searches and seizures must be supported by a valid
warrant is not an absolute rule. Among the exceptions granted by law is a search incidental to a
lawful arrest under Sec. 12, Rule 126 of the Rules on Criminal Procedure, which provides that a
person lawfully arrested may be searched for dangerous weapons or anything which may be
used as proof of the commission of an offense, without a search warrant. Herein, police officers
have personal knowledge of the actual commission of the crime when it had earlier conducted
surveillance activities of the accused. Under the circumstances (monitoring of transactions) there
existed probable cause for the arresting officers, to arrest Sucro who was in fact selling
marijuana and to seize the contraband. Thus, as there is nothing unlawful about the arrest
considering its compliance with the requirements of a warrantless arrest; ergo, the fruits
obtained from such lawful arrest are admissible in evidence.

60. People vs. Gerente [GR 95847-48, 10 March 1993]


First Division, Grino-Aquino (J): 3 concur

Facts: At about 7:00 a.m. of 30 April 1990, Gabriel Gerente, together with Fredo Echigoren and
Totoy Echigoren, allegedly started drinking liquor and smoking marijuana in Gerente’s house
which is about 6 meters away from the house of Edna Edwina Reyes who was in her house on
that day. She overheard the three men talking about their intention to kill Clarito Blace. She
testified that she heard Fredo Echigoren saying, “Gabriel, papatayin natin si Clarito Blace.” Fredo
and Totoy Echigoren and Gerente carried out their plan to kill Clarito Blace at about 2:00 p.m. of
the same day. Reyes allegedly witnessed the killing. Fredo Echigoren struck the first blow against
Clarito Blace, followed by Totoy Echigoren and Gabriel Gerente who hit him twice with a piece of
wood in the head and when he fell, Totoy Echigoren dropped a hollow block on the victim’s head.
Thereafter, the three men dragged Blace to a place behind the house of Gerente. At about 4:00
p.m. of the same day, Patrolman Jaime Urrutia of the Valenzuela Police Station received a report
from the Palo Police Detachment about a mauling incident. He went to the Valenzuela District
Hospital where the victim was brought. He was informed by the hospital officials that the victim
died on arrival. The cause of death was massive fracture of the skull caused by a hard and heavy
object. Right away, Patrolman Urrutia, together with Police Corporal Romeo Lima and Patrolman
Alex Umali, proceeded to Paseo de Blas where the mauling incident took place. There they found
a piece of wood with blood stains, a hollow block and two roaches of marijuana. They were
informed by Reyes that she saw the killing and she pointed to Gabriel Gerente as one of the
three men who killed Clarito. The policemen proceeded to the house of Gerente, who was then
sleeping. They told him to come out of the house and they introduced themselves as policemen.
Patrolman Urrutia frisked Gerente and found a coin purse in his pocket which contained dried
leaves wrapped in cigarette foil. The dried leaves were sent to the National Bureau of
Investigation for examination. The Forensic Chemist found them to be marijuana. Only Gerente
was apprehended by the police. The other suspects, Fredo and Totoy Echigoren, are still at large.
On 2 May 1990, two separate informations were filed by Assistant Provincial Prosecutor Benjamin
Caraig against him for Violation of Section 8, Art. II, of RA 6425, and for Murder. When arraigned
on 16 May 1990, Gerente pleaded not guilty to both charges. A joint trial of the two cases was
held. On 24 September 1990, the Regional Trial Court of Valenzuela, Metro Manila, Branch 172,
found Gerente guilty of Violation of Section 8 of Republic Act 6425 and sentenced him to suffer
the penalty of imprisonment for a term of 12 years and 1 day, as minimum, to 20 years, as
maximum; and also found him guilty of Murder for which crime he was sentenced to suffer the
penalty of reclusion perpetua. . Gerente appealed.

Issue: Whether the police officers have the personal knowledge of the killing of Blace to allow
them to arrest, and the subsequent searchly Gerente’s person, without the necessary warrant.

Held: The search of Gerente’s person and the seizure of the marijuana leaves in his possession
were valid because they were incident to a lawful warrantless arrest. Paragraphs (a) and (b),
Section 5, Rule 113 of the Revised Rules of Court provide that “A peace officer or a private
person may, without a warrant, arrest a person: (a) When, in his presence, the person to be
arrested has committed, is actually committing, or is attempting to commit an offense; (b) When
an offense has in fact just been committed, and he has personal knowledge of facts indicating
that the person to be arrested has committed it;” The policemen arrested Gerente only some 3
hours after Gerente and his companions had killed Blace. They saw Blace dead in the hospital
and when they inspected the scene of the crime, they found the instruments of death: a piece of
wood and a concrete hollow block which the killers had used to bludgeon him to death. The eye-
witness, Edna Edwina Reyes, reported the happening to the policemen and pinpointed her
neighbor, Gerente, as one of the killers. Under those circumstances, since the policemen had
personal knowledge of the violent death of Blace and of facts indicating that Gerente and two
others had killed him, they could lawfully arrest Gerente without a warrant. If they had
postponed his arrest until they could obtain a warrant, he would have fled the law as his two
companions did. The search conducted on Gerente’s person was likewise lawful because it was
made as an incident to a valid arrest. This is in accordance with Section 12, Rule 126 of the
Revised Rules of Court which provides that “A person lawfully arrested may be searched for
dangerous weapons or anything which may be used as proof of the commission of an offense,
without a search warrant.” The frisk and search of Gerente’s person upon his arrest was a
permissible precautionary measure of arresting officers to protect themselves, for the person
who is about to be arrested may be armed and might attack them unless he is first disarmed.

61. People vs. Rodrigueza [GR 95902, 4 February 1992]


Second Division, Regalado (J): 4 concur

Facts: [Prosecution] At around 5:00 p.m. of 1 July 1987, CIC Ciriaco Taduran was in their
headquarters at the Office of the Narcotics Regional Unit at Camp Bagong Ibalon, Legaspi City,
together with S/Sgt. Elpidio Molinawe, CIC Leonardo B. Galutan and their commanding officer,
Major Crisostomo M. Zeidem, when a confidential informer arrived and told them that there was
an ongoing illegal traffic of prohibited drugs in Tagas, Daraga, Albay. Major Zeidem formed a
team to conduct a buybust operation, which team was given P200.00 in different denominations
to buy marijuana. These bills were treated with ultraviolet powder at the Philippine Constabulary
Crime Laboratory (PCCL). Sgt. Molinawe gave the money to Taduran who acted as the poseur
buyer. He was told to look for a certain Don, the alleged seller of prohibited drugs. Taduran went
to Tagas alone and, while along the road, he met Samuel Segovia. He asked Segovia where he
could find Don and where he could buy marijuana. Segovia left for a while and when he returned,
he was accompanied by a man who was later on introduced to him as Don Rodrigueza. After
agreeing on the price of P200.00 for 100 grams of marijuana, Don halted a passing tricycle
driven by Antonio Lonceras. He boarded it and left Taduran and Segovia. When he came back,
Don gave Taduran “a certain object wrapped in a plastic” which was later identified as
marijuana, and received payment therefor. Thereafter, Taduran returned to the headquarters
and made a report regarding his said purchase of marijuana. Based on that information, Major
Zeidem ordered a team to conduct an operation to apprehend the suspects. In the evening of the
same date, CIC Galutan and S/Sgt. Molinawe proceeded to Regidor Street, Daraga, Albay and
arrested Rodrigueza, Antonio Lonceras and Samuel Segovia. The constables were not, however,
armed with a warrant of arrest when they apprehended the three accused. The arrests were
brought to the headquarters for investigation. Thereafter, agents of the Narcotics Command
(NARCOM) conducted a raid in the house of Jovencio Rodrigueza, Don’s father. Taduran did not
go with them. During the raid, they were able to confiscate dried marijuana leaves and a plastic
syringe, among others. The search, however, was not authorized by any search warrant. The
next day, Jovencio Rodrigueza was released from detention but Don Rodrigueza was detained.
[Defense] Don Rodrigueza, on the other hand, claimed that on said date he was in the house of
his aunt in San Roque, Legaspi City. He stayed there overnight and did not leave the place until
the next day when his brother arrived and told him that their father was taken by some military
men the preceding night. Rodrigueza went to Camp Bagong Ibalon and arrived there at around
8:00 a.m. of 2 July 1987. When he arrived, he was asked if he knew anything about the
marijuana incident, to which question he answered in the negative. Like Segovia, he was made
to hold a P10.00 bill and was brought to the crime laboratory for examination. From that time on,
he was not allowed to go home and was detained inside the camp. He was also tortured in order
to make him admit his complicity in the alleged sale of marijuana.

On 10 July 1987, Don Rodrigueza, Samuel Segovia and Antonio Lonceras, for possession of 100
grams of marijuana leaves and for selling, in a buy-bust operation, said 100 grams of dried
marijuana leaves for a consideration of P200.00. During the arraignment, all the accused pleaded
not guilty to the charge against them. The Regional Trial Court of Legaspi City, Branch 10, found
Don Rodrigueza guilty beyond reasonable doubt of violating Section 4, Article II of the Dangerous
Drugs Act of 1972 (Republic Act 6425, as amended) and sentenced him to suffer the penalty of
life imprisonment and to pay a fine of P20,000.00 and costs. The court, however, acquitted
Segovia and Lonceres. Rodrigueza appealed.

Issue: Whether the time of Don Rodrigueza’s arrest is material in determining his culpability in
the crime charged.

Held: As provided in the present Constitution, a search, to be valid, must generally be


authorized by a search warrant duly issued by the proper government authority. True, in some
instances, the Court has allowed government authorities to conduct searches and seizures even
without a search warrant. Thus, when the owner of the premises waives his right against such
incursion; when the search is incidental to a lawful arrest; when it is made on vessels and aircraft
for violation of customs laws; when it is made on automobiles for the purpose of preventing
violations of smuggling or immigration laws; when it involves prohibited articles in plain view; or
in cases of inspection of buildings and other premises for the enforcement of fire, sanitary and
building regulations, a search may be validly made even without a search warrant. Herein,
however, the raid conducted by the NARCOM agents in the house of Jovencio Rodrigueza was not
authorized by any search warrant. It does not appear, either, that the situation falls under any of
the aforementioned cases. Hence, Rodrigueza’s right against unreasonable search and seizure
was clearly violated. The NARCOM agents could not have justified their act by invoking the
urgency and necessity of the situation because the testimonies of the prosecution witnesses
reveal that the place had already been put under surveillance for quite some time. Had it been
their intention to conduct the raid, then they should, because they easily could, have first
secured a search warrant during that time. Further, the inconsistencies made by prosecution
witnesses give more credibility to the testimony of Don Rodrigueza. While it is true that
Rodrigueza’s defense amounts to an alibi, and as such is the weakest defense in a criminal
prosecution, there are, nonetheless, some evidentiary aspects pointing to the truth in his
testimony. Firstly, the Joint Affidavit of Arrest corroborates his testimony that he was not among
those who were arrested on the night of 1 July 1987. His co-accused Segovia also testified that
Rodrigueza was not with them when they were apprehended by the NARCOM agents. Hence,
Rodrigueza is acquitted of the crime charged, due to the failure of the prosecution to establish its
cause.

62. People vs. Enrile [GR 74189, 26 May 1993]


First Division, Cruz (J): 3 concur

Facts: At about 6:30 p.m. of 25 October 1985, a buy-bust team composed of Pat. Jaime Flores
and Pat. Wilson Rances of the Quezon City Police Anti-Narcotics Unit was dispatched to entrap
Rogelio Abugatal at Roosevelt Avenue in San Francisco del Monte, Quezon City. The plan was
made on the strength of a tip given by Renato Polines, a police informer, who was himself to
pose as the buyer. On that occasion the policemen saw Polines hand over to Abugatal the
marked money representing payment for the mock transaction. Abugatal left with the money
and returned 10 minutes later with a wrapped object which he gave Polines. The two policemen
then approached Abugatal and placed him under arrest, at the same time confiscating the
wrapped object. Subsequent laboratory examination revealed this to be marijuana with flowering
tops weighing 22 grams. Upon prodding, Abugatal led the policemen to a house at 20 De Vera
Street, also in San Francisco del Monte, Quezon City, where he called out for Antonio Enrile.
Enrile came out and met them at the gate. Abugatal pointed to Enrile as the source of the
marijuana, whereupon the policemen immediately arrested and frisked him. They found in the
right front pocket of his trousers the marked money earlier delivered to Abugatal. At the police
headquarters, Abugatal signed a sworn confession. Enrile refused to make any statement
pending consultation with a lawyer. Antonio Enrile y Villaroman and Rogelio Abugatal y Marquez
were charged for violation of the Dangerous Drug Act by the Regional Trial Court of Quezon City.
The RTC, after trial and on 14 February 1986, found Enrile and Abugatal guilty beyond
reasonable doubt and sentenced them to life imprisonment and a fine of P30,000.00. Both
appealed. Abugatal, however, was killed in an attempted jailbreak and thus the appeal is
dismissed as to him.

Issue: Whether the mark money found in Enrile’s possession, pursuant to a warrantless arrest,
search and seizure, provide for his criminal culpability.

Held: It was Abugatal who was allegedly caught red-handed by the policemen as he sold the
marijuana to Polines. Enrile was not even at the scene of the entrapment at that time. Abugatal
said he did lead the policemen to Enrile’s house where he pointed to Enrile as the source of the
marijuana. Even assuming this to be true, that circumstance alone did not justify Enrile’s
warrantless arrest and search. Under Rule 113, Section 5, of the Rules of Court, a peace officer
or a private person may make a warrantless arrest only under any of the following
circumstances: (a) When, in his presence, the person to be arrested has committed, is actually
committing, or is attempting to commit an offense; (b) When an offense has in fact just been
committed, and he has personal knowledge of facts indicating that the person to be arrested has
committed it; and (c) When the person to be arrested is a prisoner who has escaped from a penal
establishment or place where he is serving final judgment or temporarily confined while his case
is pending, or has escaped while being transferred from one confinement to another. Paragraphs
(a) and (b) are clearly inapplicable. Paragraph (b) is also not in point because the policemen who
later arrested Enrile at his house had no personal knowledge that he was the source of the
marijuana. According to the policemen themselves, what happened was that they asked
Abugatal who gave him the marijuana and were told it was Enrile. It was for this reason that they
proceeded to Enrile’s house and immediately arrested him. What the policemen should have
done was secure a search warrant on the basis of the information supplied by Abugatal and then,
with such authority, proceeded to search and, if the search was fruitful, arrest Enrile. They had
no right to simply force themselves into his house on the bare (and subsequently disallowed)
allegations of Abugatal and bundle Enrile off to the police station as if he had been caught in
flagrante delicto. The discovery of the marked money on him did not mean he was caught in the
act of selling marijuana. The marked money was not prohibited per se. Even if it were, that fact
alone would not retroactively validate the warrantless search and seizure.

63. People vs. Pasudag [GR 128822, 4 May 2001]


First Division, Pardo (J): 4 concur

Facts: On 26 September 1995, at around 1:30 p.m., SPO2 Pepito Calip of the PNP Sison,
Pangasinan, went to Brgy. Artacho to conduct anti-jueteng operations. He urinated at a bushy
bamboo fence behind the public school. About 5 meters away, he saw a garden of about 70
square meters. There were marijuana plants in between corn plants and camote tops. He
inquired from a storekeeper nearby as to who owned the house with the garden. The storeowner
told him that Alberto Pasudag y Bokang owned it. SPO2 Calip went to the Police Station and
reported to Chief of Police Romeo C. Astrero. The latter dispatched a team (composed of SPO2
Calip, SPO3 Fajarito, SPO3 Alcantara and PO3 Rasca) to conduct an investigation. At around 2:30
p.m., the team arrived at Brgy. Artacho and went straight to the house of Pasudag. SPO3 Fajarito
looked for Pasudag and asked him to bring the team to his backyard garden which was about 5
meters away. Upon seeing the marijuana plants, the policemen called for a photographer, who
took pictures of Pasudag standing beside one of the marijuana plants. They uprooted 7
marijuana plants. The team brought Pasudag and the marijuana plants to the police station. On
17 December 1996, 4th Assistant Provincial Prosecutor of Pangasinan Emiliano M. Matro filed
with the Regional Trial Court, Pangasinan, Urdaneta an Information charging Pasudag with
violation of RA 6425, Sec. 9. On 10 February 1997, the trial court arraigned the accused. He
pleaded not guilty. Trial ensued. The Regional Trial Court, Pangasinan, Branch 46, Urdaneta
found Pasudag guilty beyond reasonable doubt of illegal cultivation of marijuana and sentenced
him to reclusion perpetua and to pay a fine of P500,000.00, without subsidiary penalty and other
accessories of the law. Pasudag appealed.

Issue: Whether time was of the essence to uproot and confiscate the marijuana plants.

Held: As a general rule, the procurement of a search warrant is required before a law enforcer
may validly search or seize the person, house, papers or effects of any individual. The
Constitution provides that “the right of the people to be secure in their persons, houses, papers
and effects against unreasonable searches and seizures of whatever nature and for any purpose
shall be inviolable.” Any evidence obtained in violation of this provision is inadmissible. Herein,
the police authorities had ample opportunity to secure from the court a search warrant. SPO2
Pepito Calip inquired as to who owned the house. He was acquainted with marijuana plants and
immediately recognized that some plants in the backyard of the house were marijuana plants.
Time was not of the essence to uproot and confiscate the plants. They were three months old
and there was no sufficient reason to believe that they would be uprooted on that same day.
With the illegal seizure of the marijuana plants, the seized plants are inadmissible in evidence
against Pasudag.

64. People vs. Plana [GR 128285, 27 November 2001]


En Banc, Per Curiam. 15 concur.

Facts: On 23 September 1994, at around 10:30 a.m., Felix Lagud was walking at the feeder road
in Barangay Cobe, Dumarao, Capiz. He just came from his farm in Alipasyawan, Dumarao and
was on his way home to Poblacion Ilawod. A movement at about 50 meters to his left side caught
his attention. He saw 3 persons who seemed to be wrestling. He came nearer so he would be
able to see them more clearly. From about a distance of 20 meters, he saw the 3 men holding a
girl while another man was on top of her. The girl was being raped and she was later stabbed.
Frightened that the assailants would see him, Lagud ran away. He intended to go straight home
but when he passed by the house of Porferio Haguisan, the latter invited him for a “milagrosa.”
Lagud obliged and stayed at the house of his “kumpare” until 2:00 a.m. On 26 September 1994,
the victim, Helen Perote, was found dead by her brother and the police in Brgy. Cobe, Dumarao,
Capiz. The body was in prone position and was already in an advance state of decomposition.
Lagud identified Antonio Plana (@ “Catong”), Edgardo Perayra and Rene Saldevea as the three
men who were holding the girl while their fourth companion was raping her. At the time of the
incident, he did not yet recognize the fourth man who was on top of the girl. However, when he
saw Plana, et. al. at the municipal hall where they were brought when they were arrested on 26
September 1994, he identified the fourth man to be Richard Banday. On the other hand, per the
post mortem examination conducted by Dr. Ricardo Betita, Rural Health Officer of Cuartero,
Capiz, the victim sustained the following injuries: (1) Clean edges stab wound 2×5 cm left
anterior chest; (2) Avulsion with irregular edges wound 8×12 cm middle chest area; (3) Avulsion
of the nose and upper lip portion/area; (4) Clean edges wound or stab wound 2×5 cm epigastric
area; (5) Clean edges stab wound 2×5 cm left hypogastric area; (6) Clean edges stab wound 2×5
cm hypogastric area; (7) Clean edges stab wound 2×5 cm left posterior upper back; (8) Clean
edges stab wound 2×5 cm mid upper portion of the back; (9) Clean edges stab wound 2×5 cm
left posterior back level of 8th ribs; (10) Clean edges stab wound 2×5 cm left back level of left
lumbar area; (11) Clean edges wound 2×5 cm middle low back area; (12) Clean edges wound
2×5 cm right low back area at level of lumbar area; (13) Clean edges wound 2×5 cm left gluteal
area near the anus; (14) Vagina: Introitus can easily insert 2 fingers/Hymen with laceration 3 and
9 o’clock (old laceration) and on the state of decomposition; and that the most probable cause of
death was massive hemorrhage or blood loss secondary to multiple stab wounds. According to
Dr. Betita, the victim died more than 72 hours already before the police authorities found her
body. An information was filed against Plana, et. al. for the crime of rape with homicide before
the Regional Trial Court, Branch 15 of Roxas City (Criminal Case 4659). At their arraignment,
Plana, et. al. pleaded not guilty. On 23 November 1996, after due trial, a judgment was rendered
by the trial court finding Plana, et. al. guilty beyond reasonable doubt of the crime of rape with
homicide. The trial court imposed upon them the supreme penalty of death, and ordered them to
pay jointly and severally the heirs of the victim, Helen Perote, P25,000.00 as actual damages and
P50,000.00 as civil liability. Hence, the automatic review.

Issue: Whether the trial court erred in not censuring the actuation of the police authorities in
detaining Plana, et. al. without benefit of Court filed information nor judicial order of detention as
well as violation of their constitutional rights during their so-called custodial invitation and
interrogation.

Held: Plana, et. al. already waived their right to question the irregularity, if any, in their arrest.
They respectively entered a plea of “not guilty” at their arraignment. By so pleading, they
submitted to the jurisdiction of the trial court, thereby curing any defect in their arrest, for the
legality of an arrest affects only the jurisdiction of the court over their persons.

65. People vs. Conde [GR 113269, 10 April 2001]


Second Division, Quisumbing (J): 4 concur

Facts: On 25 May 1992 at about 8:00 A.M., Apollo Romero was home sitting by the window and
drinking coffee when he saw 4 men in Santolan Street block the path of 2 Indian nationals
(bombay) on a motorcycle. Oscar Conde y Lutoc poked a gun at the two Indians while his three
companions (Alejandro Perez Jr. y Carsillar, Allan Atis y Abet, and another unidentified man)
approached and stabbed the Indians. Atis took the goods which were being sold by the two
Indians on installment. After the stabbing, the four men fled from the crime scene towards
Mabolo Street. PO3 Rodencio Sevillano of the Intelligence and Investigation Division (IID) of the
PNP, Kalookan City investigated the incident. On 30 May 1992, the police arrested Conde, Perez
and Atis. Police recovered the weapons used in the robbery, when Felicidad Macabare, Conde’s
wife, went to the police station to talk to Conde. These weapons were discovered inside her bag
after a routine inspection. Sevillano admitted, however, that they did not have a warrant of
arrest when they apprehended the accused. Nor did they have a search warrant when they
inspected Felicidad’s bag and when they searched the house of a certain Jimmy where they
found the stolen items. Conde, Perez and Atis were charged with the crime of robbery with
homicide. The accused entered pleas of not guilty. On 15 December 1993 the Regional Trial
Court, Branch 129, Kalookan City found Conde, Atis and Perez guilty of the special complex crime
of robbery with homicide and sentenced each of them to suffer the penalty of reclusion perpetua
with the accessory penalties under the law, and to jointly and severally indemnify the heirs of
each of the victims, Sukhdev Singh and Biant Singh, in the amount of P50,000.00. Conde, et. al.
appealed. However, the counsel de parte for Perez, Atty. Jose M. Marquez, failed to file brief for
Perez, prompting this Court to dismiss his appeal. The decision of the trial court became final and
executory with respect to Perez. Hence the present appeal concerns only Atis and Conde, who
filed their separate briefs.

Issue: Whether the illegal warrantless arrest, which was waived, is sufficient cause for setting
aside a valid judgment rendered upon a sufficient complaint after trial free of error.

Held: The arrests of Conde, et. al. came after the lapse of 5 days from the time they were seen
committing the crime. At the time they were arrested, the police were not armed with any
warrants for their arrests. Section 5 of Rule 113, of the Revised Rules of Criminal Procedure 27
enumerates the instances when an arrest can be made without warrant, namely: (a) When, in his
presence the person to be arrested has committed, is actually committing, or is attempting to
commit an offense; (b) When an offense has in fact just been committed, and he has probable
cause to believe based on personal knowledge of facts or circumstances that the person to be
arrested has committed it; and (c) When the person to be arrested is a prisoner who has escaped
from a penal establishment or place where he is serving final judgment or temporarily confined
while his case is pending, or has escaped while being transferred from one confinement to
another. None of the above circumstances is present herein. Conde, et. al. were merely walking
along Tandang Sora Avenue and were not committing any crime. Neither can it be said that the
crime had just been committed as 5 days had already passed from the time of the robbery with
homicide. It cannot also be said that the arresting officers had probable cause based on personal
knowledge, as PO3 Sevillano admitted that they learned about the suspects from Apollo Romero
and certain unnamed informants. Further, the lapse of 5 days gave the police more than enough
time to conduct surveillance of the appellants and apply for a warrant of arrest. Clearly, the
rights of Conde, et. al., provided in Sec. 2, Art. III of the Constitution 28 were violated.
Unfortunately, they did not assert their constitutional rights prior to their arraignment. This is
fatal to their case. An accused is estopped from assailing the legality of his arrest if he failed to
move for the quashing of the Information against him before his arraignment. When they entered
their pleas on arraignment without invoking their rights to question any irregularity, which might
have accompanied their arrests, they voluntarily submitted themselves to the jurisdiction of the
court and the judicial process. Any objection, defect, or irregularity attending their arrests should
had been made before they entered their pleas. It is much too late for them to raise the question
of their warrantless arrests. Their pleas to the information upon arraignment constitute clear
waivers of their rights against unlawful restraint of liberty. Furthermore, the illegal arrest of an
accused is not sufficient cause for setting aside a valid judgment rendered upon a sufficient
complaint after trial free from error. The warrantless arrest, even if illegal, cannot render void all
other proceedings including those leading to the conviction of the appellants and his co-accused,
nor can the state be deprived of its right to convict the guilty when all the facts on record point
to their culpability.

You might also like